Вы находитесь на странице: 1из 363

Princeton University

Ph501 Electrodynamics Problem Set 1


Kirk T. McDonald (1998) kirkmcd@princeton.edu
http://puhep1.princeton.edu/~mcdonald/examples/

References: R. Becker, Electromagnetic Fields and Interactions (Dover Publications, New York, 1982). D.J. Griths, Introductions to Electrodynamics, 3rd ed. (Prentice Hall, Upper Saddle River, NJ, 1999). J.D. Jackson, Classical Electrodynamics, 3rd ed. (Wiley, New York, 1999). The classic is, of course: J.C. Maxwell, A Treatise on Electricity and Magnetism (Dover, New York, 1954). For greater detail: L.D. Landau and E.M. Lifshitz, Classical Theory of Fields, 4th ed. (ButterworthHeineman, Oxford, 1975); Electrodynamics of Continuous Media, 2nd ed. (ButterworthHeineman, Oxford, 1984). N.N. Lebedev, I.P. Skalskaya and Y.S. Ulfand, Worked Problems in Applied Mathematics (Dover, New York, 1979). W.R. Smythe, Static and Dynamic Electricity, 3rd ed. (McGraw-Hill, New York, 1968). J.A. Stratton, Electromagnetic Theory (McGraw-Hill, New York, 1941). Excellent introductions: R.P. Feynman, R.B. Leighton and M. Sands, The Feynman Lectures on Physics, Vol. 2 (Addison-Wesely, Reading, MA, 1964). E.M. Purcell, Electricity and Magnetism, 2nd ed. (McGraw-Hill, New York, 1984). History: B.J. Hunt, The Maxwellians (Cornell U Press, Ithaca, 1991). E. Whittaker, A History of the Theories of Aether and Electricity (Dover, New York, 1989). Online E&M Courses: http://www.ece.rutgers.edu/~orfanidi/ewa/ http://farside.ph.utexas.edu/teaching/jk1/jk1.html

Princeton University 1998

Ph501 Set 1, Problem 1

1. (a) Show that the mean value of the potential over a spherical surface is equal to the potential at the center, provided that no charge is contained within the sphere. (A related result is that the mean value of the electric eld over the volume of a charge-free sphere is equal to the value of the eld at its center.) (b) Demonstrate Earnshaws theorem: A charge cannot be held at equilibrium solely by an electrostatic eld.1 (c) Demonstrate that an electrostatic eld E cannot have a local maximum of E 2, using the mean value theorem mentioned in part (a) or any other technique. Remark: An interesting example of nonelectrostatic equilibrium is laser trapping of atoms. Briey, an atom of polarizability takes on an induced dipole moment p = E in an electric eld. The force on this dipole is then (Notes, p. 26), F = (p E) = E 2 . Since an electrostatic eld cannot have a local maximum of E 2 , it cannot trap a polarizable atom. But consider an oscillatory eld, in particular a focused light wave. The time-average force, F = E 2 draws the atom into the laser focus where the electric eld is a maximum. See, http://puhep1.princeton.edu/~mcdonald/examples/tweezers.pdf

http://puhep1.princeton.edu/~mcdonald/examples/EM/earnshaw_tcps_7_97_39.pdf

Princeton University 1998

Ph501 Set 1, Problem 2

2. Calculate the potential (z ) along the axis of a disk of radius R in two cases: (a) The disk is a uniform layer of charge density , and (b) The disk is a uniform dipole layer of dipole moment density p = p z per unit area.

Princeton University 1998

Ph501 Set 1, Problem 3

3 1, rather then

3. Suppose the electric eld of point charge q were E = q r/r2+ where 2 E = q r/r .

(a) Calculate E and E for r = 0. Find the electric potential for such a point charge. (b) Two concentric spherical conducting shells of radii a and b are joined by a thin conducting wire. Show that if charge Qa resides on the outer shell, then the charge on the inner shell is Qb Qa [2b ln 2a (a + b) ln(a + b) + (a b) ln(a b)] 2(a b) (1)

Princeton University 1998

Ph501 Set 1, Problem 4

4. (a) Starting from the dipole potential = p r/r3 explicitly show that E= 3(p r) r p 4 p 3 (r ). 3 r 3 (2)

Hint: to show the need for the 3(r) term, consider the volume integral of E over a small sphere about the dipole. You may need a variation of Gauss theorem:
V

dVol =

dS, n

(3)

is the outward normal to the surface. where n (b) The geometric denition of the lines of force is that this family of curves obeys the dierential equation: dy dz dx = = . (4) Ex Ey Ez For a dipole p = p z, nd the equation of the lines of force in the x-z plane. It is easiest to work in spherical coordinates. Compare with the gure on the cover of the book by Becker.

Princeton University 1998

Ph501 Set 1, Problem 5

5. Find the two lowest-order nonvanishing terms in the multipole expansion of the potential due to a uniformly charged ring of radius a carrying total charge Q. Take the origin at the center of the ring, and neglect the thickness of the ring.

Princeton University 1998

Ph501 Set 1, Problem 6

6. (a) A long, very thin rod of dielectric constant is oriented parallel to a uniform electric eld Eext. What are E and D inside the rod? (b) What are E and D inside a very thin disc of dielectric constant perpendicular to Eext? if the disc is

(c) Find E and D everywhere due to a sphere of xed uniform polarization density P. Then calculate E D dVol for the two volumes inside and outside the spheres surface. Hint: this problem is equivalent to two oppositely charged spheres slightly displaced. (d) Show that for any nite electret, a material with xed polarization P, E D dVol = 0.
all space

(5)

Princeton University 1998

Ph501 Set 1, Problem 7

7. A spherical capacitor consists of two concentric conducting shells of radii a and b. The gap is half lled with a (non-conducting) dielectric liquid of constant . You may assume the elds are radial. The inner shell carries charge +Q, the outer shell Q. Calculate E and D in the gap, and the charge distribution in the inner shell. Also calculate the capacitance, dened as C = Q/V , where V is the potential dierence between the inner and outer shells.

Princeton University 1998

Ph501 Set 1, Problem 8

8. (a) As a classical model for atomic polarization, consider an atom consisting of a xed nucleus of charge +e with an electron of charge e in a circular orbit of radius a about the nucleus. An electric eld is applied at right angles to the plane of the orbit. Show that the polarizability is approximately a3. (This happens to be the result of Beckers (26-6), but the model is quite dierent!) Assuming that radius a is the Bohr radius, 5.3 109 cm, use the model to estimate the dielectric constant of hydrogen gas at S.T.P. Empirically, 1 + 2.5 104 . (b) Another popular classical model of an atom is that the electron is bound to a neutral nucleus by a spring whose natural frequency of vibration is that of some characteristic spectral line. For hydrogen, a plausible choice is the Lyman line at 1225 Angstroms. In this model, show that = e2/m 2 , and estimate . Recall that e = 4.8 1010 esu, and m = 9.1 1028 g.

Princeton University 1998

Ph501 Set 1, Solution 1 Solutions

1. a) We oer two solutions: the rst begins by showing the result holds for small spheres, and then shows the result is independent of the size of the (charge-free) sphere; the second applies immediately for spheres of any size, but is more abstract. We consider a charge-free sphere of radius R centered on the origin. In a charge-free region, the potential (r) satises Laplaces equation: 2 = 0. First, we simply expand the potential in a Taylor series about the origin: (r) = (0) +
i

(6)

(0) 1 xi + xi 2

i,j

2(0) xixj + ... xixj

(7)

We integrate (7) over the surface of the sphere: (r)dS = 4R2 (0) +
i

(0) xi

xi dS +

1 2

i,j

2(0) xixj

xi xj dS + ...

(8)

For a very small sphere, we can ignore all terms except the rst, In this case, eq. (8) becomes 1 (r)dS = (0), [R small], (9) 4R2 S which was to be shown. Does the result still hold at larger radii? One might expect that since small is not well dened, (9) holds for arbitrary R, so long as the sphere is charge free. Progress can be made staying with the Taylor expansion. By spherical symmetry, the integral of the product of an odd number of xi vanishes. Hence, only the terms with even derivatives of the potential survive. And of these, only some terms survive. In 2 2 particular, for the 2nd derivative, only the integrals of x2 1 , x2 , and x3 survive, and these 3 are all equal. Thus, the 2nd derivative term consists of 1 2 2(0) 2(0) 2(0) + + x2 x2 x2 1 2 3 x2 1 dS, (10)

which vanishes, since 2(0) = 0. It is less evident that the terms with 4rth and higher even derivatives vanish, although this can be shown via a systematic multipole expansion in spherical coordinates, which emphasizes the spherical harmonics Ylm . But by a dierent approach, we can show that the mean value of the potential over a charge-free sphere is independent of the radius of the sphere. That is, consider M (r ) = 1 4r2
S

dS =

1 4

d cos

d(r, , ),

(11)

Princeton University 1998

Ph501 Set 1, Solution 1

10

in spherical coordinates (r, , ). Then dM (r) 1 1 d cos d d cos d r = = dr 4 r 4 1 1 dS = 2 dVol = 0, = 2 4r S 4r2 V

(12)

for a charge-free volume. Hence, the mean value of the potential over a charge-free sphere of nite radius is the same as that over a tiny sphere about the center of the larger sphere. But, as shown in the argument leading up to (9), this is just the value of the potential at the center of the sphere. A second solution is based on one of Greens theorems (sec. 1.8 Of Jackson). Namely, for two reasonable functions (r) and (r), 2 2 dVol = dS, n n (13)

where n is coordinate normally outward from the closed surface S surrounding a volume V. With as the potential satisfying Laplaces equation (6), the second term on the l.h.s. of (13) vanishes. We seek an auxiliary function such that 2 = 3 (0), so the l.h.s. is just (0). Further, it will be helpful if vanishes on the surface of the sphere of radius R, so the second term on the r.h.s. vanishes also. These conditions are arranged with the choice = 1 4 1 1 , R r (14)

recalling pp. 8-9 of the Notes. On the surface of the sphere, coordinate n is just the radial coordinate r, so 1 = . (15) n 4r2 Thus, we can evaluate the expression (13), and get: (0) = 1 4R2 dS, (16)

which means that the value of the potential at the center of a charge-free sphere of any size is the average of the potential on the surface of the sphere. b) The potential energy of a charge q at point r, due to interaction with an electrostatic eld derivable from a potential (r), is: U = q(r). (17)

For the point r0 to be the equilibrium point for a particle, the potential should have a minimum there. But we can infer from part (a) that:

Princeton University 1998

Ph501 Set 1, Solution 1

11

Harmonic functions do not have minima, harmonic functions being a name for solutions of Laplaces equation (6). Indeed, a minimum of U at point r0 would imply that there is a small sphere centered on r0 such that (r0) is less than at any point on that sphere. This would contradict what we have shown in part (a): (r0 ) is the average of over the sphere. We continue with an example of Earnshaws theorem. Consider 8 unit charges located at the corners of a cube of edge length 2, i.e., the charges are at the locations (xi , yi, zi ) = (1,1,1), (1,1,-1), (1,-1,1), (1,-1, -1), (-1,1,1), (-1,1,-1), (-1,-1,1), (-1,-1,1). It is suggestive, but not true, that the electric eld near the origin points inwards and could trap a positive charge. The symmetry of the problem is such that a series expansion of the electric potential near the origin will have terms with only even powers, and we must go to 4rth order to see that the potential does not have a maximum at the origin. To simplify the series expansion, we consider the electric eld, for which we need expand only to third order. The electric potential is given by
8

=
i=1

1 (xi x)2 + (yi y )2 + (zi z )2

(18)

The x component of the electric eld is Ex = =


8 xi x = 2 2 2 3/2 x i=1 [(xi x) + (yi y ) + (zi z ) ]

1 33/2 1 33/2 1 33/2 1

8 i=1 8 i=1 8 i=1 8

xi x [1 + (2xi x 2yi y 2zi z + x2 + y 2 + z 2 )/3]3/2 xi 1 + yi y + z i z + xi 2x2 + y 2 + z 2 + 5yi yzi z 3

11yi y 3 + 11zi z 3 x2yi y + x2zi z 3y 2zi z + 3z 2 yi y + 54 6 2

33/2 i=1 28 (x3 9xy 2 9xz 2 ), = 81 3


2 2 noting that x2 i = yi = zi = 1 and that

2xyi y + 2xzi z 7x3 7xy 2 + 7xz 2 20xyi yziz + + 3 54 6 9 (19) xi = 0 = and Ez xi yi , etc. Similarly, 28 (z 3 9zx2 9zy 2). 81 3 (20)

Ey

28 (y 3 9yx2 9yz 2 ) 81 3

The radial component of the electric eld is therefore Er = xEx + yEy + zEz 28 Er = [x4 + y 4 + z 4 18(x2 y 2 + y 2z 2 + z 2x2)]. (21) r r 81 3r

Princeton University 1998

Ph501 Set 1, Solution 1

12

Along the x axis, r = x and the radial eld varies like 28 3 r > 0, 81 3 but along the diagonal x = y = z = r/ 3 it varies like Er Er 476 3 r < 0. 243 3 (22)

(23)

It is perhaps not intuitive that the electric eld is positive along the positive x axis, although Earnshaw assures us that the radial electric eld must be positive in some direction. A clue is to consider the point (1,0,0) on the face of the cube whose corners hold the charges. At this point the electric elds due to the 4 charges with x = 1 sum to zero, so the eld here is due only to the 4 charges with x = 1, and now obviously the x component of the electric eld is positive. The charges at the corners of the cube force a positive charge toward the origin along the diagonals, but cannot prevent that charge from escaping near the centers of the faces of the cube. c) If E 2 has a local maximum at some point P in a charge-free region, then there is a nonzero r such that E 2 < E 2 (P ) for all points (other than P ) within a sphere of radius r about P . Consequently, E < E (P ) in that sphere. Let z point along E(P ). Then the mean-value theorem can be written 4r3 E (P ), Ez dVol = 3 (24)

for the sphere about P . In general, Ez E , and by assumption E < E (P ) for all points other than P within the sphere, so Ez dVol EdVol < E (P )dVol = 4r3 E (P ), 3 (25)

which contradicts eq. (24). Hence, E 2 cannot be locally maximal at P . However, E 2 can take on a local minimum....

Princeton University 1998

Ph501 Set 1, Solution 2

13

2. a) The potential (z ) along the axis of a disk of radius R of charge density (per unit area) is given by the integral:
R

(z ) =
r =0

2rdr

= 2 R2 + z 2 |z | . r2 + z 2

(26)

R, which is consistent with the obNotice that (z ) behaves as R2 /|z | for |z | servation that in this limit the disk may be considered as a point charge q = R2 . Notice also, that at z = 0 the potential is continuous, but its rst derivative (E) jumps from 2 at z = 0+ to 2 at z = 0 . This reects the fact that at small |z | (| z | R) the potential may be calculated, in rst approximation, as the potential for the innite plane with charge density . can be thought of as composed of a layer b) A disk of dipole-moment density p = pz of charge density + separated in z from a layer of charge density by a distance d = p/ . Say, the + layer is at z = d/2, and the layer is as z = d/2. Then, the potential b at distance z along the axis could be written in terms of a (z ) found in (26) as b (z ) = a (z d/2) a(z + d/2) d a (z ) z . (27) = 2p sign(z ) 2 z R + z2

In this, we have taken the limit as d 0 while , but p = d is held constant. At large z we get the potential of a dipole P = pR2 on its axis. But near the plate (| z | R), the potential has a discontinuity: b (0+ ) b (0 ) = 4p. (28)

We may explain this in our model of the dipole layer as a system of two close plates with charge density and distance d between them, where p = d. The eld between the plates is Ez = 4 , and the potential dierence potential between two plates is = Ez d = 4p, as found in (28). (29)

Princeton University 1998

Ph501 Set 1, Solution 3

14

3. a) For a charge q at the origin, the proposed electric eld is r r E = q 2+ = q 3+ . (30) r r This still has spherical symmetry, so we can easily evaluate the divergence and curl in spherical coordinates: 1 3q 3+ q r (31) E = q 3+ + q r 3+ = 3+ q 3+ = 3+ ; r r r r r 1 r r (32) E = q 3+ + 3+ q r = 0 (3 + ) 5+ q r = 0. r r r Since E = 0, the eld can be derived from a potential. Indeed,
r r

E = ,

where

Er dr = q

q dr 1 = . 2+ r + 1 r +1

(33)

b) Let us rst compute the potential due to a the spherical shell of radius a that carries charge Q, as observed at a distance r from the center of the sphere. We use (33) and integrate in spherical coordinates (r, , ) to nd (r ) = Q 1 2 4a 1 +
1 1

2ar2 d cos (a2 + r2 2ar cos )


1+ 2

Q (a + r)1 |a r|1 . 2ar 1 2

(34)

Now consider the addition of a sphere of radius b < a that carries charge Qb . The total potential at r = a is a = while that at r = b is b = Qa (a + b)1 (a b)1 Qb (2b)1 + . 2ab 2b2 1 2 1 2 (36) Qa (2a)1 Qb (a + b)1 (a b)1 + , 2a2 1 2 2ab 1 2 (35)

We require Qb such that a = b , (as guaranteed by the wire connecting the two spheres). However, we neglect terms of O(2 ), assuming to be small. Then, (35-36) lead to the relation (b/a)(2a)1 (a + b)1 + (a b)1 Q b = Q a (37) (a/b)(2b)1 + (a + b)1 (a b)1 What about the factors of form x , which are approximately 1 for small ? Let x 1 + . Taking logarithms, ln x ln(1 + ) . Thus, x x 1 + ln x, so x1 x(1 ln x). (38) 1 + ln x Using (38) in (37), we nd Qb = Qa [2b ln 2a (a + b) ln(a + b) (a b) ln(a b)] . 2(a b) (39)

Measurement of the ratio Qb/Qa provides a stringent test of the accuracy of the 1/r2 law for electrostatics.

Princeton University 1998

Ph501 Set 1, Solution 4

15

4. a) The rst terms in E can be obtained by explicit dierentiation, perhaps best done using vector components. For r > 0, Ei = pj xj 3(pj xj )xi pi = 3. 3 5 xi r r r (40)

To justify the -term, consider the integral over a small sphere surrounding the (point) dipole: pr dS = EdVol = dVol = n rdS, (41) V V S S r3 according to the form of Gauss law (3) given in the hint. Evaluating the last integral in component a spherical coordinate system with z axis along p, we nd that only the p is nonzero: 1 4p . (42) EdVol = p 2d cos cos2 = 3
1

No matter how small the sphere, the integral (42) remains the same. On the other hand, if we insert the eld E from (40) in the volume integral, only the z component (along p) does not immediately vanish, but then
r V 1

Ez dVol =
0

2rdr
1

d cos

p(3 cos 2 1) = 0, r3

(43)

if we adopt the convention that the angular integral is performed rst. To reconcile the results (42) and (43), we write that the dipole eld has a spike near the origin symbolized by (4p/3) 3(r). Another qualitative reason for the -term is as follows. Notice, that without this term we would conclude that the electric eld on the axis of the dipole (z -axis) would always along +z . This would imply that if we moved some distribution of charge from large negative z to large positive z , then that charge would gain energy from the dipole eld. But this cannot be true: after all, the dipole may be thought as a system of two charges, separated by a small distance, and for such a conguration the potential at large distances is certainly extremely small. We can also say that the -term, which points in the p direction, represents the large eld in the small region between two charges that make up the dipole. b) In spherical coordinates with z along p, the dipole potential in the x-z plane is (r, ) = Then, Er = 2p cos , = r r3 and E = p sin . = r r3 (45) p cos r2 (44)

Princeton University 1998

Ph501 Set 1, Solution 4

16

Thus, the dierential equation of the eld lines dr rd = , Er E which integrates to r = C sin2 = C etc. x2 , r2 (47) implies dr d sin =2 , r sin (46)

Princeton University 1998

Ph501 Set 1, Solution 5

17

5. The multipole expansion of the potential about the origin due to a localized charge distribution (r) is Q P r 1 Qij r i r j + ... (48) (r ) = + 2 + 3 r r 2 r where Q= (r)dVol, Pi = ri dVol, Qij = (3ri rj ij r2 )dVol, . . . (49)

For the ring, Q is just the total charge, while the dipole moment P is zero because of the symmetry. Let us nd the quadrupole moment, Qij . Take the z axis to be along that of the ring. Then, the quadrupole tensor is diagonal, and due to the rotational invariance, 1 Qxx = Qyy = Qzz . 2 We compute Qxx in spherical coordinates (r, , ): Qxx = (3x2 r2 )dVol =
2 0

(50)

Qa2 Q 2 a (3 cos2 1)ad = . 2a 2

(51)

Thus, the third term in the expansion (48) is: Qa2 1 2 2 2 2 2 Q sin cos + Q sin sin + Q cos = (3 cos2 1). xx yy zz 2r3 4r3 (52)

Princeton University 1998

Ph501 Set 1, Solution 6

18

6. a) Remember that E is dened as a mean electric eld, due to both the external eld and the microscopic charges. In the presence of external eld Eext, polarization P is induced in the rod. Since E = 0, the tangential electric eld is continuous across the surface of the rod. This suggests that inside the rod, which is parallel to Eext, we have E = Eext. To check for consistency, note that in this case, P is constant apart from the ends. There is no net polarization charge in the bulk of the rod, and no change in the electric eld from Eext. But at the ends there are charges Q, where Q = P A, and A is the cross-sectional area of the rod. Since A is very small for the thin rod, and the ends of a long rod are far away from most of the rod, these charges do not contribute signicantly to the electric eld. Hence our hypothesis is satisfactory. The electric displacement can then be deduced as D = E = Eext inside the rod. b) For a dielectric disk perpendicular to Eext, the normal component of the elds is naturally emphasized. Recall that D implies that the normal component of the displacement D is continuous across the dielectric boundary. Outside the disk, where = 1, D = E = Eext is normal to the surface. (That E = Eext may be justied by noting that the induced charges on two surfaces of the thin disk have opposite signs and do not contribute to the eld outside of the disk.) Thus, inside the disk, D = Eext. Lastly, inside the disk E = D/ = Eext/ . c) The problem of a dielectric sphere of uniform polarization density P is equivalent to two homogeneous spheres with charge densities and , displaced by distance d, such that in the limit d 0, but d = P. Recall that for a sphere of uniform charge density , the interior electric eld is E= 4 r. 3 (53)

Thus, inside the polarized sphere we have E = lim The displacement is D = E + 4 P = Then, E DdVol =
inside

4 4 4 P 4 (r d) r = lim d = . 3 3 3 3 8P . 3

(54)

(55)

128 2 r3 P 2 4P 8P 4r3 = . 3 3 3 27

(56)

Outside the sphere, the electric eld is eectively due to two point charges q = 4r3 /3 separated by small distance d, where d = P. The external eld is simply that of a point dipole at the origin of strength p= 4r3 P . 3 (57)

Princeton University 1998 Then,

Ph501 Set 1, Solution 6

19

E= and D = E. Thus, E DdVol =


outside 4r2 dr 2 p r6 r 2 r

3 r (p r) p , r3

(58)

p2 + 3(p r )2 ) dVol r6 2r2 dr r6


1

(59) 8p2 128 3r3 P 2 = , 3r3 27

+ 3p

cos2 d cos =
1

and the sum of the inside and outside integrals vanishes. d) Let us show, on general grounds, that for an electret the integral of E D over the whole space is zero. First, note that D = 0 everywhere for an electret, and that the electric eld can be derived from a potential, . Then E DdVol =
all space V

DdVol =

(D)dVol =

D dS.

(60)

But if electret occupies nite volume, then 1/r at large r, as seen from multipole expansion; in the same limit, D 1/r2 , while dS r2 . So the integral over the surface at innity is zero. What would be dierent in this argument if the material were not an electret? In general, we would then have D = 4free, and the 3rd step in (60) would have the additional term V D = 4 V free = 8U . Thus, the usual electrostatic energy is contained in V E D/8 , as expected. This argument emphasizes that the work done in putting a eld on an ordinary dielectric can be accounted for using only the free charges (which establish D). One need not explicitly calculate the energy stored in the polarization charge distribution, which energy is accounted for via the modication to the potential in the presence of the dielectric. But the whole argument fails for an electret, which remains polarized (energized) even in the absence of a free charge distribution.

Princeton University 1998

Ph501 Set 1, Solution 7

20

7. In the upper half of the capacitor (where there is no dielectric) we have via Gauss law: Eup(r) = Dup (r) = 4Qup 2Qup = , 2r2 r2 (61)

where Qup is the charge on the upper half of the inner sphere, assuming the elds are radial. In the lower part we have: Edown(r) = Eup(r), (62) which follows from the continuity of the tangential component of E across the boundary between dielectric and vacuum. Also, Ddown (r) = Edown(r), and Ddown (r) = (63)

2Qdown , (64) r2 as follows from D = 4free, where Qdown is the charge on the lower part of the inner sphere. Combining (61-64), Qdown = Qup, holds for the free charge on the inner shell. What about the total charge distribution, which include polarization charges in the dielectric? The polarization vector P obeys 4 P = ( 1)E. Thus, the charge density, , which appears microscopically on the boundary of the dielectric adjacent = 4 P n to the inner shell, equals 1 times the charge density on the upper shell (since = n r). In other words, the total microscopic charge densities on the lower and upper parts of the shell are equal. This ensures that the electric eld is the same in the upper and lower part of the capacitor. Of course, Qdown = Q Qup. From (65) and (67), Q Qup = Qup , and hence, Q . 1+ This implies that the potential dierence V between the shells is Qup =
b

(65)

(66) (67) (68)

V =
a

2Q Edr = 1+

b a

2Q dr = 2 r 1+

1 1 . b a

(69)

So, the capacitance is C= 1 + ab Q = . V 2 ab (70)

Princeton University 1998

Ph501 Set 1, Solution 8

21

8. a) In the presence of an electric eld along the z direction, which is perpendicular to the plane of the orbit of our model atom, the plane is displaced by a distance z . We calculate this displacement from the equilibrium condition: the axial component of the Coulomb force between the electron and the proton should be equal to the force eE on the electron of charge e due to the electric eld. Namely, Fz = e2 z = eE, r2 r (71)

where r = a2 + z 2, and a is the radius of the orbit of the electron. The induced atomic dipole moment p is (72) p = ez = r3 E a3E, where the approximation holds for small displacements, i.e., small electric elds. Since p = E in terms of the atomic polarizability , we estimate that a3 , where a is the radius of the atom. The dielectric constant is related to the atomic polarizability via 1 = 4N, (74) (73)

where N is the number of atoms per cm3. For hydrogen, there are 2 atoms per molecule, and 6 1023 molecules in 22.4 liters, at S.T.P. Hence N = 2(6 1023 )/(22.4 103 ) = 5.4 1019 atoms/cm3 . Estimating the radius a as the Bohr radius, 5.3 109 cm, we nd (75) 1 4 (5.4 1019 )(5.3 109 )3 1.0 104 . b) If an electric eld E is applied to the springlike atom, then the displacement d of the electron relative to the xed (neutral) nucleus is related by F = kd = eE , where k = m2 is the spring constant in terms of characteristic frequency . The induced dipole moment p is given by p = ed = Thus the polarizability is given by = e2 . m 2 (77) e2 E e2 E = . k m 2 (76)

The frequency that corresponds to the Lyman line at 1225 A is 2 (3 1010 ) 2c = = 1.54 1016 Hz. = 1225 108 From (77), (4.8 1010 )2 = 1.07 1024 cm3 . = 28 16 2 (9.1 10 )(1.54 10 )

(78)

(79)

Princeton University 1998 Finally,

Ph501 Set 1, Solution 8

22

1 = 4N = 4 (5.4 1019 )(1.07 1024 ) = 7.3 104

(80)

Thus, our two models span the low and high side of the empirical result. Of course, we have neglected the fact that the hydrogen atoms are actually paired in molecules.

Princeton University

Ph501 Electrodynamics Problem Set 2


Kirk T. McDonald (1998) kirkmcd@princeton.edu http://puhep1.princeton.edu/~mcdonald/examples/

Princeton University 1998

Ph501 Set 2, Problem 1

1. Show that the electromagnetic energy of a dielectric subject to elds E and D = E is U= 1 8 E D dVol, (1)

by considering the model of atoms as springs (Problem 8b, set 1). The energy U then has two parts: 1 E2 dVol, (2) U1 = 8 stored in the electric eld, and U2 = n kx2 dVol, 2 (3)

stored in the spring-like atoms (n is the number of atoms per unit volume). Assume n is small so that the dielectric constant is nearly 1.

Princeton University 1998

Ph501 Set 2, Problem 2

2. (a) Show that the energy of a quadrupole in an external electric eld E, 1 Ej Uquad = Qij , 6 xi in terms of its quadrupole tensor Qij , can be rewritten as Uquad = Qxx Ex , 4 x (5) (4)

if the quadrupole is rotationally symmetric about the x axis. Give an expression for the force F on the quadrupole. (b) A rotationally symmetric quadrupole of strength Qxx (zero net charge, zero dipole moment) is located at distance r from a point charge q . What is the force on the quadrupole if: i. The x axis is along the line joining Qxx and q ? ii. The x axis is perpendicular to the line joining Qxx and q ? For your own edication, conrm your answer by considering the simple quadrupole:
t y t

q 1

2q1

q 1

Princeton University 1998

Ph501 Set 2, Problem 3

3. The principle of an electrostatic accelerator is that when a charge e escapes from a conducting plane that supports a uniform electric eld of strength E0, then the charge gains energy eE0 d as it moves distance d from the plane. Where does this energy come from? Show that the mechanical energy gain of the electron is balanced by the decrease in the electrostatic eld energy of the system.

Princeton University 1998

Ph501 Set 2, Problem 4

4. (a) Two point dipoles of strength p are aligned along their line of centers, and distance 2d apart. Calculate the force between the dipoles via F = (p )E, and by means of the Maxwell stress tensor. (b) A spherical conducting shell of radius a carries charge q . It is in a region of zero external eld. Calculate the force between two hemispheres in two dierent ways.

Princeton University 1998

Ph501 Set 2, Problem 5

5. (a) Two coaxial pipes of radii a and b (a < b) are lowered vertically into an oil bath:

If a voltage V is applied between the pipes, show that the oil rises to height h= ( 1)V 2 4g ln
b a

(b 2 a 2 )

(6)

where g is the acceleration due to gravity. (b) Recalling prob. 1(c) of set 1, discuss qualitatively how the force arises the pulls the liquid up into the capacitor.

Princeton University 1998

Ph501 Set 2, Problem 6

6. According to a theorem of Green, the potential (x) in the interior of a volume V can be deduced from a knowledge of the charge density (x) inside that volume plus knowledge of the potential and the normal derivative /n of the potential on the surface S that bounds the volume, (x ) = (x ) 1 dVol + R 4 (x ) n 1 1 (x ) R R n dS , (7)

where R = |x x | is the distance between the point of observation and the element of the integrand. However, further insights of Green indicate that it suces to specify only one of or /n on the bounding surface to determine the potential within. As a particular example, show that the potential within a charge-free sphere of radius a, centered on the origin, can be determined from knowledge of only the potential on its surface according to (Poisson, 1820) (x ) = a 2 x2 4a (x ) dS . R3 (8)

Green (1828) gave a derivation of Poissons integral (8) that can be generalized to many other problems in electrostatics. Recall that a key step towards eq. (7) is the identity ( ) dVol = =
S

(2 2 ) dVol n n dS. (9)

( ) dS =

For problems in which the interior of volume V is charge free the potential obeys 2 = 0 there. To have a nonzero potential inside V there must, of course, be charges on the surface of or exterior to volume V . If function also obeys 2 = 0 inside V (and so might be the potential for some other distribution of charges exterior to V ), then the identity (9) reduces to 0=
S

n n

dS.

(10)

Hence, we could combine eqs. (7) and (10) to yield the relation (x) = 1 4 1 = 4 1 1 (x ) + + dS n R R n G(x, x ) (x ) (x ) G(x, x ) dS , n n (x ) 1 + . R

(11)

where G(x, x ) =

(12)

Princeton University 1998

Ph501 Set 2, Problem 6

IF the Greens function G(x, x ) vanishes on the surface S , then we have the desirable relation between the potential in the interior of V and its value on the bounding surface S , 1 G(x, x ) (x ) = (x ) dS . (13) 4 S n Green noted that the auxiliary potential can be thought of as due to exterior charges that bring the surface S to zero potential when there is unit charge at position x inside volume V , and G as the total potential of that charge conguration. Further, we may think of the bounding surface S as being a grounded conductor for the purposes of determining the potentials and G, in which case the exterior charges reside on the surface S . Hence, it is plausible that these exist for interesting physical surfaces S (although it turns out that mathematicians have constructed examples of surfaces for which a Greens function does not exist). Since the function G is the potential for a speciable charge conguration, the normal derivative G/n corresponds to the electric eld (whose only nonzero component is En ) at the surface S produced by those charges. If we consider surface S to be a grounded conductor when determining function G, then the charge density G at position x on that surface, caused by the hypothetical unit charge at x, would be G (x, x ) = En /4 = (1/4 )G/n. Green emphasized this phyisical interpretation in his original work, and wrote eq. (13) as (x ) = G (x, x )(x ) dS . (14)

Turning at last to Poissons integral (8), we see that the needed Greens function for a sphere corresponds to the potential at x due to unit charge at x in the presence of a grounded conducting sphere of radius a. Use the method of images to construct the Greens function and its normal derivative, and thereby verify Poissons result.

Princeton University 1998

Ph501 Set 2, Problem 7

7. A parallel-plate capacitor is connected to a battery which maintains the plates at constant potential dierence V0 . A slab of dielectric constant is inserted between the plates, completely lling the space between them. (a) Show that the battery does work Q0V0 ( 1) during the insertion process, if Q0 is the charge on the plates before the slab is inserted. (b) What is the change in the electrostatic energy of the capacitor? (c) How much work is done by the mechanical forces on the slab when it is inserted? Is this work done by, or on, the agent inserting the slab? Suppose the battery was disconnected before the dielectric was inserted. (d) Repeat (b). (e) Repeat (c).

Princeton University 1998

Ph501 Set 2, Problem 8

8. (a) Find the escape velocity of an electron initially 1 A above a grounded conducting plate. (b) Point electric dipoles p1 and p2 lie in the same plane at a xed distance apart. If p1 makes angle 1 to their line of centers, show that the equilibrium angle 2 of p2 is related to 1 by tan 1 = 2 tan 2 . (15)

Princeton University 1998

Ph501 Set 2, Problem 9

10

9. We may dene the capacity of a single conductor with respect to innity as C = Q/V , where V is the potential (with respect to potential = 0 at ) when charge Q is present on the conductor. Calculate the capacity of a conductor composed of two tangent spheres of radius a.

Princeton University 1998

Ph501 Set 2, Problem 10

11

. 10. A grounded conducting sphere of radius a is placed in a uniform external eld E = E0z (This eld changes after the sphere is added.) This problem may be solved by the method of images if we suppose the eld E0 is due to two charges Q at positions z = R, with Q and R appropriately large. (a) Show that the image of the source of E0 is then a dipole p = a3E0 located at the center of the sphere. (b) Give an expression for the potential (r, ) in spherical coordinates (r, , ) centered on the sphere. Sketch the electric eld lines. (c) Show that the induced charge distribution on the sphere is =
3 E 4 0

cos .

(d) Show that the force between the two hemispheres with equator perpendicular to 9 2 E0 is F = 16 a E0 .

Princeton University 1998

Ph501 Set 2, Problem 11

12

11. A hollow innite rectangular conducting tube of sides a and b has two faces grounded and two faces at potentials V1 and V2 as shown:
y

=0 = V1 b =0 a x = V2

Find the potential (x, y ) inside the tube. Remember to use a sum of products of all solutions to the separated equations which do not violate the boundary conditions.

Princeton University 1998

Ph501 Set 2, Problem 12

13

12. A hollow rectangular conducting box has walls at x = 0 and a, at y = 0 and b, and at z = 0 and c. All faces are grounded except that at z = c, for which = V :
z

= V =0 =0 c =0 =0 =0 a x y b

Find the potential (x, y, z ) inside the box. (Choose the signs of the separation constants carefully!)

Princeton University 1998

Ph501 Set 2, Solution 1 Solutions

14

1. The energy stored in a dielectric composed of spring like atoms can be written in two parts, 1 kx2 2 U = U1 + U2 = E dVol + n dVol, (16) 8 2 where E is the applied electric eld, and where n is the number of molecules per unit volume. The displacement x in the spring-like atom is related by kx = eEon charge of an electron. Then, U2 = n
2 e2Eon 1 atom dVol = 2k 2 atom,

where e is the

2 e2Eon 1 atom dVol = 2 m 2

2 nEon

atom

dVol,

(17)

where = k/m is the frequency of oscillation of the electron of mass m, and = e2/m 2 is the atomic polarizability introduced in eq. (69) of set 1. On p. 20 of the Notes, we argued that Eon atom = E + 4 P/3, in terms of the applied eld E and the induced polarization P. But, P = nEon atom, so P= n E, 1 4n/3 and Eon
atom

= E 1+

n 1 4n/3

E,

(18)

where the approximation holds for small n. In this case, U2 and 1 8 4nE 2 dVol, (19)

1 1 1 (1 + 4n)E 2 dVol E 2 dVol = E D dVol, (20) 8 8 8 using the Lorenz-Lorentz approximation for the dielectric constant in terms of the polarizability , and supposing that D = E. U

Princeton University 1998

Ph501 Set 2, Solution 2

15

2. (a) As argued on p. 13 of the Notes, rotational symmetry of a charge distribution about the x axis implies that its quadrupole tenson Qij can be written

Qxx 0 0

0 Qxx/2 0

0 0 Qxx/2

Qij =

(21)

and hence, from (4), Qxx Ex 3 Ex 1 E = , 2 x 2 4 x (22) using E = 0, assuming that the external eld is produced by charges not at the location of the quadrupole. The force on the quadrupole is: U = Ex 1 Ey 1 Ez x 2 y 2 z = F = U = (b) Qxx 4 2Ex 2Ex 2 Ex , , . x2 xy xz (23) Qxx 6 Qxx 6

i. Consider a point charge q at at the origin and the quadrupole at (x, y, z ) = (R, 0, 0). The x-component of the electric eld from q observed at (x, y, z ) is Ex = Then, qx , r3 where r 2 = x2 + y 2 + z 2 . (24)

r2 3x2 Ex =q , x r5 and the force is evaluated from (23) at (R, 0, 0) as F= 3 qQxx , 0, 0 . 2 R4


y t

(25)

(26)

Let us check this for the simple quadrupole shown in the picture.
t

q 1

2q1

q 1

Suppose the distance between q1 and 2q1 is a. The force on the quadrupole due to charge q at distance R from the center of the quadrupole, and along the latters axis, is Fx = q1 q 2q1q q1 q 6a2q1 q + = (1 + O(a/R)). (R a ) 2 R2 (R + a )2 R4 (27)

This agrees with (26), since Qij = (3ri rj r 2 ij ) dVol Qxx = 2q r 2 = 4q1a2 . (28)

Princeton University 1998

Ph501 Set 2, Solution 2

16

ii. If, instead, the quadrupole is at (0, R, 0) (but still oriented parallel to the x axis), eqs. (23) and (25) combine to reveal that only the derivative 2Ex /xy is nonvanishing, and 3qQxx F = 0, ,0 . (29) 4R4 Again, we can directly compute the force on the simple quadrupole: q1qR q1 q 2 Fy = 2 2 R (R + a 2 )3 / 2 using (28). 3qq1a2 3qQxx = , 4 R 4R4 (30)

Princeton University 1998

Ph501 Set 2, Solution 3

17

3. Once the charge has reached distance d from the plane, the static electric eld Ee at an arbitrary point r due to the charge can be calculated by summing the eld of the charge plus its image charge, e r1 e r2 Ee (r, d) = 3 3 , (31) r1 r2 where r1 (r2) points from the charge (image) to the observation point r, as illustrated + Ee . below. The total electric eld is then E0z

The charge e and its image charge e at positions (r, , z ) = (0, 0, d) with respect to a conducting plane at z = 0. Vectors r1 and r2 are directed from the charges to the observation point (r, 0, z ). It turns out to be convenient to use a cylindrical coordinate system, where the observation point is r = (r, , z ) = (r, 0, z ), and the charge is at (0, 0, d). Then,
2 2 2 r1 ,2 = r + (z d) .

(32)

The part of the electrostatic eld energy that varies with the position of the charge is the interaction term, E0 z Ee dVol Uint = 4 eE0 zd z+d = dz dr2 2 2 2 3 / 2 4 0 [r + (z d) ] [r + (z + d)2 ]3/2 0 = eE0 4
0

dz

if z > d

2 if z < d

2 (33)

= eE0

d 0

dz = eE0d.

When the particle has traversed a potential dierence V = E0 d, it has gained energy eV and the electromagnetic eld has lost the same energy. In a practical electrostatic accelerator, the particle is freed from an electrode at potential V and emerges with energy eV in a region of zero potential. However, the particle could not be moved to the negative electrode from a region of zero potential by purely electrostatic forces unless the particle lost energy eV in the process, leading to zero overall energy change. An electrostatic accelerator must have an essential component (such as a battery) that provides a nonelectrostatic force that can absorb the energy extracted from the electrostatic eld while moving the charge from potential zero, so as to put the charge at rest at potential V prior to acceleration.

Princeton University 1998

Ph501 Set 2, Solution 4

18

4. (a) First, we calculate the force directly. The electric eld from one of the dipoles, , is taken to be at the origin and with moment p = px E= 3px r px 3(p r) rp = . r3 r4 r3 (34)

, we have For a second dipole at (x, y, z ) = (2d, 0, 0), also with moment p = px F = (p )E = p E x =
(2d,0,0)

3p2 . x 8d4

(35)

The minus sign indicates that the dipoles attract. As an aside, we can also calculate F = U , where U is the energy of interaction of the two dipoles. First, the energy of a charge q2 at position r2 in the eld of a dipole p1 at position r1 is p1 r U = q2 3 , (36) r where r = |r| = |r2 r1|, as on p. 12 of the Notes. A point dipole p2 is the limit 2 , and the product of a pair of charges q2 at positions r2 and r2 s where s = sp q2s is held constant at value p2 . Thus, the interaction energy of two point dipoles is obtained from (36) as U= lim q2 p1 r p1 r r3 r3 = (p2 2 ) p1 r , r3 (37)

s0, q2 s=p

separated by distance 2d along x, (37) where r = |r2 s r1|. For p1 = p2 = px reduces to 1 U = p2 . (38) x x=2d x2 Then, the force F = U is along x with magnitude F = p2 2 1 x2 x2 =
x=2d

3p2 , 8d4

(39)

as found in (35). Now, let us calculate the force via the Maxwell stress tensor. The force on the charges within a (closed) surface S is given by Fi = Tij dSj , (40)

as on p. 33 of the Notes, where the Maxwell tensor in empty space is given by Tij = 1 1 Ei Ej ij E 2 . 4 2 (41)

In the problem with two dipoles, it is convenient to choose the surface as the midplane perpendicular to the line connecting two dipoles (the x axis), closing

Princeton University 1998

Ph501 Set 2, Solution 4

19

the surface at innity around one of the dipoles. On this plane (x = d) the only nonzero component of E is Ex , and this is twice Ex from the dipole at x = 0. At radius r from (d, 0, 0) in the symmetry plane, the total eld is then Ex (r) = 2p 1 3d2 2 2 2 5 / 2 [r + d ] [r + d2 ]3/2

2 Ex

= 2p

2d2 r2 . [r2 + d2 ]5/2

(42)

The Maxwell stress tensor is thus, 1 Tij = 8

0
2 Ex

0 0
2 Ex

0 0

(43)

We take our surface element to be dS = (2rdr, 0, 0) in cylindrical coordinates, the sign of which implies that the surface S encloses the dipole at x = 0. Then, (40) and (43) indicate that only Fx is nonzero, and it is given by
1 (r2 2d2 )2 2 2 Fx = r drEx = p r dr 2 4 0 (r + d2 )5 0 p2 (t 2d2 )2 p2 [(t + d2 ) 3d2 ]2 = dt = dt (44) 2 0 (t + d2 )5 2 0 (t + d2 )5 6d2 9d4 p2 1 dt + = 2 0 (t + d2 )3 (t + d2 )4 (t + d2 )5 p2 1 2 9 3p2 = + = . 2 2d4 d4 4d4 8d4 This agrees with (35), noting that since the dipoles attract, the force on the dipole at x = 0 is in the +x direction. (b) The electric eld outside the conducting sphere of radius a is E = q r/r2 . The pressure (= force per unit area) on the surface charges is P = E/2, where is the surface charge density; hence, P = q 2 r/8a4 . (The coecient 1/2 is needed because E is the eld outside the surface, while the eld inside the sphere is zero, thus the average eld inside the charge layer is E/2.) To nd the force between two hemispheres, we integrate the component of pressure normal to the equatorial plane (P cos ) over one hemisphere:

q 2 cos q2 = . (45) 8a4 8a2 0 Now, let us calculate the force using the Maxwell stress tensor. We integrate Fz over the x-y plane separating our sphere into two hemispheres. Since dS = (0, 0, 2r dr) there, and the only nonzero components of E on that surface are Ex and Ey , only Tzz = E 2/8 = q 2/8r4 contributes to the force. Integrating from r = a to , we nd
1

F =

2a2 d cos

Fz = in agreement with (45).

2r drTzz =

q2 4

q2 dr = , r3 8a2

(46)

Princeton University 1998

Ph501 Set 2, Solution 5

20

5. (a) The electrical force F required to pull oil of density into a cylindrical capacitor of inner and outer radii a and b, respectively, to height h above the bath is equal to the force of gravity: F = gh(b2 a2). (47) A second relation for F can be computed from the balance of electrical energy, noting that the capacitor is held at constant voltage by a battery. Suppose we increase the height of the oil by h. Then, work F h is done on the oil, the energy U = CV 2 /2 stored in the capacitor changes by U , and the battery loses energy V Q. Conservation of energy implies 0 = F h + U V Q. Since V = Q/C , we nd for constant voltage, V Q = V 2 C = 2 Together, (48) and (49) imply that F =+ U h .
V

(48)

CV 2 2

= 2U.

(49)

(50)

As the liquid is drawn into the capacitor, the energy for this must come from elsewhere; yet, the energy of the capacitor increases because the battery loses energy in twice the amount of work done on the liquid. We now calculate the stored energy U by integrating the electric eld energy density. By cylindrical symmetry and Gausss law, the electric eld between the pipes has form Er (r) = /r, where is xed by V =
a b

b Er dr = ln , a

or

V b. ln a

(51)

Suppose the total height of the capacitor (above the bath) is H . Then, the energy of the electric eld in the capacitor is: U= 1 h 8
b a

E 2 2r dr +

1 (H h) 8

b a

E 22r dr,

(52)

where the rst term on the right is the contribution from the space lled with the oil whose dielectric constant is , while the second term is from the empty space above. Evaluating the integrals: 2 we then nd: U=
b a

E 2 r dr = 2

V2 b ln2 a

b a

V2 dr = 2 b , r ln a

(53)

1 V2 b [( 1)h + H ] . 4 ln a

(54)

Princeton University 1998

Ph501 Set 2, Solution 5

21

The force is obtained from (50) and (54): Fel = U h =


V

V 2 ( 1) . b 4 ln a

(55)

Equating this to the force of gravity, (47) we obtain the height h of the oil column: h= ( 1)V 2 4g ln
b a

(b 2 a 2 )

(56)

(b) The force on the liquid arises from the eect of gradients of the electric eld on the molecular dipoles in the liquid. The spatially varying electric eld E results in a bulk dielectric polarization given by P = E = 1 E, 4 (57)

where is the dielectric susceptibility and is the dielectric constant. The energy density associated with the induced polarization is u = P E = 1 2 E , 4 (58)

and so the force density on the liquid is given by f = u = 1 E 2 . 4 (59)

The gradient E 2 in the fringe eld of the capacitor points from the outside to the interior of the capacitor, with a generally vertical component for the liquid below the capacitor in the present problem. It is interesting to consider a variant on this problem: a capacitor with horizontal plates completely immersed in a dielectric liquid. Here, the fringe elds of the capacitor pull the liquid in from all sides, trapping it inside the capacitor. That is, work would be required to pull the liquid out of the capacitor in any direction. Is this an example of electrostatic trapping which is claimed not to exist? No! The trapping in the direction perpendicular to the capacitor plates is not provided by purely electrostratic elds, but by the material of the capacitor plates (whose stability is not a result of purely electrostatic eects). See prob. 7 of set 4 for further discussion. We have concluded that the liquid is drawn into the interior of the capacitor and that the liquid near the middle of the capacitor is forced up against the capacitor plates by electrostatic forces on the induced dipoles. If we drill a hole in the center of one capacitor plate, would liquid squirt out? (If yes, we would have a perpetual motion machine.) No, the fringe elds around the hole will pull liquid into the interior of the capacitor creating a static equilibrium much as before.

Princeton University 1998

Ph501 Set 2, Solution 6

22

6. We work from eq. (13), for which we rst need the potential at a point r inside a grounded conducting sphere of radius a when unit charge is located at x, also inside the sphere. Then we need the normal derivative of this potential on the inner surface of the sphere, i.e.when |r| = r = a. The image method for a grounded conducting sphere tells us that the potential inside the sphere can be calculated as that due to unit charge at x together with charge a/x at position x = a2x/x2 . We denote the angle between vectors r and x as , so that (60) R = |r x| = r2 + 2rx cos + x2 , and R = |r x | = We see that when r = a, then R = r2 + 2r a R. x a2 a4 cos + 2 . x x (61)

(62)

The potential inside the sphere can now be written (r ) = a 1 , R Rx (63)

The normal derivative of the potential on the inner surface of the sphere is the negative of its radial derivative when r = a, (r = a) a + x cos a[a + (a2/x) cos ] a 2 x2 = = , = n r R3 aR3 R 3x using eq. (62). Inserting this in eq. (13), we obtain Poissons integral, (x ) = a 2 x2 4a (x ) dS . R3 (65) (64)

Princeton University 1998

Ph501 Set 2, Solution 7

23

7. (a) The capacitance C of a parallel-plate capacitor of area A, gap thickness d and dielectric constant is A Q C= . (66) d V Adding the dielectric increased the capacitance to Cf = C0 , (67)

and hence the charge also increase, if the voltage is kept xed. Thus, the work done by the battery as the dielectric is inserted, Wbatt = V0 Q = V02C = V02 ( 1)C0 = Q0 V0 ( 1), is positive. (b) As the dielectric is inserted, the eld energy U = CV 2 /2 stored in the capacitor changes by 1 1 1 U = CV02 = C0 V02 ( 1) = Q0V0 ( 1). (69) 2 2 2 (c) The work done by the battery, (68), is only partly accounted for in increase in the eld energy, (69). The rest of the work done by the battery is done on the external agent that held the dielectric during insertion (the external agent gained energy): 1 (70) Won agent = Q0V0 ( 1). 2 (d) If the battery had been disconnected before the dielectric was inserted, then the charge Q0 would be constant. From (66) we see that the nal voltage would be only V0 / . Recalling (67), the change in the electrostatic eld energy would then be 1 1 V0 2 1 1 1 < 0. U = C0 C0 V02 = Q0V0 (71) 2 2 2 (e) By conservation of energy, the work done on the external agent that held the dielectric during insertion is equal and opposite to the change in stored energy. Hence the work done on the agent is again positive, but now with the value Won
agent

(68)

1 1 . = Q0V0 2

(72)

That is, the dielectric is pulled into the capacitor whether or not the battery is still connected.

Princeton University 1998

Ph501 Set 2, Solution 8

24

8. (a) The eld energy associated with an electron at distance r from a grounded conducting plane is 1/2 that associated with the corresponding image charge, i.e., with that electron plus a positron at distance r, in the absence of the conducting plane. Hence, 1 e2 e2 U = = . (73) 2 2r 4r The elds in the image solution have reality only outside the conducting plane; there is no energy associated with the ctitious image elds inside the conductor. Equation (73) indicates that an electron is bound to the conducting plane, and so to escape, must have a minimum velocity related by vmin = 2|U | = m e2 = 2mr e 2 c2 re =c , 2 2mc r 2r (74)

where re = e2/mc2 = 2.8 1013 cm is the classical electron radius. Thus, for r=1 A, vmin 2.8 1013 = 0.0037. (75) = c 2 108 (Notice that the nonrelativistic approximation suces.) The binding energy can be estimated from (73) as U = e2 re mc2 2.8 1013 5.11 105 eV 2 = mc = = 3.6 eV. 4mc2 r r 4 108 4 (76)

(b) In equilibrium, the torque on dipole p2 must vanish, and so p2 will be directed along the electric eld created by dipole p1 . The electric eld of the latter is given by 3(p1 r) r p1 . (77) E= 3 r The projection of E onto the line connecting two dipoles is p1 r = 2 3 cos 1 . (78) E = E r The orthogonal projection is E = E E , p1 sin 1 , r3 where the minus sign indicates that E is directed opposite to p1, . The angle of the eld line, and hence of p2 is E = tan 2 = E 1 = tan 1 . E 2 leading to (79) (80)

(81)

Princeton University 1998

Ph501 Set 2, Solution 9

25

9. We solve the problem of the capacity of two tangent, conducting spheres of radii a by the method of images. We rst nd the image-charge distribution needed to bring one sphere to potential V , but leaving the other at zero potential. Then, we complete the solution by superposing the mirror distribution, obtained by reection symmetry about the plane through the point of tangency of the two spheres. (a) Place charge q = aV at the center of sphere 1, bringing its surface to otential V . (b) To bring sphere 2 to zero potential, place charge q (a/2a) = q/2 at distance a2/2a = a/2 from the center of sphere 2, following the prescription on p. 41 of the Notes. (c) The image charge (b) takes sphere 1 away from potential V . To bring it back, add an image charge (c) inside sphere 1 so that this sphere is at zero potential under the eect of charges (b) and (c). That is, add charge (q/2)(a/(3a/2)) = +q/3 at distance a2/(3a/2) = 2a/3 from the center of sphere 1, i.e., a/3 from the point of contact. (d) Add charge q/4 at 3a/4 from the center of sphere 2 to bring it back to zero potential. (e) ....

-q/2

...

... -q/2

The total charge needed to bring both spheres to potential V is double that described in the sequence above. Hence, Q = 2q 1 and the capacitance is C = Q/V = 2a ln 2 = 1.386a. (83) Note that since the dimensions of potential are [charge]/[length], capacitance has the dimension of [length] in Gaussian units. Thus, we expect that C a for this problem, since a is the only relevant length. 1 1 1 + + ... = 2aV ln 2, 2 3 4 (82)

Princeton University 1998

Ph501 Set 2, Solution 10

26

10. (a) The uniform eld E0 = E0 z is approximated as being due to charges Q at z = R, where Q and R in such a way as to keep Q/R2 constant. In the limit, the eld in the region of the sphere is homogeneous and equal to z. According to the image method, we can make the potential E = (2Q/R2 ) on the sphere vanish by adding charge q = Qa/R at z = a2/R and q at z = a2/R. Thus, the perturbation to the eld due to the sphere is eectively that due to a dipole with the moment p=2 a2 a Q z = a 3 E0 . R R (84)

(b) The potential outside the sphere is thus, = 0 + dipole = E0r cos + E0a3 cos . r2 (85)

The eld lines bend in to be normal to the sphere at r = a:

(c) We nd the surface charge density from the normal component of the electric eld at the surface of the sphere: Er (a, ) = and so ( ) = r = 3E0 cos ,
r =a

(86)

3E0 cos Er (a, ) = . 4 4

(87)

2 (d) The force acting on the surface charge density is F = Er (a) r/2 = Er (a) r/8 (where the latter form follows immediately from the Maxwell stress tensor). The force on the right hemisphere is directed along z and is obtained by integrating the z component of F:

Fz =

1 8

1 0

2a2 d cos (3E0 cos )2(cos ) =

9 2 2 a E0 . 16

(88)

Since the force on the hemisphere at z > 0 is positive, the hemispheres repel each other.

Princeton University 1998

Ph501 Set 2, Solution 11

27

11. We seek solutions to Laplaces equation in 2 dimensions, 2 (x, y ) = 0, of the form = X (x)Y (y ). This leads to solutions of the form ekx eiky or eikx eky . Since the boundary conditions include = 0 at y = 0 and b, it is advantageous to consider functions Y of the type eiky , which can be immediately restricted to the form: Y (y ) = sin ky, where k = n/b, n = 1, 2, . . . . (89) This also xes the separation constants k . The general expression for the potential is now: (x, y ) =
n

Xn (x)Yn (y ) =
n

An enx/b + Bn enx/b sin

ny . b

(90)

The boundary conditions at x = 0 and x = a are (0, y ) = V1 =


n

(An + Bn ) sin

ny , b ny . b

(91) (92)

(a, y ) = V2 =
n

An ena/b + Bn ena/b sin

A straigthforward approach to nd An and Bn is to multiply (91) and (92) by sin(ny/b) and integrate from y = 0 to b:
b 0

(0, y ) sin

bV1 ny ny dy = cos b n b

=
0

b bV1 2, n odd = (An + Bn ) , (93) n 0, n even 2

and similarly,

b bV2 2, n odd = An ena/b + Bn ena/b . n 0, n even 2 Thus, for n even, An = Bn = 0, while for n odd, An = 2 V2 V1 ena/b , n sinh na b Bn = 2 V1 ena/b V2 . n sinh na b

(94)

(95)

Finally, we get for the potential: 4 (x, y ) = sin ny nx n (a x) b + V1 sinh . na V2 sinh n sinh b b b odd (96)

To verify that this solution satises the boundary conditions, note that (91) and (93) combine to yield the expansion: 1= 4 ny 1 sin . n b odd (97)

Princeton University 1998

Ph501 Set 2, Solution 11

28

We also note that the potential is symmetric about the midplanes, (x, y ) = (a x, y ) = (x, b y ), which could have been invoked as far back as (90) to show that only odd n contributes. Remark: This problem could also usefully be solved as the superposition of two cases, each with three walls at potential zero and the fourth at a nonzero value. The form of the solution (96) displays this superposition.

Princeton University 1998

Ph501 Set 2, Solution 12

29

12. Since = 0 at x = 0, a and y = 0, b, solutions = X (x)Y (y )Z (z ) must have the form Xm (x) = sin mx , a and Yn (y ) = sin ny , b (98)

where n and m are positive integers (and odd, recalling the remark at the end of problem 9). The functions Z (z ) then have the form ekz . Since = 0 at z = 0, we can make the further restriction: Zmn (z ) = sinh kmn z, (99)

where kmn is determined by inserting the trial solutions into Laplaces equation, yielding m 2 n 2 2 = + . (100) kmn a b The general solution satisfying all the boundary conditions except for the one at the face z = c is: (x, y, z ) =
m,n

Amn sin

ny mx sin sinh a b

m a

n b

z.

(101)

The remaining boundary condition tells us that V =


m,n

Amn sin

ny mx sin sinh a b

m a

n b

c.

(102)

To nd Amn , multiply (102) by sin mx sin ny and integrate from 0 to a in x and from a b 0 to b in y . Similarly to (93), we nd Amn = mn2 sinh 16V
m a 2

n b

, c

(103)

for odd m and n, and 0 otherwise. Hence, 16V (x, y, z ) = 2 mx 1 ny sinh 1 sin sin a n b odd m sinh
m a m a 2

+ +

n b n b

z . c (104)

m,n

Note that we have demonstrated the expansion 1=


m

mx 4 sin m a

ny 4 sin n b

for

0 < x < a,

0 < y < b.

(105)

Since this follows from (97), we could have used it to go from (102) to (103) without performing the integrations.

Princeton University

Ph501 Electrodynamics Problem Set 3


Kirk T. McDonald (1999) kirkmcd@princeton.edu http://puhep1.princeton.edu/~mcdonald/examples/

Princeton University 1999

Ph501 Set 3, Problem 1

1. A grid of innitely long wires is located in the (x, y ) plane at y = 0, x = na, n = 0, 1, 2, . . .. Each line carries charge per unit length.

Obtain a series expansion for the potential (x, y ). Show that for large y the eld is just that due to a plane of charge density /a. By noting that z complex or real, sum the series to show (x, y ) =
n=1 zn n

= ln(1 z ) for

2y 2x + ln 1 2e2y/a cos + e4y/a a a 2x 2y cos = ln 2 cosh . a a

(1)

Show that the equipotentials are circles for small x and y , as if each wire were alone.

Princeton University 1999

Ph501 Set 3, Problem 2

2. Two halves of a long, hollow conducting cylinder of inner radius b are separated by small lengthwise gaps, and kept at dierent potentials V1 and V2 .

Give a series expansion for the potential (r, ) inside, and sum the series to show (r, ) = Note that
zn n

V1 + V2 V1 V2 2br sin + tan1 2 b2 r 2

(2)

n odd

1 2

z ln 1+ , and Im ln z = phase(z ). 1z

Princeton University 1999

Ph501 Set 3, Problem 3

3. The two dimensional region a < r < b, 0 is bounded by conducting surfaces held at ground potential, except for the surface at r = b.

Give an expression for (r, ) satisfying these boundary conditions. Give the lowest order terms for Er and E on the surfaces r = a, and = 0. As an application of the case = 2 , consider a double gap capacitor designed for use at very high voltage (as in streamer chamber particle detectors): The central electrode is extended a distance b beyond the ground planes, and is terminated by a cylinder of radius a b. Calculate the maximal electric eld on the guard cylinder compared to the eld E inside the capacitor, keeping only the rst-order term derived above.

You may approximate the boundary condition at r = b as


(r = b )

Eh(1 /0 ), || < 0 , 0, 0 < ||,

(3)

1 and h is the gap height. Note that the surfaces r = a and = 0 where 0 h/b are not grounded, but are at potential Eh. Answer: Emax
2 Eh . ab

Princeton University 1999

Ph501 Set 3, Problem 4

4. Find the potential distribution inside a spherical region of radius a bounded by two conducting hemispheres at potential V/2 respectively. Do the integrals to evaluate the two lowest-order nonvanishing terms.

Princeton University 1999

Ph501 Set 3, Problem 5

5. Find the potential both inside and outside a spherical volume of charge of radius a in which the charge density varies linearly with the distance from some equatorial plane (Qtot = 0).

Princeton University 1999

Ph501 Set 3, Problem 6

6. A uniform eld E0 is set up in an innite dielectric medium of dielectric constant . Show that if a spherical cavity is created, then the eld inside the cavity is: E= 3 E0 . 2 +1 (4)

This problem diers from our discussion of the actual eld on a spherical molecule in that the eld inside the remaining dielectric can change when the cavity is created. The result could be rewritten as Ecavity = E0 + 4P , 2 +1 (5)

where P is the dielectric polarization. A Clausius-Mosotti relation based on this analysis is, however, less accurate experimentally than the one discussed in the lectures.

Princeton University 1999

Ph501 Set 3, Problem 7

7. A spherical capacitor consists of two conducting spherical shells of radii a and b, a < b, but with their centers displaced by a small amount c a. Take the center of the sphere a as the origin. Show that the equation of the surface of sphere b in spherical coordinates with z along the line of centers is r = b + cP1 (cos ) + O(c2 ) . (6)

Suppose sphere a is grounded and sphere b is at potential V . Show that the electric potential is (r, ) = V ra ba b r abc 2 r (b a ) r 3 a3 P1 (cos ) + O(c2) . b3 a 3 (7)

What is the capacitance, to order c?

Princeton University 1999

Ph501 Set 3, Problem 8

8. a) A charge Q is distributed uniformly along a line from z = a to z = a at x = y = 0. Show that the electric potential for r > a is (r, ) = Q r a r
2n

P2n (cos ) . 2n + 1

(8)

b) A at circular disk of radius a has charge Q distributed uniformly over its area. Show that the potential for r > a is (r, ) = 1 a Q 1 r 4 r
2

P2 (cos ) +

1 a 8 r

P4

5 a 64 r

P6 + . . . .

(9)

For both examples, also calculate the potential for r < a.

Princeton University 1999

Ph501 Set 3, Problem 9

9. A conducting disk of radius a carrying charge Q has surface charge density (r) = (both sides combined). a) Show that the potential in cylindrical coordinates is Q (r, z ) = a

Q 2a a2 r2

(10)

ek|z| J0 (kr)
0

sin ka dk . k

(11)

See section 5.302 of the notes on Bessel functions for a handy integral. b) Show that the potential in spherical coordinates is (r > a): (r, ) = Q r (1)n a 2n + 1 r
2n

P2n (cos ) .

(12)

Note the relation for P2n (0) (see sec. 5.157) to obtain a miraculous cancelation.

Princeton University 1999

Ph501 Set 3, Problem 10

10

10. A semi-innite cylinder of radius a about the z axis (z > 0) has grounded conducting walls. The disk at z = 0 is held at potential V . The top of the cylinder is open. Show that the electric potential inside the cylinder is (r, z ) = 2V a ekl z J0 (kl r) . kl J1(kl a) (13)

Refer to the notes on Bessel functions for the needed relations.

Princeton University 1999

Ph501 Set 3, Problem 11

11

11. a) Calculate the electric potential everywhere outside a grounded conducting cylinder of radius a if a thin wire located at distance b > a from the center of the cylinder carries charge q per unit length.

Use separation of variables. Interpret your answer as a prescription for the image method in two dimensions. b) Use the result of a) to calculate the capacitance per unit length between two conducting cylinders of radius a, whose centers are distance b apart.

Answer (in Gaussian units, the capacitance per unit length is dimensionless): C=
b+ b2 4a2 2a

4 ln

(14)

c) [A bonus.] You have also solved the problem: What is the resistance between two circular contacts of radius a separated by distance b on a sheet of conductivity ? Apply voltage V between the contacts. Field E appears, and current density J = E arises as well. The total current is I= J dS = E dS = 4Qin , (15)

by Gauss law, and Qin is the charge on one of the contacts needed to create the eld E. But if t is the thickness of the sheet, then the disk is like a length t of the cylindrical conductor considered in part a). Therefore, Qin/t = V C with capacitance C as in part b), assuming that J and E are two-dimensional. Hence, I = 4tV C = V/R by Ohms law, and (15) leads to R 1 = . (16) R= 4tC 4C where 1 L R = = (17) t tL is the resistance of a square of any size on the sheet.

Princeton University 1999

Ph501 Set 3, Solution 1 Solutions

12

1. This problem is 2-dimensional, and well described in rectangular coordinates (x, y ). We try separation of variables: (x, y ) = X (x )Y (y ) . (18)

Away from the surface y = 0, Laplaces equation, 2 = 0, holds, so one of X and Y can be oscillatory and the other exponential. The X functions must be periodic with period a, and symmetric about x = 0. This suggests that we choose Xn (x) = cos kn x, with kn = 2n . a (19)

We rst consider the regions y > 0 and y < 0 separately, and then match the solutions at the boundary The Y functions are exponential, and should vanish far from the plane y = 0. Hence we consider

Yn (y ) =

ekn y , y > 0, ekn y , y < 0.

(20)

However, we must remember that the case of index n = 0 is special in that the separated equations are X0 = 0 = Y0 , so that we can have X0 = 1 or x, and Y0 = 1 or y . In the present case, X0 = 1 is the natural extension of (19) for nonzero n, so we conclude that Y0 = y is the right choice; otherwise X0 Y0 = 1, which is trivial. Then the potential = y will be associated with a constant electric eld in the y direction, which is to be expected far from the grid of wires. Combining X and Y , our series solution thus far is (x, y ) =

a0 y + a 0 y +

n>0

an cos(2nx/a) e2ny/a ,

y > 0,

n>0

an cos(2nx/a) e2ny/a , y > 0,

(21)

where we have used continuity of the potential at y = 0 to use the same an for both y > 0 and y < 0. Note, however, the sign change for a0, corresponding to the constant electric eld that points away from the wire plane. At the boundary, y = 0, the surface charge density is 1 1 (x, 0+ ) (x, 0 ) + (Ey (0 ) Ey (0 )) = + = (x na) = 4 4 y y n 1 a0 + = nan cos(2nx/a). (22) 2 a n We evaluate the an by considering the interval [a/2 < x < a/2]. Multiplying by cos(2nx/a) and integrating, we nd a0 = 2 , a and an = 2 . n (23)

Princeton University 1999 The potential is then, (x, y ) =

Ph501 Set 3, Solution 1

13

2|y | 1 + 2 cos(2nx/a) e2n |y|/a. a n>0 n

(24)

To sum the series, we write it as (x, y ) = = = where 1 2nx/a 2n |y|/a 2|y | + 2Re e e a n>0 n 1 2ix/a 2|y|/a n 2|y | + 2Re (e e ) a n>0 n 2|y | 2Re ln(1 z ), a z = e2ix/a e2|y|/a. (25)

(26)

To take the real part, we note that if ln(1 z ) u + iv, and Re ln(1 z ) = u = ln |1 z | = ln |1 e2|y|/a [cos(2x/a) + i sin(2x/a)] = ln 1 2 cos(2x/a)e2|y|/a + e4|y|/a 1 = ln[1 2 cos(2x/a)e2|y|/a + e4|y|/a ]. 2 The potential is now (x, y ) = = = = For x, y small: cosh 2x 1 2y 2 |y | cos 1+ a a 2 a
2

then

1 z = eu eiv ,

|1 z | = e u ,

(27)

(28)

ln e2|y|/a ln[1 2 cos(2x/a)e2|y|/a + e4|y|/a ] ln[e2|y|/a 2 cos(2x/a) + e2 |y|/a] ln[2 cosh(2 |y |/a) 2 cos(2x/a)] ln[cosh(2 |y |/a) cos(2x/a)] ln 2.

(29)

+ ... 1+

1 2x 2 a

+ ... =

1 2r 2 a

(30)

where r2 = x2 + y 2. Thus, at small x, y , (x, y ) 2 ln 2r , a (31)

which is just the potential for an individual line charge . Close to each wire, the equipotentials are cylinders around this wire.

Princeton University 1999

Ph501 Set 3, Solution 2

14

2. For a 2-dimensional potential problem with cylindrical boundaries, it is appropriate to use polar coordinates (r, ). In general, the potential could be expanded in a series of terms in rn cos n and rn sin n. For a bounded potential in the region r < b, only factors of rn can occur. In the present problem, we measure from the plane that separates the two half cylinders, and take 0 < < on the half cylinder at potential V1 . Since varies over the full range [0, 2], n must be an integer. The average potential is (V1 + V2 )/2, and the variable part of the potential has the symmetries (r, ) = (r, ), and (r, ) (r, ). The rst implies that only factors of sin n can occur, and the second tells us that n must be odd. Thus, the potential has the form: (r, ) = V1 + V2 + 2 n an rn sin n.
odd

(32)

To x the coecients an , we use the boundary conditions at r = b, which can be written as V1 V2 an bn sin n = sign(), (33) 2 n odd where sign()

+1, 0 < < , 1, pi < < 0.

(34)

Thus, we have to learn how to decompose the function sign() in Fourier series. For a straightforward evaluation of the Fourier coecients an , multiply (34) by sin n and integrate from 0 to 2 : an bn = (V1 V2 ) Thus, (r, ) = = = 2 V1 + V2 + (V1 V2 ) 2 n rn sin n n nb odd
0

sin n d =

2 (V1 V2 ). n

(35)

V1 + V2 2 + (V1 V2 )Im 2 n

(rei /b)n n odd

V1 + V2 V1 V2 1 + rei /b + Im ln 2 1 rei /b V1 + V2 V1 V2 1 (r/b)2 + 2i(r/b) sin = + Im ln 2 1 + (r/b)2 2(r/b) cos V1 + V2 V1 V2 2br sin . = + tan1 2 b2 r 2

(36)

Princeton University 1999

Ph501 Set 3, Solution 2

15

In the above, we used the facts about logarithms stated in the problem; the second of which follows from (27). As a sidelight, we can compare (33) with (36) to learn that sign() = 4 n sin n . n odd (37)

This is, of course, also the famous Fourier expansion of a square wave, since that is the result of periodically extending the denition (33).

Princeton University 1999

Ph501 Set 3, Solution 3

16

3. The general possibilities for a series expansion for this problem are similar to those of problem 2. Since (r, 0) = 0 = (r, ), the angular factors can only be sin(n/). Then, since the radial extent includes neither the origin nor , factors of both rn/ and rn/ can occur. Thus, the potential can be written

(r, ) =
n=1

an

r a

n/

+ bn

a r

n/

sin

n .

(38)

The use of factors r/a and a/r is convenient for enforcing the boundary condition (a, ) = 0, since this simply requires bn = an . For the electric eld, we get: Er = E n n 1 r n/ 1 a n/ + sin = an , r r a r r n n 1 r n/ 1 a n/ 1 n = an = cos r r a r r n (39) (40)

At r = a, E = 0. At = 0, Er = 0, as expected. At r = a, Er = At = 0, E = r
n

2 a

nan sin
n/

n . a r
n/

(41)

n an

r a

(42)

To obtain this, we ignored the small terms proportional to (a/b)n/2 compared to the terms proportional to (b/a)n/2 in (r = b). We cannot determine the coecients an until the boundary condition at r = b is specied. For the example of a Streamer Chamber, = 2 . The surfaces r = a and = 0, 2 are at potential Eh rather than 0, but we can accommodate this by simply adding Eh b, we to (38). To evaluate an , we use the boundary condition (3) at r = b. Since a can approximate the potential there as (r = b) Eh + an b a
n/2

sin

Eh(1 /0 ), || < 0 , n = 2 0, 0 < | |.

(43)

Subtracting Eh from both sides, we nd an b a


n/2

sin

Eh/ 0 , || < 0 , n = 2 Eh, 0 < | |.

(44)

On multiplying by sin(n/2) and integrating from 0 to , we nd n an = 2Eh a b


n/2

cos

2 n n 0 sin . 2 n0 2

(45)

Princeton University 1999

Ph501 Set 3, Solution 3

17

From (41), the eld on the surface of the guard cylinder is Er (r = a) = 2Eh a a b
n/2

cos

2 n n0 n . sin sin 2 n 0 2 2

(46)

Since a/b is very small, it suces to keep only the rst term, which is maximal at = : 2Eh . Er,max(r = a) (47) ab For reasonable values of a, b and h, we have Er,max < E. A sign that our approximations are somewhat delicate is obtained by evaluating E (r = b) using (42). If we keep only the rst term, we nd that E (r = b) Eh/b, instead of E . However, because of the form (45) of the an , the leading term at each order n in series (42) does not have any factors of a/b, and this series converges much more slowly than does (41). The terms are of similar magnitude until n0 , i.e., until n b/h, and E (r = b) sums to E .

Princeton University 1999

Ph501 Set 3, Solution 4

18

4. This problem involves a spherical boundary, so we seek a solution in spherical coordinates (r, , ). Since the problem has axial symmetry, the potential will be independent of , and of the general form (r, ) = An r a
n

+ Bn

a r

n+1

Pn (cos ).

(48)

The region of interest contains the origin, so we must have Bn = 0 for a nite potential there. To nd the An , we use the boundary condition at r = a: multiply (48) by Pn and integrate over cos to nd 2n + 1 An = 2
1

(a, )Pn (cos ) d cos =


0

2n + 1 V 4

1 0

(Pn (z ) Pn (z )) dz .

(49)

Since Pn (z ) = (1)n Pn (z ), we get An = 0 for even n. For odd n, we get An = 2n + 1 V 2


1 0

Pn (z ) dz.

(50)

Using the explicit expressions for the polynomials Pn , we nd for the rst 2 nonvanishing terms: A1 = A3 The potential is: 7 3 r (r, ) = V P1 (cos ) V 4 a 16 r a
3

3 V 2 7 V = 2

3 zdz = V, 4 0 1 1 7 (5z 2 3z )dz = V. 16 0 2

(51) (52)

P3 (cos ) + . . .

(53)

Princeton University 1999

Ph501 Set 3, Solution 5

19

5. This problem has an axially symmetric charge distribution (r), so we can evaluate the potential via the multipole expansion. This is the sum of two series, one for contributions from charge at radius r < r and the other for charge at r > r: (r, ) =
n

Pn (cos ) rn+1
n

r 0

2r 2 dr
r

1 1

d cos (r )r n Pn (cos )
1 1

rn Pn (cos )

2r 2 dr

d cos (r )

Pn (cos ) . (r )n+1

(54)

In the present problem, the charge distribution is nonzero only for r < a, where it has the form = 0 z = 0 r cos = 0 rP1 (cos ). We rst evaluate the potential outside the sphere of radius a, for which we need only the rst series of (54). The integral is
a 0

2r 2 dr

1 1

d cos 0 r P1 (cos )r n Pn (cos ) =

5 2a 2
5

, 3 0

n = 1, n = 1.

0,

(55)

using the orthogonality relation


1 1

d cos Pn (cos )Pm (cos ) =

2 nm . 2n + 1

(56)

Thus, 4a50 cos , 15 r2 which the potential due to a dipole of strength p = 4a40 /15. (r > a, ) = The total charge in the upper hemisphere is Q0 =
a 0 1 0

(57)

2r2 dr

d cos 0 rP1 (cos ) = 2

a40 a4 0 = , 4 2 4

(58)

with Q0 in the lower hemisphere. The eective height z0 of this charge is such that the dipole moment is p = 2Q0 z0, so z0 = 8a/15. For the potential inside the sphere, we must evaluate both series in (54), but we see in each case that only the n = 1 term survives the angular integration. Therefore, (r < a, ) = 2 2r 2 dr 0 r 2 + rP1 (cos ) 3 0 2a2 r 2r3 = 0 cos . 3 5 P1 (cos ) r2
r a r

2 r 2r 2 dr 0 2 3 r (59)

Expressions (57) and (59) give the same value at r = 1, as expected. For possible instruction, we give a second solution for the potential inside the sphere, where Poissons equation applies: 2 = 4 = 40rP1 (cos ). (60)

Princeton University 1999

Ph501 Set 3, Solution 5

20

Since this is a linear partial dierential equation, a solution can be found in terms of a particular solution p to (60) plus the general solution to the homogeneous equation, which is Laplaces equation 2h = 0 in the present case. Also, we must match our solution for r < a to that found for r > a. This problem has axial symmetry, so the general solution to the homogeneous equation for r < a can be written h(r, ) = An r2 Pn (cos ). (61)
n

Since the solution for r > a involves only P1 , we expect that the solution for r < a will also. Then, h = Ar cos . Returning to Poissons equation, writing = cos , we get 1 1 r2 + 2 (1 2 ) = 40 r. 2 r r r r (62)

We hope for a simple power law solution in r, and expect the angular function to be just P1 = . That is, we try = Brn . Inserting this into (62), we learn that the particular solution is p = 20 r3 /5. The complete solution then has the form (r < a, ) = h + p = Ar cos Matching this to (57) at r = a requires: Aa 40 a3 20 a3 = , 5 15 (64) 20 r3 cos . 5 (63)

which gives A = 2 0 a2, and hence the solution (59). 3

Princeton University 1999

Ph501 Set 3, Solution 6

21

6. Let the potential be given by the function 1 (r, ) inside the sphere (r < a) and 2(r, ) outside. The boundary conditions for the potential are 1(a, ) = 2(a, ), 1(a, ) = (a, ), r r 2 where is the dielectric constant of the medium at r > a. (65) (66)

, the potential 2 approaches Since the asymptotic electric eld at r is E0 = E0z 2(r ) E0z = E0rP1 (cos ), (67)

even after we have created the cavity at the origin. Hence, it is clear that the decomposition of in spherical harmonics should contain only terms proportional to P1 (cos ). Recalling the general form (48), we expect r 1 (r, ) = A P1 (cos ), a 2 (r, ) = E0 r + B a r
2

(68) P1 (cos ). (69)

From the boundary conditions at r = a, we get A = E0 a + B = (E0 a 2B ). Solving for A and B , we get 3 3 E0 r cos = E0 z, 2 +1 2 +1 3 . E0z E (r < a ) = 2 +1 1 = The asymptotic value of polarization is related to E0 via P= 1 E0 . 4 (73) (71) (72) (70)

Thus, we may rewrite E(r < a) in terms of E0 and asymptotic value of P as E (r < a ) = E 0 + 4 P . 2 +1 (74)

Princeton University 1999

Ph501 Set 3, Solution 7

22

7. The equation of the surface of the sphere of radius b and center at z = c is c b2 = r2 2rc cos + c2 = r2 1 2 cos + c2 , r (75)

where r is measured from the origin in a spherical coordinate system. For small c, we approximate this as r = b + c cos + O(c2 ) = bP0(cos ) + cP1 (cos ) + O(c2 ). (76)

Between the spherical shells of radii a and b, the potential has the general axially symmetric form (48). From the boundary condition (r = a) = 0, we conclude that (r, ) = An r a
n

a r

n+1

Pn (cos ).

(77)

The boundary condition on the outer shell can expressed via (76) in terms of P0 (cos ) and P1 (cos ). Hence, it is plausible that only A0 and A1 are nonzero in (77), which then reads r a2 a + A1 P1 . (78) = A0 1 r a r2 [For a discussion that does not make this leap, see eqs. ] The boundary condition on the outer shell now implies that (r = b + cP1 ) = V = A0 1 A0 a b + cP1 a2 + A1 P1 b + cP1 a (b + cP1 )2 ac b a2 a 1 + A0 2 + A1 P1 b b a b2 (79)

2 where we have dropped terms in P1 in the last line of (79) as these lead to a correction 2 to A0 of order c . Hence, the constant term on the last line of (79) equals V , while the coecient of P1 must be zero. This determines the values of A0 and A1, and the potential is

(r, ) = V

abc rab 2 b a r r (b a )

r 3 a3 P1 (cos ) + O(c2) . b3 a 3

(80)

To nd the capacitance C = Q/V , we have to nd the charge Q on the spherical shells. It is simpler to calculate this for the inner shell which is at r = q . Q= dS = Er (r = a) a2 dS = 4 2
1 1

d cos

(a, ) . r

(81)

The terms in the integrand proportional to P1 (cos ) will integrate to zero, so the capacitance is unchanged by a small oset c.

Princeton University 1999

Ph501 Set 3, Solution 7

23

For the record, Q = abV/(b a) = CV so the capacitance is C = ab/(b a) (which is a length, as always in Gaussian units). As a footnote, we show how the boundary condition on the outer shell could be implemented without immediately assuming that only coecients A0 and A1 are important in (77). Neglecting terms of O(c2), we nd (r = b + c cos ) = V =
n n

b An a

c b 1 + n cos b a b a
n n1

n1

b An a c + b

c 1 (n + 1) cos P (cos ) n b

Pn (cos )
n1

P1 (cos )Pn (cos ).

b An n a

b + (n + 1) a

(82)

At this point, we invoke the recurrence relation P1 (cos )Pn (cos ) = n+1 n Pn+1 (cos ) + Pn1 (cos ). 2n + 1 2n + 1 (83)

The constants An are determined from the requirement that the coecients of Pn in (82) should be zero for n > 0, and the coecient of P0 = 1 is V . This leads to recurrence relations for An of the following form (schematically): ( )An + c( )An+1 + c( )An1 = 0, n > 0. (84)

By iteration, we nd a solution with the property An O(cn ). It is straightforward to nd the rst two coecients A0 and A1, which again gives (80).

Princeton University 1999

Ph501 Set 3, Solution 8

24

8. The problem concerns two examples of specied, axially symmetry charge distributions. Hence, the multipole expansion (54) can be used to calculate the electric potential. a) The charge distribution is = Q/2a along the line from z = a to z = a. Thus, there is charge only for cos = 1 and 1, and the charge distribution is symmetric in cos . Since Pn (1) = (1)n Pn (1), the integrals in (54) will be nonzero only for even n. For an observer at r > a, the multipole expansion simplies to (r > a, ) = Similarly, (r < a, ) =
r P2n (cos ) Q 2 dz (z )2n P2n (1) 2 n +1 2a n r 0 a dz Q r2n P2n (cos ) 2 P2n (1) + 2a n r (z )2n+1 Q 1 r 2n 1 = P2n (cos ) 1 + a n 2n + 1 2n a

Q 2a

P2n (cos ) 2 r2n+1

a 0

dz (z )2n P2n (1) =

Q r

a r

2n

P2n (cos ) . (85) 2n + 1

(86)

As expected (85) and (86) agree at r = a. b) As in example a), the charge distribution is symmetric in cos , so only even n will contribute to the multipole expansion of the potential. The charge distribution on the disc r < a, cos = 0 is = Q/a2 . Hence, the potential for r > a is (r > a, ) = Q P2n (cos ) a 2r dr (r )2n P2n (0) a2 n r2n+1 0 (1)n (2n 1)P2n (cos ) a 2n Q = , r n 2n (n + 1) r

(87)

which gives (9), noting that P2n (0) = (1)2 (2n 1)/2n . Similarly, (r < a, ) = Q P2n (cos ) r 2r dr (r )2n P2n (0) 2 a n r2n+1 0 a P2n (0) Q + 2 r2n P2n (cos ) 2r dr . a n (r )2n+1 r Qr (1)n P2n (cos ) 4n + 1 r 2n1 . 2 = a2 n 2n n+1 a

(88)

Again, these two expressions agree at r = a. The charged surfaces in these examples are not conductors, so those surfaces are not equipotentials. If you had forgotten the multipole expansion, you could have proceeded by rst solving the simpler problem of the potential on the axis. For example, in the case of the charged

Princeton University 1999 disk, Q (z > a ) = a2

Ph501 Set 3, Solution 8

25

2Q 2 2r dr = z + a2 z a2 r2 + z 2
2

Q 1 a = 1 z 4 z

1 a 8 z

5 a 64 z

+ ... .

(89)

The potential (r, ) is obtained from this simply by replacing z by r and multiplying the term in 1/rn by Pn (cos ), etc.

Princeton University 1999

Ph501 Set 3, Solution 9

26

9. a) We expect any solution of Laplaces equation having axial symmetry in cylindrical coordinates (r, , z ) to be a sum of expressions of the form ekz J0 (kr). (90)

In the example of a conducting disk, there are no boundary surfaces, so the separation constant k will be continuous. The problem is symmetric about the plane z = 0, so we look for a solution of the form (r, z ) =
0

dk f (k )ek|z| J0 (kr).

(91)

To nd the Fourier coecients f (k ), we note that the electric eld experiences a jump across the conducting disk at z = 0: (r < a, z = 0+ ) (r < a, z = 0 ) = 4. z z Hence, 2
0

(92)

kf (k )J0 (kr) dk =

2Q (a r) , a a2 r 2

(93)

using expression (10) for the charge density . Given the integral relation
0

sin kaJ0(kr) dk =

0,

(a r ) = . 2 r2 a 1 , r < a a2 r 2 r>a

(94)

we nd that f (k ) = Q sin(ka)/ka, and the potential is given as in (11). b) To give a solution in spherical coordinates for the potential due to a specied, axially symmetric charge distribution, we again use the multiple expansion (54). The present problem is quite similar to problem 8b, so we write (r > a, ) = P2n (cos ) a (r )2 n Q 2 r dr P2n (0) 2a n r2n+1 0 a2 r 2 Q P2n (0)(n + 1) a 2n+1 = P2n (cos ), a n (2n + 1)(n + 3/2) r (1)n a 2n+1 Q P2n (cos ), = a n 2n + 1 r

(95)

Princeton University 1999

Ph501 Set 3, Solution 10

27

10. This problem has boundary conditions on the potential that (r = a) = 0, and (z = 0) = V , so a solution in cylindrical coordinates (r, , z ) for r < a, z > 0 will have the form = Al ekl z J0 (kl r). (96)
l

where J0(kl a) = 0. At z = 0, we have V =


l

AlJ0 (kl r).

(97)

The {J0(kl r)} are an orthogonal set of functions on the interval [0, a] upon integration with respect to dr2 rather than dr. Hence, we can evaluate the Fourier coecients Al by multiplying (97) by J0 (km r) and integrating: Al
l a 0

r dr J0 (kl r)J0 (km r) = Am = V

a2 [J1(km a)]2 2
a 0

a r dr J0(km r) = V J1 (km a), k

(98)

using 5.297(3) and 5.294(7) of Smythe. With this, we obtain the expansion (r, , z ) = 2V a ekl z J0 (kl r) . kl J1 (kl a) (99)

Princeton University 1999

Ph501 Set 3, Solution 11

28

11. This two dimensional problem involves cylinders about the z axis, so we use cylindrical coordinates to discuss the potential (r, ). The rst conducting cylinder of radius a has its axis along the z . a) A wire at (r, ) = (b, 0) carries charge density q per unit length. We present three related solutions; the rst two use Fourier series, where the rst decomposes the potential into = wire + cylinder, while the second does not; the third solution is more elementary. In all cases, we have the symmetry () = (), so the Fourier expansion for the potential contains terms in cos n , but not sin n. The potential due to the wire has the general form

n

wire (r, ) =

a0 +

n=1

an

r b

cos n,
b r n

r < b, cos n, r > b,

a0 + b0 ln r + b

(100)

n=1 an

since this should not blow up at the origin, should be continuous at r = b, and can have a logarithmic divergence at innity. The potential due to the conducting cylinder has the form cylinder(r > a, ) = A0 + B0 ln r + An cos n. n n=1 r (101)

The cylinder is grounded, so the total potential (and not cylinder) obeys (r = a) = 0. Hence, n a2 A n = a n , (102) a0 + A0 + B0 ln a = 0, b and so the potential due to the cylinder is a2/b r cylinder(r > a, ) = a0 + B0 ln an a n=1 r where coecient B0 is not yet determined. Comparing (103) to the form (100) of the potential due to the wire for r > b, we see that these are the same for terms with n > 0, except for an overall sign, and the substitution b a2/b. Since we are still free to choose the value of B0 , we set it to b0 ln b/a, and the potential of the cylinder becomes cylinder(r > a, ) = a0 b0 ln r a2 /b a n a2/b n=1 r
n n

cos n,

(103)

cos n,

(104)

and now the n = 0 terms are also related to those of eq. (100) in the same way as the terms with n > 0. This suggests that the potential due to a grounded, conducting cylinder of radius a in the presence of a line charge density q at r = b is the same that due to a line charge density q at r = a2/b. This is the desired image method for cylindrical geometry.

Princeton University 1999

Ph501 Set 3, Solution 11

29

There was no need to evaluate the Fourier coecients an to reach this conclusion! In the second solution, we do not separate the potential into two parts, and we carry out the evaluation of the Fourier coecients. The cylinder r = a is at zero potential, so the most general form that satises these conditions for a < r < b is (r, ) = b0 ln r b0 ln a +
n=1

An

r a

a r

cos n

(a < r < b ).

(105)

Beyond the wire at r = b, we can only have the form

(r, ) = c0 + d0 ln r +

n=1

Bn

b r

cos n

(r > b ).

(106)

As this problem is meant to represent a real 2-wire system, the energy per unit length must be nite. Therefore, we must have d0 = 0, so that no eld lines from the wire escape to innity. This also means that the charge on the cylinder at r = a must be q . The potential is continuous at r = b, which leads to the conditions b c0 = b0 ln , a Bn = An b a
n n

a b

(107)

The remaining condition is obtained by considering a Gaussian surface (of unit length in z ) that surrounds the cylindrical surface (b, ): 4qin = For this we learn that 4q() = b(Er+ Er ) = b = b0 + (b+ ) (b ) + r r n b a n cos n Bn + An + a b E dS = b d (Er+ Er ). (108)

(109)

Multiply this by cos n and integrate over to nd b0 = 2q, Bn + An b a


n

a b

4q . n

(110)

Combining this with (107), we learn that b c0 = 2q ln , a An = 2q n a b


n

Bn =

2q a 1 n b

2n

(111)

Princeton University 1999

Ph501 Set 3, Solution 11

30

For a < r < b we now have r 1 a n r n a n (r, ) = 2q ln + 2q cos n a b a r n n n 1 r n a2 r (ei )n = 2q ln + 2qRe a n b br n r rei a2ei = 2q ln 2qRe ln 1 ln 1 a b br i 2 i r re ae = 2q ln 2q ln 1 + 2q ln 1 a b br b a2ei = 2q ln 2q ln b rei + 2q ln r a b b = 2q ln 2q ln r2 2br cos + b2 + 2q ln a r2 a2 a2 2r cos + b b

(112)
2

Recall that the potential at distance R from a line of charge q per unit length is = 2q ln R + const. Thus the second term of the last line of (112) is the potential due to the line of charge density q at (r, ) = (b, 0). The second term is equal to the potential due to a line of charge density q at (r, ) = (a2 /b, 0). Hence, we have demonstrated the cylindrical image method: the image in a conducting cylinder of radius a of a line of charge density q at radius b is a line of charge density q at radius a2/b. In terms of distances r1 and r2 shown in the gure below, the potential is then, br2 . (113) (r, ) = 2q ln ar1 [We skip the demonstration that this prescription also works for r > b.] For a third solution, we suppose that there exists an image wire carrying charge density q at distance c from the center of the conducting cylinder, as shown in the gure.

Then the potential at an aribtrary point (r, ) due to the two wires is (r, ) = q + q = K 2q ln r1 2q ln r2 = K q ln(r2 + b2 2br cos ) q ln(r2 + c2 2cr cos ). (114)

Princeton University 1999 The cylinder is grounded, so

Ph501 Set 3, Solution 11

31

(r = a, ) = 0 = K q ln(a2 + b2 2ab cos ) q ln(a2 + c2 2ac cos ). This can be arranged by putting q = q , so that (115) simplies to K = q ln a2 + b2 2ab cos . a2 + c2 2ac cos

(115)

(116)

If we take c = a2/b (inspired by our knowledge of the spherical image method), we nd that the argument of the logarithm becomes b2/a2 , which is independent of , as desired. Hence, K = 2q ln(b/a), and the potential is again (113). b) Suppose we have two parallel conducting cylinders of radius a each, carrying charge +q and q per unit length, whose axes are distance b apart. We want to nd locations for two line charge densities +q and q such that the elds from these lines charges are the same as those due to the two conducting cylinders. Clearly, these lines charges should be placed symmetrically in the plane containing the axes of the cylinders, say distance c apart. Then the rst line charge is distance (b + c)/2 from the center of the second cylinder. Its image charge would then be located distance 2a2 /(b + c) from the center of the second cylinder. We want the image charge to be the same as the second line charge, which is at distance (b c)/2 from the center of the second cylinder. Equating the two distances, we nd that (117) c = b2 4a2 . To nd the capacitance C = q/V , we need the potential dierence V between the two cylinders. We can calculate the potential on a conducting cylinder at any convenient point. For example, consider the point on one cylinder closest to the other. This point is distance a (b c)/2 from one line charge, and distance (b + c)/2 a from the second line charge. The potential at this point is therefore V = 2q ln[a (b c)/2] + 2q ln[(b + c)/2 a] = 2q ln = 2q ln b+c . 2a c + b 2a c (b 2a) (118)

The corresponding point on the second cylinder is at potential V , so V = 2V , and the capacitance is 1 C= . (119) b+ b2 4a2 4 ln 2a

Princeton University

Ph501 Midterm Examination Electrodynamics


Kirk T. McDonald (Oct. 23, 2000) mcdonald@puphep.princeton.edu http://puhep1.princeton.edu/mcdonald/examples/

Princeton University

Ph501 Midterm Exam

Oct. 25, 2000

Please do all work in the exam booklets provided. You may use either Gaussian or MKSA units on this exam. 1. (10 pts.) Show that the charge induced in a small area A on a grounded conducting plane by a point charge not in that plane is proportional to the solid angle subtended at the point charge by area A. 2. (20 pts.) A hollow dielectric sphere of dielectric constant = 3 has inner radius one half its outer radius. When this sphere is placed in an initially uniform electric eld E0 , what is the resulting electric eld strength at the center of the sphere? 3. (30 pts.) Two circular wires of radii a and b have a common center, and are free to turn on an insulating axis which is a diameter of both. Find the torque about this diameter required to hold the two wire loops at rest when their planes are at right angles and they are carrying currents I and I , supposing that b a. Give both the leading term, and the rst correction in a power of the small ratio b/a. Hint: This requires evaluating the rst correction to both the axial and transverse magnetic eld components near the center of the larger loop. Recall that the torque about a point is = r F where force F is applied at distance r.

Princeton University

Ph501 Midterm Exam Solutions

Oct. 25, 2000

1. Let charge q be at perpendicular distance a from the grounded conducting plane. The small area A has its center at distance r from the foot of the perpendicular to charge q . The charge q induced in the area A is related by q = A = EA , 4 (1)

where E is the electric eld strength at the surface of the conducting plane. We calculate E using the image method, supposing that charge q is located at distance a on the other side of the conducting plane from charge q . Then, E= 2q a cos = 2q 2 , 2 R R R (2)

where R = a2 + r2 is the distance from charge q to area A, and is the angle between vector R and the perpendicular from q to the plane. Combining eqs. (1) and (2), we have q = q 2qA cos = , 2 4R 2 (3)

where = A cos /R2 is the solid angle subtended by area A at charge q . For the whole plane, = 2 and q = q . 2. This problem is closely related to that of a dielectric sphere in an otherwise uniform electric eld. We choose the z axis antiparallel to the initial eld E0 , with the origin at the center of the dielectric sphere, where the potential is taken to be zero. The potential of the initial eld is then 0 = E0 z = E0 r cos = E0 rP1 (), (4)

where is the polar angle with respect to the z axis and P1 is the Legendre polynomial of order 1. We recall from the case of a uniform dielectric sphere that the potential contains terms only in P1 , and we expect the same here. Writing the inner radius of the sphere as a and the outer radius as b, we expect that the potential will have the form r 1 = E0 rP1 + A P1 , a r b2 2 = E0 rP1 + B P1 + C 2 P1 , a r b2 3 = E0 rP1 + D 2 P1 , r (0 < r < a) (a < r < b) (a < r < b) (5) (6) (7)

since the perturbation to eld E0 must be nite at r = 0 and .

Princeton University

Ph501 Midterm Exam

Oct. 25, 2000

The potential is continuous at r = a and b, so that A = B+C b2 , a2 (8)

b B + C = D. (9) a Also, the normal component of the electric displacement D = E is continuous at the boundaries, since D = 0. Hence, 1 (a) 2 (a) = , r r and which yields 2 (b) 3 (b) = , r r B Cb2 A 2 3 , E0 + = E0 + a a a E0 + Inserting eq. (8) in (12), we get b2 1 B (2 + 1) 3 C = (1 )E0 , a a while using eq. (9) in (13) gives +2 2( 1) B C = (1 )E0 . (15) a b These could be solved in general for A, B and C , but here we consider the particular case that a = 1, b = 2 and = 3, for which eqs. (14) and (15) become B 14C = E0 , and 5B 2C = 2E0 . We quickly nd that B= and from eq. (11), 13 E0 , 34 C= 3 E0 , 68 (17) (18) (16) (14) B C D 2 = E0 2 . a b b (10)

(11)

(12) (13)

and

7 E0 . 34 The electric eld strength at the center of the dielectric sphere is A = B + 4C = E (0) = E0 + A =

(19)

27 E0 . (20) 34 A dielectric sphere is not as eective as a conducting sphere in shielding its interior from an external electric eld.

Princeton University

Ph501 Midterm Exam

Oct. 25, 2000

3. (Problem 12, p. 448 of The Mathematical Theory of Electricity and Magnetism by J. Jeans.) The leading term of the torque is given by B(0), where = I b2 c (21)

is the magnetic moment of the small loop of radius b that carries current I , and B (0) = 1 c 2aI 2I I dl = = 2 2 r ca ca (22)

is the magnetic eld at the center of the loops due to the current I in the loop of radius a. When the two loops are at right angles, the vectors and B(0) are also at right angles, so the magnitude of the leading term of the torque is = 2II b2 c2 a (23)

To evaluate the torque in greater detail, we consider the variation of the magnetic eld over the small loop, and use the basic torque equation = r dF = 1 c r [I dl B(due to I)]. (24)

We use a coordinate system in which the centers of the loops are at the origin, with the axis of loop a is along the z axis. We take the sign of current I to be such that the resulting magnetic eld at the origin is in the +z direction. The axis of loop b is dened to be the y axis, and the sign of current I is such that the magnetic moment is along the +y axis. Then, we desire the x component of the torque about the origin: x = 1 ( Bz z + B b r [I bd )] c x 2 2 bI d cos (cos Bz + sin B ), = c 0

(25)

where angle is measured in the x-z plane with respect to the z axis, such that for a point on loop b, = b sin and z = b cos . If we dont recall the results of problem 7, set 4, the magnitude of B can be estimated quickly using the Maxwell equation B = 0 and a pillbox surface of radius and thickness dz whose axis is along the z axis: 0 = BdVol = B dS

2 (Bz (0, z + dz ) Bz (0, z )) + 2dzB (, z ). Bz (0, z ) + 2dzB (, z ). 2 dz z

(26)

Princeton University Hence,

Ph501 Midterm Exam

Oct. 25, 2000

B (, z )

Bz (0, z ) . 2 z

(27)

Then, near the center of loop a its magnetic eld obeys B = 0, and in particular Br (, z ) 2 Bz (0, z ) Bz (, z ) = , z 2 z 2 using eq. (27). We can integrate this to nd Bz (, z ) Bz (0, z ) 2 2 Bz (0, z ) , 4 z 2 (29) (28)

in agreement with the results of Problem 7, Set 4. For points along the z axis the magnetic eld due to loop a is Bz (0, z ) = 1 c I dl r2 =
z

2a2 I 2I 2 2 3 / 2 c(a + z ) ca

3z 2 , 2a2

(30)

where the approximation can be used when we evaluate the eld on loop b for which |z | b a. Thus, Bz (0, z ) 6a2 zI 6zI = 2 , (31) z c(a + z 2 )5/2 ca3 and 6a2 I (a2 4z 2 ) 6I 2 Bz (0, z ) = , (32) z 2 c(a2 + z 2 )7/2 ca3 Using eqs. (27) and (31), the transverse magnetic eld at a point on loop b is B (, z ) 3Iz 3b2 I cos sin = , ca3 ca3 3b2 I sin2 3b2 cos2 + . 2a2 2ca3 (33)

and eqs. (29), (30) and (32) give the axial eld as Bz (, z ) 2I ca 1 3z 2 2a2 + 3I2 2I = 3 2ca ca 1 (34)

Combining eqs. (25), (33) and (34) we nd x b2 II 2 d c2 a 0 b2 II 2 d = c2 a 0 2 2 b2 II = 1 c2 a 3b2 cos4 3b2 cos2 sin2 3b2 cos2 sin2 + + 2 cos a2 2a2 a2 3b2 cos2 15b2 sin2 2 + 2 cos2 a2 8a2 9b2 . (35) 16a2
2

[The answer in MKSA units is obtained on setting c = 1 in the magnetic force equation, and replacing 1/c by 0 /4 in the Biot-Savart law, so 2 2 /c2 0 /2.]

Princeton University

Ph501 Electrodynamics Problem Set 4


Kirk T. McDonald (1999) kirkmcd@princeton.edu http://puhep1.princeton.edu/~mcdonald/examples/

Princeton University 1999

Ph501 Set 4, Problem 1

1. a) Childs Law. Before the transistor era, a common device was a vacuum diode. This is a parallel plate capacitor with a potential dierence V across a gap d, all of which is inside a vacuum tube. The cathode (at = 0) is heated, so electrons can jump o and ow to the anode (at = V ). Positive charges have very low probability of leaving the anode and owing to the cathode. The resulting one way ow of charge from cathode to anode is the diode action. Consider a steady situation in which a constant current density j = (x)v (x) ows, and where the electrons leave the cathode with velocity v (0) = 0. Here, (x), 0 x d, is the electron charge density. Solve for the potential (x) via Poissons equation, 2 = 4. Show that (1)

1 V 3/2 2e x 4/3 , and J= . (2) d 9 d2 m where e and m are the magnitudes of the charge and mass of the electron, respectively. (x ) = V

Note that since the current density J = nev is constant, and v 0 near the cathode, the charge density n there. The eld due to this large space charge distribution near the cathode opposes the eld due to the capacitor alone, and cancels it completely very close to the cathode. That is, E (x) xp with p > 0. Then, (x) = E dx x1+p rises more quickly than the simple linear relation for an ordinary capacitor. b) Laser Driven Vacuum Photodiode. A vacuum photodiode is constructed in the form of a parallel-plate capacitor of area A, plate separation d. A battery maintains constant potential V between the plates. A short laser pulse illuminates that cathode at time t = 0 with energy sucient to liberate all of the surface-electron charge density. This charge moves across the capacitor gap as a sheet until it is collected at the anode at time T . Then another laser pulse strikes the cathode, and the cycle repeats. Estimate the average current density j that ows onto the anode from the battery, ignoring the recharging of the cathode as the charge sheet moves away. Then calculate the current density and its time average when this eect is included. You may suppose that the laser photon energy is equal to the work function of the cathode, so the electrons leave the cathode with zero velocity.

Princeton University 1999

Ph501 Set 4, Problem 2

2. Obtain a Legendre series expansion for the potential inside a conducting sphere of radius a and conductivity when a current I enters at one pole through a ne wire, also of conductivity , and leaves through the other pole via a similar ne wire. Dene the potential as = 0 on the equator. By noting that Pn () = (1)n Pn (), and referring to the expansion of 1/R given on p. 57 of the notes, show that (r, ) = I 1 1 1 + 2 R1 R2 2
r 0

1 1 R1 R2

d ln r ,

(3)

where R1,2 is the distance from the north (south) pole to the point (r, , ) in spherical coordinates. The integrals can be found in tables if desired.

, D 5

U 5

,
Finally, suppose the wires have radius b a, and their surface of contact with the sphere is an equipotential. Show that the resistance of the sphere is that of a piece of wire roughly b long. Hint: Express the radial current density at r = a in terms of delta functions, (cos 1) and (cos + 1).

Princeton University 1999

Ph501 Set 4, Problem 3

3. Charge Distributions in a Wire that carries a Steady Current a) A wire of circular cross-section carries a current I which is uniformly distributed across the wire. We consider this current to be due to a number density of free electrons moving with average drift velocity v . (In a typical situation, v 1cm sec1 !) Let 0 be the uniform number density of positive ions in the wire. For steady current ow, there must be no radial force on the electrons. Use the Lorentz force law, F=q E+ v B c (4)

to nd the relation between 0 and such that the force vanishes. (As a check, you may wish to do this problem via special relativity, but try it using Maxwells equations and the Lorentz force.) b) A resistor of resistance R, length l and cross-sectional area A carries a current I , delivered by ne lead wires. Calculate the charge that accumulates on the end faces of the resistor in order to produce the eld E which drives the current according to Ohms law. Suppose that the current in the wire varies with time. Show that the conduction current inside the resistor is dierent from that in the lead wires, but that Maxwells concept of displacement current restores the continuity of total current. A so that the current density J is uniform inside the wire. You may assume that l

Princeton University 1999

Ph501 Set 4, Problem 4

4. A straw tube chamber is a low cost version of a proportional counter. These devices consist of a pair of coaxial conducting cylinders with the region between the cylinders is lled with a gas such as argon. The inner cylinder of radius a is the anode, and is held at potential V ; the outer cylinder of radius b is the cathode, and is grounded. If a penetrating charged particle passes through the chamber, it will ionize about two gas molecules per mm of path length. The ionization electrons are pulled by the electric eld towards the anode. Close to the anode, the eld is strong enough that the electrons gain enough energy during one mean free path to ionize the molecule they hit next, liberating one or more additional electrons. In a proportional chamber, the eld is kept low enough that the resulting Townsend avalanche involves 104 -106 molecules. What is the time dependence, I (t), of the current that ows o the anode due to the avalanche of a single initial electron? What is the spatial dependence, q (z ) of the charge distribution induced on the anode during the time when the current is large, where the z axis is the chamber axis? You may restrict your attention to values of z far from the ends of the tube of length l. Measurement of the charge distribution via a segmented cathode permits localization in z of the ionization, and hence, of the initiating charged particle. [C. Leonidopoulos, C. Lu and A.J. Schwartz, Development of a Straw Tube Chamber with Pickup-Pad Readout, Nucl. Instr. and Meth. A427, 465 (1999).] You may ignore the tiny current that ows while the electron drifts towards the anode. The avalanche takes place so close to the anode, that the small remaining drift time for the electrons to reach the anode may also be ignored. In this approximation, the situation at t = 0 is that electrons of total charge q0 reside on the anode in close proximity to positive ions of total charge +q0 . Current ows o the anode only when some of the eld lines from the positive ions detach from the electrons on the anode, and extend to the cathode where charge is induced to terminate these eld lines. This occurs only as the positive ions move away from the anode, with velocity related by v = E, where positive ion mobility. I (t) via Reciprocity and Weighting Fields This problem can be solved by an application of Greens reciprocation theorem, which states that if a set of xed conductors is at potentials Vi when carrying charges Qi, and at potentials Vi when carrying charges Qi , then
i

(5)

Vi Qi =

Vi Qi .

(6)

To see this, we label the 3-dimensional potential distribution associated with charges Qi by (r), and that associated with charges Qi by . The space outside the conductors is charge free and with dielectric constant = 1. Then 2 = 0 = 2 outside the conductors.

Princeton University 1999

Ph501 Set 4, Problem 4

We invoke Greens theorem (p. 37 of the Notes) (2 2 )dvol = ( ) dS, (7)

where we take the bounding surface S to be that of the set of conductors. Hence, 0=
i

(Vi i Vi i ) dSi = 4

(Vi Qi Vi Qi ),

(8)

using Gauss Law (in Gaussian units) that 4Qi = Ei dSi = i dSi . (9)

In the present problem, we have a small charge q0 at position r0(t) that moves under the inuence of the eld due to conductors i = 1, ..., n that are held at potentials Vi . The charges Qi on the conductors obey Qi q0, so the motion of charge q0 is determined, to a very good approximation by the charges Qi on the conductors when q0 = 0. Hence, the problem can be considered as the superposition of two situations: A: charge q0 absent; conductors i = 1, ...n at potentials Vi . B: charge q0 present; conductors i = 1, ...n grounded, with charges Qi on them. We are particularly interested in the charge on electrode 1, whose time rate of change is the desired current I (t). To use the reciprocation theorem, we suppose that in case B the charge resides on a tiny conductor at position r0 that is at the potential V0 = A (r0) obtained from case A. Then, the charges and potentials in case B can be summarized as B: {q0, V0 ; Qi, Vi = 0, i = 1, ..., n}. We solve the electrostatics problem for a third case, C: {q0 = 0, V0 (r0 ); Q1, V1 = 1; Qi = 0, Vi = 0, i = 2, ..., n.}, in which conductor 1 is held at unit potential, the charges on all other conductors at zero, and all other conductors are grounded except for the tiny conductor at position r0. Again, we solve this problem as in case A, rst ignoring the tiny conductor, then evaluating V0 as C (r0). The reciprocation theorem (6) applied to cases B and C implies that 0 = q0 V0 + Q1 1. The current that moves o electrode 1 in case B is therefore, I1 = dQ1 dr0 dV (r0) = q0 0 = q0V0 (r0) = q 0 Ew v , dt dt dt (11) (10)

where the velocity v of the charge is determined using the elds from case A, and Ew = V0 (r0 ) = C (r0 ) (12)

Princeton University 1999

Ph501 Set 4, Problem 4

is called the weighting eld. For the case of two conductors (plus charge q0 ) one of which is grounded, the weighting eld is the same as the eld from case A, but in general they are distinct. As the present problem involves only two conductors, you may wish to nd a solution that does not appear to use the initially cumbersome machinery of the reciprocation theorem.

Princeton University 1999

Ph501 Set 4, Problem 5

5. Resistance of a Disk with Edge Contacts Calculate the resistance between two contacts on the rim of a disk of radius a, thickness t a, and conductivity , when each (perfectly conducting) contact extends for a small distance around the circumference, and the distance along the chord between the contacts is d .

Princeton University 1999

Ph501 Set 4, Problem 6

6. Some biological systems consist of two phases of nearly square ber bundles of differing thermal and electrical conductivities. Consider a circular region of radius a near a corner of such a system as shown below.

Phase 1, with electrical conductivity 1, occupies the bowtie region of angle , 1 , occupies the remaining region. while phase 2, with conductivity 2 Deduce the approximate form of lines of current density J when a background electric eld is applied along the symmetry axis of phase 1. What is the eective conductivity of the system, dened by the relation I = between the total current I and the potential dierence across the system? It suces to consider the case that the boundary arc (r = a, || < ) is held at electric potential = 1, while the arc (r = a, < || < ) is held at electric potential = 1, and no current ows across the remainder of the boundary. 1, the electric potential is well described by the leading term of a Hint: When 2 series expansion.

Princeton University 1999

Ph501 Set 4, Problem 7

7. A rectangular loop of size 2a by 2b carries a current I , and is free to rotate about an axis that bisects the sides of length 2b. The axis is parallel to and distance d from a wire that carries current I . If the plane of the loop makes angle to the plane containing the wire and the axis, and if the currents in the wire and in the side (of length 2a) of the loop closest to the wire ow in the same direction, show that the magnitude of the torque on the loop is N= What is its direction? (b2 + d2 ) sin 8abdII . c b4 + d4 2b2 d2 cos 2 (13)

, ,
E D[LV D G

Princeton University 1999 8. Helmholtz Coils

Ph501 Set 4, Problem 8

10

a) Each of a pair of parallel, coaxial Helmholtz coils has radius a and carries a current I in the same sense. Their centers are at z = b, where the z axis is the common axis of the coils. Calculate the magnetic eld along the axis, and determine the separation 2b such that the rst, second and third derivatives of Bz with respect to z all vanish at the mid axis. Thus, the eld is very uniform at the center of the Helmholtz coils. b) Suppose we desire an even more uniform eld at the origin. Add a second pair of Helmholtz coils of radius a = a/2. What current I should ow in the second pair so as to cancel the 4rth derivative of Bz of the rst pair? What fraction of the original central eld is lost in this conguration? c) In some applications, it is more important that the eld outside the coils be as small as possible, rather than the eld inside be highly uniform. Give an expansion for the eld along the axis of a set of Helmholtz coils as a function of u = 1/z for z a, b. Identify the rst two nonvanishing multipoles, and nd the value of b for which the second of these can be made to vanish. To cancel the leading multipole as well, add a second coil pair with a = 2a. What current I should ow in this pair? What fraction of the central eld of the rst pair is lost? What is the order of the rst remaining nonzero multipole? [See, E.M. Purcell, Am. J. Phys. 57, 18 (1989).1 ]

http://puhep1.princeton.edu/~mcdonald/examples/EM/purcell_ajp_57_18_88.pdf

Princeton University 1999

Ph501 Set 4, Problem 9

11

9. Expansion of an Axially Symmetric Magnetic Field in Terms of the Axial Field Suppose a magnetic eld in a current-free region is rotationally symmetric about the z -axis. Then, B = Br (r, z ) r + Bz (r, z ) z (14) in cylindrical coordinates. The axial eld Bz (0, z ) is often relatively easy to calculate. If we write

Bz (r, z ) =

n=0

a n (z )r n ,

and

Br (r, z ) =

n=0

b n (z )r n ,

(15)

then a0(z ) = Bz (0, z ). Use B = 0 and B = 0 to show that Bz (r, z ) = and Br (r, z ) = where
n

(1)n

a 0 (z ) r (n!)2 2
(2n+1)

(2n)

2n

,
2n+1

(16)

(1)n+1
(n)

a0 (z ) (n + 1)(n!)2

r 2

(17)

dn a0 . (18) dz n This magnetic eld can also be deduced from the vector potential whose only nonzero component is (2n) a 0 (z ) r 2n+1 n . (19) A(r, z ) = (1) (n + 1)(n!)2 2 n a0 = For the example of Helmholtz coils, prob. 5, we know that Bz (0, z ) = B0 + B4 z 4 + . . . Give Bz and Br correct to fourth order in r and z . Show also that, for small r, B = 0 leads to the relation Br (r, z ) r Bz (0, z ) . 2 z (21) (20)

Remark. An electrostatic eld with azimuthal symmetry about the z axis can also be expanded according to eqs. (16)-(17). For example, consider a capacitor with circular plates centered about (r, , z ) = (0, 0, 0). Then we can expand Ez (0, 0, z ) Ez (0, 0, 0) + and z 2 d2 Ez (0, 0, 0) + ... 2 dz 2 (22)

r d2 Ez (0, 0, 0) Ez (r, 0, 0) Ez (0, 0, 0) + ... (23) 2 dz 2 Thus, if Ez has a maximum with respect to z at the origin, it is at a minimum with respect to r, or vice versa. The eld E cannot be at a maximum with respect to both r and z , as shown in general in prob. 1(c) of set 1.

Princeton University 1999

Ph501 Set 4, Problem 10

12

10. Nonaxially Symmetric Magnetic Field in Terms of the Axial Field In the previous problem, it was demonstrated how knowledge of a static, axial magnetic eld leads to a complete characterization of the eld if that eld is axially symmetric. A variant on the electro- or magnetostatic boundary value problem arises in accelerator physics, where a specied eld, say B(0, 0, z ), that is not axially symmetric is desired along the z axis. In general there exist static elds B(x, y, z ) that reduce to the desired eld on the axis, but the boundary condition B(0, 0, z ) is not sucient to insure a unique solution. For example, nd a eld B(x, y, z ) that reduces to + B0 sin kz y B(0, 0, z ) = B0 cos kz x on the z axis. In this, the magnetic eld rotates around the z axis as z advances. Show that the use of rectangular or cylindrical coordinates leads naturally to dierent forms for B o the z axis. One 3-dimensional eld extension of (24) is the so-called helical wiggler, which obeys the auxiliary requirement that the eld at z + be the same as the eld at z , but rotated by angle k . Show that this eld pattern can be realized by a current-carrying wire that is wound in a helix of period = 2/k . See, B.M. Kincaid, A short-period helical wiggler as an improved source of synchrotron radiation, J. Appl. Phys. 48, 2684-2691 (1977);2 J.P. Blewett and R. Chasman, Orbits and elds in the helical wiggler, J. Appl. Phys. 48, 2692-2698 (1977).3 (24)

2 3

http://puhep1.princeton.edu/~mcdonald/examples/EM/kincaid_jap_48_2684_77.pdf http://puhep1.princeton.edu/~mcdonald/examples/EM/blewett_jap_48_2692_77.pdf

Princeton University 1999

Ph501 Set 4, Problem 11

13

11. Axial Field of a Solenoid Magnet A solenoidal coil of radius a and length l has n turns per unit length and carries a current I (in each turn). On the axis, show that Bz (0, z ) = 2nI (cos 1 + cos 2 ), c (25)

where 1 and 2 are the angles between the axis and the ends of the solenoid at the observation point. Near the midpoint of the solenoid (z = 0), show Br (r, z ) 288nIa2rz . cl4 (26)

At the end of the coil (z = l/2), show that Bz and Br Bz (0, 0) 2nI , c 2 nIr . ac (27)

(28)

If one is interested in the elds near the end of a long solenoid (l a), it is often sucient to approximate the coil as semi-innite, for which (25) leads to Bz (0, z ) = where z = 0 at the end of coil. 2nI c 1+ z + a2 , (29)

z2

Princeton University 1999

Ph501 Set 4, Problem 12

14

12. a) A coil is wound on the surface of a sphere such that the magnetic eld inside the sphere will be uniform. How should the turns be distributed? b) What is the eective magnetic dipole moment of a sphere of uniform surface charge density which rotates with constant angular velocity about an axis of the sphere? An electron has a permanent magnetic dipole moment of magnitude = eh . 2mc (30)

Suppose the electron is a rotating spherical shell of charge with radius a= e2 , mc2 (31)

the classical electron radius. What is the velocity at the equator?

Princeton University 1999 13. Saturation and Hysteresis

Ph501 Set 4, Problem 13

15

a) A piece of iron saturates in a magnetic eld of 20, 000 Gauss, when all available electron magnetic moments are aligned. The density of iron is 8 g/cm3 . How many electrons per iron atom have been aligned to produce this eld? b) You can understand some aspects of the hysteresis curve of a ferromagnet via a model consisting of two permanent dipoles separated by a xed distance d, but free to rotate. In the absence of any external eld, what is the equilibrium orientation and energy of the two dipoles? Suppose a magnetic eld B is applied at right angles to the line of centers of the dipoles. What is the minimum eld strength needed to align the dipoles along B? Higher elds produce no further change saturation has occurred. Suppose the dipoles were originally aligned parallel to their line of centers, and then a eld B is applied antiparallel to the dipoles. What is the minimum value of B needed to ip the dipoles? If the dipoles ip and later B is reduced to zero, the dipoles do not unip hysteresis has occurred.

Princeton University 1999

Ph501 Set 4, Problem 14

16

14. Magnetic Field Mapping. You may be familiar with the method of mapping the equipotentials in 2-dimensional electrostatic problems using conducting paper:

On the paper, J = E, where J is the two dimensional current density (= current per unit length perpendicular to J), is the surface conductivity of the resistive paper, and E is the electric eld in the paper. Outside of the sources and sink of current in the patches of conducting paint, we have J = 0, so E = 0 also. The currents and elds are steady, so E = 0, and the electric eld can be derived from a potential, E = that obeys Laplaces equation, 2 = 0. The value of the potential at any point on the paper can be read directly with a voltmeter. The boundary conditions are that J and E are perpendicular to the boundaries of the patches of conducting paint. J and E are parallel to the edges of the paper (where there is no conducting paint. Thus, the region outside the paper is like a dielectric with constant = 0. This is not very physical, so make the paper much larger than the region used to model the problem of interest (if the boundaries are not entirely conducting). The conducting paper technique can also be used to model 2-dimensional magnetostatic problems due to current distributions that are normal to the paper. Imagine that the regions of conducting paint represent the cross sections of innite conductors that are perpendicular to the paper. Let z label the unit vector normal to the paper. Then, the vector potential due to our imagined currents would be A= 1 c J dVol = Az z. r (32)

(Here, J is due to the imagined current normal to the paper, not the surface currents in the paper.) Show that the observed potential on the paper, when a battery feeds current into and out of the conductors on the paper, is proportional to the vector potential of the imagined situation: k = constant. (33) Az = k,

Princeton University 1999

Ph501 Set 4, Problem 14

17

Your analysis might include the following: Relate the magnetic eld B of the imagined 2-dimensional current distribution to on the paper. When A = Az z, B is perpendicular to z . Show that lines of B exactly follow equipotentials of . Relate B to E on the paper. Suppose current I from a battery enters a region of conducting paint on the paper, causing current density J to ow outwards:

Consider J dl for a loop enclosing the region of conducting paint to determine the constant k in (33). Show that the voltage dierence between any two points on the paper is proportional to the magnetic ux passing between these points. Show that the boundary conditions at the edge of the paper are such that we may consider the region outside the paper as being iron of a very large permeability . As an example, consider a long electromagnetic with an iron yoke:

Invoking symmetry, we could map this with an arrangement like:

Try it in the lab sometime!

Princeton University 1999

Ph501 Set 4, Solution 1 Solutions

18

1. a) The problem is 1-dimensional, so Poissons equation is d2 = 4(x). dx2 (34)

Since an electron leaves the cathode at v = 0, when it reaches position x, it has energy e(x) = mv 2/2, and velocity 2e . (35) v= m Since the current density J = v is constant, eq. (34) becomes J d2 = 4 = 4J 2 dx v m 1/2 . 2e (36)

We try (pray for) a power law solution, = axp, which quickly leads p = 4/3. Then, since (d) = V , the potential is (x ) = V Equation (36) can now be rearranged as J= 4 1 V 3/2 2e 1/2 = m 9 d2 1 V 3/2 2e = m 6.36 d2 e . m (38) x d
4/3

(37)

The electric space charge density (x) follows from eqs. (34) and (37), (x) = V = , 3 4 9d dx2 (39)

which is very large close to the cathode at x = 0. b) The initial electric eld in the capacitor is E = V/d, so the initial surface charge density on the cathode is = E/4 = V/4d. (40) The laser liberates this charge density at t = 0. The average current density that ows onto the anode from the battery is J = V = , T 4dT (41)

where T is the transit time of the charge across the gap d. We rst estimate T by ignoring the eect of the recharging of the cathode as the charge sheet moves away from it. In this approximation, the average eld on the charge sheet is always E/2 = V/2d,

Princeton University 1999

Ph501 Set 4, Solution 1

19

so the acceleration of an electron is a = eV/2dm, and the time to travel distance d is T = 2d/a = 2d m/eV . Hence, J = This is close to Childs Law (38). [This sign dierence between (38) and (42) is because the former is the current owing o the anode, while the latter is the current owing onto it.] We now make a detailed calculation, including the eect of the recharging of the cathode, which will reduce the average current density somewhat. At some time t, the charge sheet is at distance x(t) from the cathode, and the anode and cathode have charge densities A and C , respectively. All the eld lines that leave the anode terminate on either the charge sheet or on the anode, so + C = A . (43) 1 V 3/2 8 d2 e . m (42)

The magnitude of the electric eld strength in the region I between the anode and the charge sheet is (44) EI = 4A , and that in region II between the charge sheet and the cathode is EII = 4C . The voltage between the capacitor plates is therefore, V = EI (d x) + EII x = 4 A d V using (40) and (43-45). Thus, A = V x 1+ , 4d d C = Vx , 4d2 (47) x , d (46) (45)

and the time-dependent current density owing onto the anode is J (t ) = A = Vx . 4d2 (48)

This diers from the average current density (41) in that x/d = T , since x varies with time. To nd the velocity x of the charge sheet, we consider the force on it, which is due to the average eld set up by charge densities on the anode and cathode, Eon

= 2 (A + C ) =

V 2x 1+ . 2d d

(49)

Princeton University 1999

Ph501 Set 4, Solution 1

20

The equation of motion of an electron in the charge sheet is mx = eEon or x

eV 2d

1+

2x , d

(50)

eV eV . (51) x= 2 md 2md With the initial conditions that the electron starts from rest, x(0) = 0 = x (0), we readily nd that d x(t) = (cosh kt 1), (52) 2 where eV . (53) k= md2 The charge sheet reaches the anode at time T = 0.96 3 1 cosh1 = , k 2 k (54)

compared to T = 1/k as found above without the battery. The average anodoe-current density is, using (41) and (54), J = V V 3/2 = 4dT 4 cosh1 (3/2) d2 e V 3/2 = m 12.09 d2 e . m (55)

The electron velocity is x = so the anode-current density (48) is J= 1 V 3/2 8 d2 e sinh kt m (0 < t < T ). (57) dk sinh kt, d (56)

Princeton University 1999

Ph501 Set 4, Solution 2

21

2. Although current is owing inside the conducting sphere, it remains neutral. Hence, the potential satises Laplaces equation, 2 = 0. We analyze the problem in spherical coordinates (r, , ), with the origin at the center of the sphere of radius a, and = 0 and at the points of contact with the wires. The problem has axial symmetry, so will be independent of . We require the potential to be well behaved at the origin, so it can be expressed in a Legendre series,

=
n=0

An

r a

Pn (cos ) .

(58)

The convention that = 0 at the equator, = /2, implies that An = 0 for n even. Therefore, we can write r n An Pn (cos ) . (59) = a n odd To complete the solution, we need the boundary condition on at the surface of the sphere, r = a. We know that the radial component of the current density, jr is zero at the surface, except for the contact points where the current enters and exits. Since J = E = , we obtain a condition on the derivative of the potential at the boundary, J r (r = a ) = Er (r = a) = . (60) r r=a In the limit of very ne wires, the current density Jr (r = a) is zero except at the poles, so we can express it in terms of Dirac functions. The current dI that crosses an annular region on the surface of the sphere of angular extent d cos centered on angle is given by dI = 2a2Jr (a, )d cos . (61) Current I enters at cos = 1, and exits at cos = 1. Hence, the form Jr (a, ) = I [(cos 1) + (cos + 1)] 2a2 (62)

describes the entrance and exit currents upon integration of (61). Combining (59-60) and (62), we have I nAn [(cos 1) (cos + 1)] . Pn (cos ) = a 2a2 odd (63)

As usual, to evaluate the Fourier coecients we multiply by Pn (cos ) and integrate over d cos to nd 2I 2nAn = . (64) (2n + 1)a 2a2 Thus, the Legendre series expansion for the potential is (r, ) = I 2a n 2+
odd

1 n

r a

Pn (cos ).

(65)

Princeton University 1999

Ph501 Set 4, Solution 2

22

To express this series in closed form, we utilize the expansion for the distance R1 between the point (a, 0) and (r, ) given on p. 57 of the notes: 1 1 r = R1 a n=0 a 1 r 1 = R2 a n=0 a Hence, 1 2 1 = R1 R2 an It follows that
0 r odd n n

Pn (cos ) ,

(66)

Similarly, the distance R2 between the point (a, ) and (r, ) is Pn (cos()) = 1 r a n=0 a
n

Pn ( cos ) =

1 r (1)n a n=0 a

Pn (cos ). (67) (68)

r a

Pn (cos ).
n

1 1 R1 R2

2 dr = r an

1 r odd n a

Pn (cos ).

(69)

Then, (65) and (68-69) combine to give to the alternative form (3) for . As we approach the north pole, R1 0, and (we claim; details given later) the rst term in (3) dominates. That is, the potential diverges at the poles for the case of very ne wires. When considering actual wires of radius b, we suppose that our solution holds outside the region of contact between the wire and the sphere. Indeed, we expect that the potential is constant in planes perpendicular to the axis of the wire, so that the interface between the wire and the sphere is an equipotential. This cuts o the formal divergence in (3) near the poles. In this way, the potential at the interface is obtained from (3) on putting R1 = b and neglecting all but the rst term: interface = I/2b. The potential dierence across the sphere is twice this; b I =I IR. (70) V = b b2 Thus, the eective resistance of the sphere is R = b/(b2), which is also the resistance of a piece of wire of radius b, length b, and conductivity . To verify the claim that the rst term of (3) dominates for small R1 , we consider the point (r, ) = (a b, 0) for b a. Then, the rst term of (3) is 1/b, and the second term is 1/(2a b) which is negligible compared to the rst. Inside the integral term of (3), we have R1 = a r and R2 = a + r, so the integral is
ab 0

1 1 R1 R2

d ln r =
0

ab

1 2a 1 2a b 2 ln ln . dr = a2 r 2 a b a b

(71)

The ratio of the integral term to the rst term of (3) is therefore b 2a ln , 2a b which goes to zero as b becomes small. (72)

Princeton University 1999

Ph501 Set 4, Solution 3

23

3. a) Take the axis of the wire to be the z axis of a cylindrical coordinate system, (r, , z ). By rotational symmetry, there is no azimuthal component to the electric eld, and by translation symmetry, the axial and radial components can only depend on r. Similarly, there is no radial or axial component to the magnetic eld, and its azimuthal component is only dependent on r. Consider a cylindrical portion of the wire, of radius r and length l. The charge contained in this cylinder is then, Q = e(0 )r2 l, (73)

where e is the magnitude of the charge of an electron. From Gauss Law and (73), we have (74) 4Q = 42 er2 l(0 ) = E dS = 2rlEr (r), since the contributions from the at end surfaces are be equal and opposite. Hence, r + Ez z. E = 2e(0 )r (75)

The resulting radial force on the free electrons must be opposed by magnetic eects associated with the electron current, which we presume ows only in the z direction The current density in the wire is J = ev z, so from Amp` eres law, 2rB = 4 evr2 , c and B= 2 . evr c (77) (76)

The radial component of the Lorentz force on an electron, which must vanish if there is to be no current in radial direction, is then, Fr = e Er using (75) and (77). Hence, v2 0 = 1 2 , c (79) vz v2 B = e 2e(0 )r + 2e 2 r = 0, c c (78)

and the positive charge density is less than the negative by one part in 1021 for v = 1 cm/s. b) In this problem, we treat the resistor as a kind of conductive capacitor. Since the current density is uniform, the electric eld E is also. To maintain this electric eld, surface charge Q must reside at the ends of the resistor. From Gauss Law, Q= EA , 4 (80)

where A is the cross-sectional area of the resistor.

Princeton University 1999

Ph501 Set 4, Solution 3

24

If the current I into to resistor varies with time, then part of it goes to changing the charge Q at the ends of the resistor, and part of it appears as the conduction current IC across the resistor. Thus, IC is less than I according to IC = I Q. (81)

However, Maxwell advises us that inside the resistor we should also consider the displacement current, D E = Q, ID = = = Q (82) 4 4 using (80), in a medium whose dielectric constant is unity. Combining (81) and (82), the total current inside the resistor is thus, Itotal = IC + ID = I, which illustrates Maxwells notion that total currents are conserved. (83)

Princeton University 1999

Ph501 Set 4, Solution 4

25

4. The form of the current I (t) in a cylindrical straw tube can also be found without using the reciprocation theorem, so we illustrate that rst. Elementary Solution for I (t) The current that ows o the anode is equal to minus the rate of change of the charge q (t) < 0 that remains on the anode as the positive ions of total charge q0 move outward according to r(t). The key to an elementary solution is that although the positive ions occupy a very small volume around the point (r, , z ) = (r(t), 0, 0) in cylindrical coordinates, the charge they induce on the cathode is exactly the same as if those ions were uniformly spread out over a cylinder of radius r. Because the superposition principle holds in electrostatics, the problem of the chamber with voltage V on the anode plus ions at a xed position between the anode and cathode can be separated into two parts. First, an empty chamber with voltage V on the anode, and second, a grounded chamber with positive ions inside. [That is, we decompose the problem into cases A and B of the discussion of the reciprocation theorem, even though we wont use that theorem here.] For the second part, the radial electric eld in the region a < r < r(t) can be calculated from the charge q on the anode as E (r ) = 2q (t) , rl (84)

using Gauss Law, where l b is the length of the cylinder. Similarly, the electric eld in the region r(t) < r < b is 2(q0 + q (t)) . (85) E (r ) = rl The potential dierence between the inner and outer cylinder must be zero. Hence, 0= and so q (t ) = q 0 The current is I (t ) = q (t ) = 2q (t) l
r (t) a

dr 2(q0 + q (t)) + r l

dr 2q0 b 2q (t) b = ln + ln , l r (t ) l a r (t) r ln(b/r(t)) . ln(b/a) q 0 v (t ) . ln(b/a) r(t)

(86)

(87)

(88)

To calculate the dynamical quantities r(t) and v (t), we must return to the full problem of the ions in a chamber with voltage V . The electric eld in the chamber is only slightly perturbed by the presence of the ions, and so is given by E (r ) = V . r ln(b/a) (89)

Princeton University 1999

Ph501 Set 4, Solution 4

26

According to (5), the positive ions have velocity v (r ) = which integrates to give r 2 (t ) = a 2 + Inserting (90-91) in (88), we nd I (t ) = where t0 = 1 q0 , 2t0 ln(b/a) 1 + t/t0 a2 ln(b/a) . 2V (92) 2V t. ln(b/a) (91) V , r ln(b/a) (90)

(93)

The idealized current pulse has a very sharp rise, and falls o rapidly over characteristic time t0, which is about 20 nsec in typical straw tube chambers. I (t) via Reciprocity Referring to the prescription in the statement of the problem, we rst solve case C, in which the inner electrode is at unit potential and the outer electrode is grounded. We quickly nd that ln(b/r) . (94) VC (r) = ln(b/a) According to (11), the current o the inner electrode is therefore, I (t ) = q 0 q 0 v (t ) dVC v (r ) = , dr ln(b/a) r(t) (95)

as previously found in (88). We again solve for v and r(t) as in (89-91), which corresponds to the use of case A, to obtain the solution (92-93). The Charge Distribution q (z ) on the Cathode The more detailed question as to the longitudinal charge distribution on the cathode can be solved by the reciprocation method if we conceptually divide the cathode cylinder into a ring of length dz at position z1 plus two cylinders that extends to z = l/2 where l is the length of the cylinder. We label the ring as electrode 1, and calculate the charge Q1 = q (z )dz induced on this ring when the positive ion charge q0 is at position (r0, 0, z0 ) in cylindrical coordinates (r, , z ). According to the prescription (10) given in the statement of the problem, Q1 = q0VC (r0 , 0, z0 ), (96)

where case C now consists of a cylinder of radius b grounded except for the ring at position z1 at unit potential, and a grounded cylinder at radius a. For z not close to

Princeton University 1999

Ph501 Set 4, Solution 4

27

the ends of the cylinder, the end surfaces z = l/2 may be approximated as at ground potential. This problem is very similar to that discussed in sec. 5.36 of W.R. Smythe, Static and Dynamic Electricity, 3rd ed. (Mcgraw-Hill, New York, 1968). Laplaces equation, 2 C (r) = 0 holds for the potential in the region a < r < b. The problem has azimuthal symmetry, so C will be independent of . Since the planes z = l/2 are grounded, the longitudinal functions in the Fourier series expansion, C =
n

Rn (r)Zn (z ),

(97)

must have the form Zn = sin 2nz/l. The equation for the radial functions Rn (r) follows from Laplaces equation as 2n d2 Rn 1 dRn + 2 dr r dr l
2

Rn = 0.

(98)

The solutions of this are the modied Bessel functions of order zero, I0(2nr/l) and K0 (2nr/l). Both of these are nite on the interval a < r < b, so the expansion (97) will include them both. The boundary condition that C (a, , z ) = 0 is satised by the expansion C =
I0 (2nr/l) I0 (2na/l) An I0 (2nb/l) n I0 (2na/l)

K0 (2nr/l) K0 (2na/l) K0 (2nb/l) K0 (2na/l)

sin

2nz , l

(99)

where the form of the denominator is chosen to simplify the evaluation of the boundary condition at r = b. Here, C = 0, except of an interval dz long about z where it is unity. Hence, the Fourier coecients are An = 2 2nz1 sin dz. l l (100)

In sum, the charge distribution q(z) on the cathode at radius b due to positive charge q0 at (r0 , 0, z0) follows from (96) and (98-99) as 2q0 q (z ) = l
I0 (2nr0 /l) I0 (2na/l) I0 (2nb/l) I0 (2na/l)

K0 (2nr0 /l) K0 (2na/l) K0 (2nb/l) K0 (2na/l)

sin

2nz 2nz0 sin . l l

(101)

A numerical evaluation of (101) is illustrated in Fig. 1. As is to be expected, the induced charge distribution on the cathode has characteristic width of order b r0 , the distance of the positive charge from the cathode.

Princeton University 1999

Ph501 Set 4, Solution 4

28

1.2 0.9
Q (rel.)

0.6 0.3

1.0 0.0 0.8 0.6 0.4 0.2 0.0


R/ Rc

-1.0

-0.5

0.0 Z (cm)

0.5

1.0

Figure 1: The induced charge distribution (101) on the cathode of a straw tube chamber of radius RC = 0.25 cm due to positive ion charge at radius R.

Princeton University 1999

Ph501 Set 4, Solution 5

29

5. This problem is posed on p. 363 of The Mathematical Theory of Electricity and Magnetism, 5th ed., by James Jeans. We will evaluate the resistance R via Ohms Law, R = V/I , by calculating the current I that ows when a potential dierence V is established between the two contacts. For a thin disk, the current ow is 2-dimensional. Since J = E, where J is the current density and E is the electric eld, the electric eld is 2-dimensional also. And, since E = , where is the electric potential, the potential is 2-dimensional as well. The form of the 2-dimensional potential is well approximated (for distances more than /2 from the centers of the contacts) by considering a cylinder of radius a, rather than the disk, with a line charge density that passes through the center of one contact, and line charge that passes through the center of the other contact. The electric eld from the wire of charge density has magnitude E1 = 2 , r1 (102)

according to Gauss law, where r1 is the distance from the wire to the observer. The corresponding electric potential is 1 = 2 ln r1 , r0 (103)

where r0 is a constant of integration. The potential due to the wire with charge density is similarly r2 (104) 2 = 2 ln , r0 where r2 is the distance from the observer to wire 2. The potential at an arbitrary point is then given by r1 (105) = 1 + 2 = 2 ln . r2 The total potential dierence between the two line charges is formally divergent. To make physical sense, we can suppose that expression (105) holds only for r1 and r2 greater than /2, the half width of the electrical contacts, and the potential is essentially constant for smaller values of r1 and r2. That is, we approximate the contacts of width by perfectly conducting wires of radii /2, as shown in the gure below. Then, the potential of contact 2 is estimated from eq. (105) by setting r1 = d /2 and r2 = /2 (contact 2) = 2 ln 2d d /2 2 ln . /2 (106)

The potential at the surface of contact 1 is just the negative of eq. (106), so the potential dierence is 2d (107) V 4 ln .

Princeton University 1999

Ph501 Set 4, Solution 5

30

We note that the current and the electric eld must be tangential to the edge of the disk. We recall that the equipotentials of eq. (105) are circles, and that the corresponding electric eld lines are also circles which, of course, pass through the line charges. Hence, the boundary condition on the electric eld at the edge of the disk is indeed satised. To complete the solution, we must calculate the current I that is owing. For this, we can integrate the current density J across any surface between the two contacts. For convenience, consider a cylindrical surface of radius r centered on one of the contacts, such that /2 < r d. Since r d, this surface is essentially an equipotential, and the electric eld is essentially that due to the nearby charge density . Namely, the electric eld is normal to this surface, with magnitude E= 2 . r (108)

The current density across this surface is given by J = E . Restricting the problem to a disk of thickness t, the relevant area of the surface is rt, so the total current is I = rt which is independent of the choice of r. Finally, the resistance is found by combining eqs. (107) and (109): R= 4 ln 2d/ 2 2d V = ln , I 2t t (110) 2 = 2t, r (109)

independent of the radius a of the disk.

Princeton University 1999

Ph501 Set 4, Solution 6

31

6. The series expansion approach is unsuccessful in treating the full problem of a checkerboard array of two phases if those phases meet in sharp corners as shown above. However, an analytic form for the electric potential of a two-phase (and also a four-phase) checkerboard can be obtained using conformal mapping of certain elliptic functions; see R.V. Craster and Yu.V. Obnosov, Checkerboard composites with separated phases, J. Math. Phys. 42, 5379 (2001).4 If the regions of one phase are completely surrounded by the other phase, rather lengthy series expansions for the potential can be given; see Bao Ke-Da, J orger Axell and G oran Grimvall, Electrical conduction in checkerboard geometries, Phys. Rev. B 41, 4330 (1990).5 The present problem is based on M. S oderberg and G. Grimvall, Current distribution for a two-phase material with chequer-board geometry, J. Phys. C: Solid State Phys. 16, 1085 (1983),6 and Joseph B. Keller, Eective conductivity of periodic composites composed of two very unequal conductors, J. Math. Phys. 28, 2516 (1987).7 In the steady state, the electric eld obeys E = 0, so that E can be deduced from a scalar potential via E = . The steady current density obeys J = 0, and is related to the electric eld by Ohms law, J = E. Hence, within regions of uniform conductivity, E = 0 and 2 = 0. Thus, we seek solutions to Laplaces equations in the four regions of uniform conductivity, subject to the stated boundary conditions at the outer radius, as well as the matching conditions that , E , and j are continuous at the boundaries between the regions. We analyze this two-dimensional problem in a cylindrical coordinate system (r, ) with origin at the corner between the phases and = 0 along the radius vector that bisects the region whose potential is unity at r = a. The four regions of uniform conductivity are labeled I , II , III and IV as shown below.

Since J = Jr = Er = /r at the outer boundary, the boundary conditions at r = a can be written I (r = a) = 1,


4 5

(111)

http://puhep1.princeton.edu/~mcdonald/examples/EM/craster_prsla_456_2741_00.pdf http://puhep1.princeton.edu/~mcdonald/examples/EM/ke-da_prb_41_4330_90.pdf 6 http://puhep1.princeton.edu/~mcdonald/examples/EM/soderberg_jpc_16_1085_83.pdf 7 http://puhep1.princeton.edu/~mcdonald/examples/EM/keller_jmp_28_2516_87.pdf

Princeton University 1999

Ph501 Set 4, Solution 6

32

IV (r = a) II (r = a) = = 0, r r III (r = a) = 1.

(112) (113)

Likewise, the condition that J = J = E = (/r)/ is continuous at the boundaries between the regions can be written I ( = ) III ( = ) 1 1 = = etc . From the symmetry of the problem we see that ( ) = ( ), ( ) = (), and in particular (r = 0) = 0 = ( = /2). We recall that two-dimensional solutions to Laplaces equations in cylindrical coordinates involve sums of products of rk and eik , where k is the separation constant that in general can take on a sequence of values. Since the potential is zero at the origin, the radial function is only rk . The symmetry condition (116) suggests that the angular functions for region I be written as cos k, while the symmetry condition (117) suggests that we use sin k (/2 ||) in regions II and IV and cos k ( ) in region III . That is, we consider the series expansions I = II = IV = Ak rk cos k, Bk rk sin k | | , 2 Ak rk cos k ( ). (118) (119) (120) (116) (117) II ( = ) , ( = ) 2 II , 2 (114) (115)

III =

The potential must be continuous at the boundaries between the regions, which requires Ak cos k = Bk sin k . 2 (121)

The normal component of the current density is also continuous across these boundaries, so eq. (114) tells us that 1Ak sin k = 2 Bk cos k On dividing eq. (122) by eq. (121) we nd that tan k = 2 cot k . 1 2 (123) . 2 (122)

Princeton University 1999

Ph501 Set 4, Solution 6

33

There is an innite set of solutions to this transcendental equation. When 2 / 1 1 we expect that only the rst term in the expansions (118)-(119) will be important, and in this case we expect that both k and k (/2 ) are small. Then eq. (123) can be approximated as 2 / 1 , (124) k k ( 2 ) and hence k2 2 /1 ( ) 2 1. (125)

Equation (121) also tells us that for small k, Ak Bk k . 2 (126)

Since we now approximate I by the single term Ak rk cos k Ak rk , the boundary condition (111) at r = a implies that Ak and eq. (126) then gives Bk 1 kak ( 2 ) Ak . (128) 1 , ak (127)

The boundary condition (112) now becomes 0 = kBk ak1 sin k ) k( 2 , 2 a( 2 ) (129)

which is approximately satised for small k . So we accept the rst terms of eqs. (118)-(120) as our solution, with k , Ak and Bk given by eqs. (125), (127) and (128). In region I the electric eld is given by Er = E rk1 I rk1 k k cos k k k , r a a k1 k1 r 1 I r k k sin k k 2 k . = r a a (130) (131)

Thus, in region I , E /Er k 1, so the electric eld, and the current density, is nearly radial. In region II the electric eld is given by Er = E = rk1 II rk1 k k sin k k k r ka ( 2 ) 2 a
2 2

(132) (133)

1 II rk1 rk1 k k cos k k . r ka ( 2 ) 2 a ( 2 )

Princeton University 1999

Ph501 Set 4, Solution 6

34

Thus, in region II , Er /E k (/2 ) density, is almost purely azimuthal.

1, so the electric eld, and the current

The current density J follows the lines of the electric eld E, and therefore behaves as sketched below:

The total current can be evaluated by integrating the current density at r = a in region I: 1 2 . 0 0 0 2 (134) In the present problem the total potential dierence is -2, so the eective conductivity is 1 2 I = . (135) = 2 I = 2a Jr d = 2a1 Er (r = a)d 2k 1 d = 2k 1 = 2 For a square checkerboard, = /4, and the eective conductivity is = 1 2 . It turns out that this result is independent of the ratio 2/ 1 , and holds not only for the corner region studied here but for the entire checkerboard array; see Joseph B. Keller, A Theorem on the Conductivity of a Composite Medium, J. Math. Phys. 5, 548 (1964).8

http://puhep1.princeton.edu/~mcdonald/examples/EM/keller_jmp_5_548_64.pdf

Princeton University 1999

Ph501 Set 4, Solution 7

35

7. Taking the wire that carries current I to be along the z axis, the magnetic eld at distance r from the wire is 2I B= . (136) cr The force on an element dl of the loop that carries current I is dF = I dl B. c (137)

The torque about the axis of the loop due to that force element is dF , dN = sn (138)

is the unit vector from the where s is the distance from the axis to the element and n axis to the element. Combining (136-138), the total torque on the loop is N= 2II c ) (dl sn . r (139)

The two sides of length 2b will have equal and opposite contributions which therefore cancel, leaving only the contributions from the two sides of length 2a. For the side nearest the wire, s = b, and dl = zdl, since the currents ow in the same direction, so = that dl rneardl (parallel currents attract), ) = bn (dl sn rneardl = b sin zdl = using the law of sines for the triangle shown. bd sin zdl, rnear (140)

U QHDU
G

U IDU

The distance r from the wire to this side of the loop is rnear = b2 + d2 2bd cos , so that the contribution of this side of the loop to the torque is Nnear = 4abdII sin z. 2 2 c b + d 2bd cos

(141)

(142)

= For the side furthest the wire, again s = b, but dl = zdl, so dl rfardl. Then, bd sin ) = bn (dl sn rfardl = b sin zdl = zdl. rfar (143)

Princeton University 1999 and

Ph501 Set 4, Solution 7

36

rfar =

b2 + d2 + 2bd cos ,

(144)

so that the contribution of the far side of the loop to the torque is Nfar = sin 4abdII z. 2 2 c b + d + 2bd cos (145)

The total torque on the loop is then, N = Nnear + Nfar = 8abdII (b2 + d2 ) sin z, c b4 + d4 2b2 d2 cos 2 (146)

where the identity 2 cos2 = cos 2 + 1 has been used. This torque is down the axis, that is, it acts to decrease and bring the loop into the plane of the wire and the axis.

Princeton University 1999

Ph501 Set 4, Solution 8

37

8. a) The magnetic eld on the axis of the loop centered at z = b is obtained from the Biot-Savart law: I dl r 2Ia2 B1 = = z, (147) 3 c r3 c(a2 + (z b)2 ) 2 and that due to the other loop is B2 = 2Ia2 c(a2 + (z + b)2) 2
3

z.

(148)

The total eld is then the sum of the (147) and (148). This is unchanged under z z , so all odd derivatives with respect to z automatically vanish at the origin. We can choose the separation b to cancel any desired even derivative at the origin. We rst accumulate a catalog of derivatives (some of which are needed in prob. 6): 1 1 cBz (z ) = 3 + 3 , 2 2Ia ( a 2 + (z b ) 2 ) 2 (a2 + (z + b)2 ) 2 3(z b) 3(z + b) cBz (z ) = 5 5 , 2Ia2 (a2 + (z b)2) 2 (a2 + (z + b)2) 2 cBz (z ) 12(z b)2 3a2 12(z + b)2 3a2 = 7 + 7 . 2Ia2 (a2 + (z b)2) 2 (a2 + (z + b)2 ) 2 90(z b)3 15a2 (z b) 90(z + b)3 15a2 (z + b) cBz (z ) = , 9 9 2Ia2 (a2 + (z b)2) 2 (a2 + (z + b)2) 2 cBz (z ) 540(z b)4 450a2 (z b)2 + 15a4 = 11 2Ia2 (a2 + (z b)2) 2 + 540(z + b)4 450a2 (z + b)2 + 15a4 (a2 + (z + b)2 ) 2
11

(149) (150) (151) (152)

(153)

The second derivative of Bz with respect to z at the origin is proportional to 4b2 a2 (a 2 + b 2 ) 2


7

(154)

which vanishes when a = 2b. That is, the separation of a pair of Helmholtz coil is equal to their radius. b) In a Helmholtz coil pair, the eld at the origin is proportional to I/a according to (149), so the 4rth derivative at the origin is proportional to I/a5. If we add a second Helmholtz pair with current I and radius a = 2/a, the combined 4th derivative at the origin is proportional to I/a5 + 32I /a5. Hence, the current I = I/32 will cancel the 4rth derivative at the origin. The eld at the origin is then proportional to I/a (I/32)(2/a) = 15I/16. That is, the central eld has been reduced by 1/16. c) Far outside the coils, the leading behavior of the magnetic eld is due to the dipole moment, 2Ia2/c, of the coils. If a second Helmholtz pair is placed at a = 2a to cancel the dipole moment of the rst pair, we need I = I/4. Since the central eld of a

Princeton University 1999

Ph501 Set 4, Solution 8

38

Helmholtz pair varies as I/a, the combined central eld will be I/a (I/4)(1/2a) = 7I/8a, i.e., 1/8 of the central eld is lost to insure that the eld far outside the coils is extremely weak. To characterize the eld well outside a set of Helmholtz coils in more detail, we use the variable u = 1/z , and expand about u = 0. From (149), and using a = 2b,
(0) u3 u3 (u ) cBz = + 3 3 . 2Ia2 (1 au + 5a2u2 /4) 2 (1 + au + 5a2u2 /4) 2

(155)

Using the Taylor expansion, 1 (1 + ) we nd that


3 2

= 1

3 15 + 2 8

105 48

+ ....,

(156)

4Ia2 3 75 3 6 u + a u + ... . (157) c 16 The leading term, u3 = 1/z 3 , is due to the dipole moment of the pair, which, of course, is proportional to Ia2. The next nonvanishing term, u6 , is due to the hexadecupole moment, which is proportional to Ia5. The brevity of this derivation hides that fact that the Helmholtz condition, a = 2b, served to cancel the octupole moment. The quadrupole moment vanishes due to the symmetry of the coil pair.
(0) (u ) = Bz

Princeton University 1999

Ph501 Set 4, Solution 9

39

9. Since the divergence of the magnetic eld vanishes, the proposed expansions (15) obey B= 1 Br Bz + = r r z
n (n + 1)bn rn1 + a(1) = 0, n r

(158)

where a(m)(z ) dm a/dz m . For this to be true at all r, the coecients of rn must separately vanish for all n. Hence, b0 = 0, bn = Since the curl of the eld vanishes, ( B ) = Br Bz = z r
n n1 b(1) =0, n r nan r (1) an1

(159) n+1 . (160)

(161)

Again, the coecient of rn must vanish for all n, so that b(1) n = (n + 1)an+1 . Using (162) in (160), we nd bn = bn2 . (n + 1)(n + 3)
(2)

(162)

(163)

Since b0 vanishes, b2n vanishes for all n, and from (162), a2n+1 vanishes for all n. Then, using (163) in (162), we nd (2) a 2 a2n = 2n . (164) 4n2 Repeatedly applying this to itself gives a a2n = (1) 2n0 2 . 2 (n!)
n (2n)

(165)

Inserting this in (160), we get b2n+1 = (1)n+1 a0 . 2 n +1 2 (n + 1)(n!)2


(2n+1)

(166)

Combining (165-166) with (15), we arrive at the desired forms (16-17) for the elds. The axial eld of a pair of Helmholtz coils has the form a0(z ) = B0 + B4 z 4 + ... The rst four derivatives are a0 = 4B4 z 3 ,
(1)

(167)

a0 = 12B4 z 2 ,

(2)

a0 = 24B4 z,

(3)

a0 = 24B4 .

(4)

(168)

Princeton University 1999

Ph501 Set 4, Solution 9

40

From (165-166), the other non-vanishing functions through fourth order are a2 = 3B4z 2 , a4 = 3B4 , 8 b1 = 2B4 z 3 , b3 = 3B4 z. 2 (169)

The elds, correct to fourth order, are Bz = B0 + B4z 4 3B4 r2 z 2 + Br = 2B4 rz 3 + 3B4 4 r + ... , 8 (170) (171)

3B4 3 r z + ... 2

The constants B0 and B4 are obtained from the catalog of derivatives in prob. 5, using Bz (0, z ) = B0 + B4 z 4 = Bz (0, z ) + From (149), 4Ia2 Bz (0, z )z 4 + ... 4! (172)

32 5I B0 = , 3 = 25ca c (a 2 + b 2 ) 2 5Ia2(a4 30a2 b2 + 36b4 ) 2c(a2 + b2) 2


11

(173)

and from (153), B4 = 4864 5I 152 = = B0 , 3125ca5 125a4 (174)

using b = a/2. Since B4 < 0, the axial eld Bz decreases as we move away from the origin, as is to be expected. These results are overly detailed for some purposes. If one is interested only in the leading behavior at small r, then (170-171) simplify to Bz (r, z ) Bz (0, z ), Br (r, z ) r Bz (0, z ) . 2 z (175)

The result for Br also follows quickly from B = 0, according to eq. (158), Br (r, z ) =
r 0

Bz (r, z ) dr z

r 0

Bz (0, z ) r Bz (0, z ) dr = . z 2 z

(176)

It is also instructive that the approximation (176) can be deduced quickly from the integral form of Gauss law (without the need to recall the form of B in cylindrical coordinates). Consider a Gaussian pillbox of radius r and thickness dz centered on (r = 0, z ). Then, 0 = B dS r2 [Bz (0, z + dz ) Bz (0, z )] + 2r dz Br (r, z ) Bz (0, z ) + 2r dz Br (r, z ) , z (177)

r2 dz

which again implies eqs. (175).

Princeton University 1999

Ph501 Set 4, Solution 10

41

10. We rst seek a solution in rectangular coordinates, and expect that separation of variables will apply. Thus, we consider the form Bx = f (x)g (y ) cos kz, Bx = F (x)G(y ) sin kz, Bz = A(x)B (y )C (z ). Then B = 0 = f g cos kz + F G sin kz + ABC , (181) where the indicates dierentiation of a function with respect to its argument. Equation (181) can be integrated to give ABC = FG fg sin kz + cos kz. k k (182) (178) (179) (180)

The z component of B = 0 tells us that Bx By = fg cos kz = = F G sin kz, y x which implies that g and F are constant, say 1. Likewise, f Bx Bz = fk sin kz = = A BC = sin kz, z x k using (182-183). Thus, f k 2 f = 0, so f = f1 ekx + f2 ekx . Finally, By G Bz = Gk cos kz = = AB C = sin kz, z y k so G = G1 eky + G2 eky . (186) (187) (185) (184) (183)

The boundary conditions f (0) = B0 = G(0) are satised by f = B0 cosh kx, G = B0 cosh ky, (188)

which together with (182) leads to the solution Bx = B0 cosh kx cos kz, By = B0 cosh ky sin kz, Bz = B0 sinh kx sin kz + B0 sinh ky cos kz, This satises the last boundary condition that Bz (0, 0, z ) = 0. However, this solution does not have helical symmetry. (189) (190) (191)

Princeton University 1999

Ph501 Set 4, Solution 10

42

Suppose instead, we look for a solution in cylindrical coordinates (r, , z ). We again expect separation of variables, but we seek to enforce the helical symmetry that the eld at z + be the same as the eld at z , but rotated by angle k. This symmetry implies that the argument kz should be replaced by kz , and that the eld has no other dependence. We begin constructing our solution with the hypothesis that Br = F (r) cos(kz ), B = G(r) sin(kz ). (192) (193)

To satisfy the condition (24) on the z axis, we rst transform this to rectangular components, Bz = F (r) cos(kz ) cos + G(r) sin(kz ) sin , By = F (r) cos(kz ) sin + G(r) sin(kz ) cos , from which we learn that the boundary conditions on F and G are F (0) = G(0) = B0 . A suitable form for Bz can be obtained from ( B)r = 0: B 1 Bz = = kG cos(kz ), r z so Bz = krG sin(kz ), which vanishes on the z axis as desired. From either ( B) = 0 or ( B)z = 0 we nd that F = Then, B = 0 leads to
2 2 d (krG) (kr) d(kr)2

(194) (195)

(196)

(197)

(198)

d(rG) . dr

(199)

+ kr

d(krG) [1 + (kr)2 ](krG) = 0. d(kr)

(200)

This is the dierential equation for the modied Bessel function of order 1. See, for example, M. Abramowitz and I.A. Stegun, Handbook of Mathematical Functions (National Bureau of Standards, Washington, D.C., 1964), sec. 9.6. Hence, C (kr)2 I1(kr) = + , 1+ kr 2 8 I1 C 3(kr)2 dI1 = C I0 + . = 1+ F =C d(kr) kr 2 8 G=C (201) (202)

Princeton University 1999

Ph501 Set 4, Solution 10

43

The boundary conditions (196) require that C = 2B0 , so our second solution is Br = 2B0 I0 (kr) B = 2B0 Bz I1(kr) cos(kz ), kr (203) (204) (205)

I1 sin(kz ), kr = 2B0 I1 sin(kz ),

which is the form discussed by Blewett and Chasman. For a realization of the axial eld pattern (24), we consider a wire that carries current I and is wound in the form of a helix of radius a and period = 2/k . A suitable equation of this helix is x1 = a sin kz, y1 = a cos kz. (206)

The magnetic eld due to this winding has a nonzero z component along the axis, which is not desired. Therefore, we also consider a second helical winding, x2 = a sin kz, y2 = a cos kz, (207)

which is oset from the rst by half a period and which carries current I . The combined magnetic eld from the two helices has no component along their common axis. The unit vector l1,2 that is tangent to helix 1(2) at a point r1,2 = (x1,2, y1,2, z ) = (a sin kz , a cos kz , z ) has components (2a cos kz , 2a sin kz , ) , l 1 ,2 = 2 + (2a)2 (208)

(209)

and the element dl1,2 of arc length along the helix is related by dl1,2 = l1,2dz 2 + (2a)2 = dz (ka cos kz , ka sin kz , 1). (210)

The magnetic eld B at a point r = (0, 0, z ) on the axis is given by B(0, 0, z ) = I dl1 (r1 r) I dl2 (r2 r) c 1 |r1 r|3 c 2 |r2 r|3 2Ia dz = [ x(k (z z ) sin kz + cos kz ) 2 c [a + (z z )2 ]3/2 + y(k (z z ) cos kz + sin kz )] dt 2I = [ x(kat sin(kat + kz ) + cos(kat + kz )) ca (1 + t2 )3/2 + y(kat cos(kat + kz ) + sin(kat + kz ))] (211) t sin kat 4Ik 1 cos kat sin kz ) ( x cos kz + y dt + dt , = c ka 0 (1 + t2)3/2 (1 + t2)3/2 0

Princeton University 1999

Ph501 Set 4, Solution 10

44

where we made the substitution z z = at in going from the second line to the third. Equation 9.6.25 of Abramowitz and Stegun tells us that
0

cos kat dt = kaK1 (ka) , (1 + t2 )3/2

(212)

where K1 also satises eq. (200). We integrate the last integral by parts, using u = sin kat, Thus, dv = t dt , (1 + t2)3/2 so du = ka cos kat dt, 1 v = . 1 + t2 (213)

cos kat dt = kaK0 (ka) , 0 0 1 + t2 using 9.6.21 of Abramowitz and Stegun. Hence B(0, 0, z ) = 4Ik sin kz ). [kaK0(ka) + K1 (ka)] ( x cos kz + y c

t sin kat dt = ka (1 + t2 )3/2

(214)

(215)

Both K0 (ka) and K1 (ka) have magnitudes 0.5eka for ka 1. That is, the eld on the axis of the double helix is exponentially damped in the radius a for a xed current I .

Princeton University 1999

Ph501 Set 4, Solution 11

45

11. We analyze the magnetic eld of the solenoid in cylindrical coordinates, (r, , z ), with the origin at the center of the solenoid and z axis along that of the solenoid. First, the eld at a point on the axis, (0, 0, z ), to a current loop, with current dI , centered on and perpendicular to the z -axis at z follows from the Biot-Savart law as B(0, 0, z ) = For the solenoid, dI = nI dz , where z runs from l/2 to l/2, so the total eld on the axis is B(0, 0, z ) = = 2nIa2 z c 2nIa2 z c
l 2 l 2

dI c

dl r 2dIa2 , = 3 z r3 c(a2 + (z b)2) 2

(216)

dz
3

(a2 + (z z )2) 2 l z dz 2 (a 2 + z 2 ) 2
l 2
3

l z 2nI z 2 = + l c a2 + ( 2 z )2 2nI = z, (cos 1 + cos 2) c

l z 2

+z

l a2 + ( 2 + z )2

(217)

where 1 is the angle between the z axis and the line joining the observation point, (0, 0, z ) to the point (a, 0, l/2) on the end of the solenoid, etc. For z a l, we use the next to last line of (217), and the Taylor expansion to nd that near the origin, Bz (0, 0, z ) 2nI 4a2 72a2 z 2 2 2 c l l4 . (219) 1 1+ =1 + 32 53 + ..., 8 16 (218)

As noted as the end of prob. 6, the radial eld near the axis can be obtained from the axial eld using (175). Hence, 288nIa2rz Br (r, 0, z ) . cl4 Near the end of the solenoid at z = l/2, cos 1 = sin(1 /2) z l/2 , a and cos 2 l a2 1 . 2l2 l 2 + a2 (221) (220)

Princeton University 1999 Then, from (217)

Ph501 Set 4, Solution 11

46

Bz (0, 0, z )

2nI a2 z l/2 1 2 c 2l a

(222)

Comparing with (219), we see that the axial eld at the end of the solenoid is approximately 1/2 that at the center. The radial eld at the end of the solenoid follows from (175) as nIr . (223) Br ac

Princeton University 1999

Ph501 Set 4, Solution 12

47

12. a) As discussed on p. 98 of the Notes, a sphere of uniform magnetization has a uniform magnetic eld inside. As discussed on p. 93, the eld associated with magnetization density M can be thought of as arising from a magnetization current density, Jm = , where n is the outward normal c M, and a surface current density, Km = cM n , from the bounding surface. For uniform magnetization, Jm = 0, while, if M = M z then . Km = cM sin (224) Since this is to be produced by windings on the surface of the sphere, with the same current owing through each turn of the winding, the density of windings must be proportional to sin . b) If the sphere has radius a and surface charge density and rotates with angular velocity , its magnetic moment will be = 1 dQ 2a sin a d 1 1 2 2 Area dI = Area a sin = c c T c 0 2/ 4 4 2 4a Qa a sin3 d = = , = c 3c 3c 0

(225)

where Q = 4a2 is the total charge on the sphere. For the classical model of the electron with magnetic moment = eh /2mc, the velocity at the equator is v = a = 3h 3 eh 1 mc2 3c c = c= =3 c c. 2 2 Qa 2mc e e 2e 2 (226)

Since the ne structure constant = eh /c2 , is 1/137, this velocity is more than two hundred times the speed of light!

Princeton University 1999

Ph501 Set 4, Solution 13

48

13. a) If n is the number density of alignable electrons, each of magnetic moment = eh /2mc, then the resulting bulk magnetic eld strength is B = 4M = 4n = 2neh 2nmc2 eh , = 2n 2 3 mc2 = mc mc Bcrit (227)

where Bcrit = m2c3 /eh = 4.4 1013 Gauss is the so-called QED critical eld strength. Since mc2 = 0.511 MeV = 8.2 107 erg we have n= 2 104 4.4 1013 = 1.7 1023 /cm3 . 2 8.2 107 (228)

Iron has atomic weight A = 56 and mass density 8 g/cm3 , so its number density is natom = 6 1023 8 = 8.6 1022 /cm2. 56 (229)

Thus, two electrons per iron atom participate in its bulk magnetization. b) The interaction energy U of two magnetic dipoles m1 and m2 of equal magnitude z can be calculated by supposing that, say, dipole m separated by distance r2 r1 = d 2 is held xed while dipole 1 is brought into place from a large distance. The force on dipole 1 due to dipole 2 is then F1 = 1 (m1 B2), which can be integrated to give the interaction energy U = F1 dr1 = m1 B2 = m2 B1, (231) (230)

where the second form follows from a similar argument in which dipole 1 was held xed while dipole 2 is moved into place. The eld of a dipole is B1 = so U = If dipole 1(2) makes angle 1(2) 3(m1 r) r m1 , 3 r (232)

3(m1 z)(m2 z) m 1 m 2 . (233) 3 d to the z axis, and both lie in, say, the x-z plane, then m2 [3 cos 1 cos 2 cos(1 2 )]. d3 (234)

U ( 1 , 2 ) =

This is a minimum for 1 = 2 = 0 or , and Umin = 2m2 . d3 (235)

In the absence of an external eld, the dipoles are aligned, and both are either parallel or antiparallel to the z axis.

Princeton University 1999

Ph501 Set 4, Solution 13

49

We now add a uniform external magnetic eld B that makes angle to the z axis in the x-z plane. The interaction energy of the system is then, m2 U (1 , 2, ) = 3 [3 cos 1 cos 2 cos(1 2 )] mB [cos( 1) + cos( 2 )]. (236) d First, consider when B is at right angles to the line of centers of the dipoles, = /2. Then, U ( 1 , 2 ) = m2 [3 cos 1 cos 2 cos(1 2 )] mB [sin 1 + sin 2 ]. d3 (237)

If the dipoles remain in their original orientation, say 1 = 2 = 0, then the interaction energy U0 is still given by (235). Suppose the two dipoles rotate together towards B. Then, U ( 1 = 2 ) = m2 [3 cos2 1 1] 2mB sin 1 d3 m2 2d3 B 2 = 3 sin 2 sin 1 . 1 d3 m

(238)

The dipoles will rotate from angle 0 to /2 provided that U ( 1 = 2 ) decreases monotonically along this path. Since (238) is a quadratic function of sin 1, this requires that the minimum occur for sin 1 1. The critical condition is then B= 3m , d3 (239)

above which eld strength the dipoles always align with the transverse B. Thus, (239) is the saturation magnetic eld. We note that for B < 3m/d3 , the energy minimum occurs at sin 1 = sin 2 = d3 B/3m < 1. From (237), it can be veried that this is the absolute minimum for all (1 , 2 ). So the equilibrium conguration falls short of full alignment with B until the eld is larger than the saturation value (239). Finally, we consider the case where the dipoles originally have 1 = 2 = 0, and external eld B = B z is applied. We rst suppose that the dipoles rotate together, 1 = 2, so from (236) the energy of an intermediate state is U ( 1 = 2 ) = m2 2d3 B 2 cos 1 . 3 cos 1 1 d3 m (240)

This form is concave downward in cos 1 , so the alignment can occur so long as the maximum occurs at cos 1 1. Thus, on this path the critical condition is again (239). We might wonder whether the alternative path, 2 = 1 leads to a lower critical eld. From (236), m2 2d3 B cos 1 . U (1 = 2 ) = 3 cos2 1 + 1 (241) d m

Princeton University 1999

Ph501 Set 4, Solution 13

50

This is also concave downward in cos 1 , so requiring the maximum to occur at cos 1 = 1 leads to the critical condition m B= 3. (242) d In our model of a permanent magnet as consisting of only 2 magnetic dipoles, we nd that it is favorable for the transition from one ferromagnetic (aligned dipoles) state to another at 180 due to application of an external eld to pass through an antiferromagnetic state (anti-aligned dipoles). We leave it to statistical mechanics to decide whether this can occur in a system of a large number of magnetic dipoles.

Princeton University 1999

Ph501 Set 4, Solution 14

51

14. The technique of mapping two dimensional magnetostatic elds with conducting paper is NOT based on the analogy between electrostatics and magnetostatics that was mentioned on p. 97 of the Notes. Rather, we start from (32-33). The 2-dimensional magnetic eld derived from the vector potential Az z is B=A= Az Az , ,0 . y x (243)

With the identication Az = k we can write E = = , x y = 1 k Az Az , x y = 1 B z (By , Bx) = . k k (244)

Hence, B is perpendicular to E and therefore parallel to equipotentials of . When current I is feed into a region of conducting paint, it spreads out on the paper as described by current density J. Then, current conservation and Ohms law allow us to write I z= J dl = 1 E dl = k (B z) dl = z k B dl = 4IB z, ck (245)

where IB is the current needed to produce magnetic eld B. Thus, if we set k= 4IB , cI (246)

we can extract numerical values of B from the potential distribution on the paper. To be more precise, note that = E dl = 1 k (B z) dl = 1 k B (dl z) = B , k (247)

where B is the magnetic ux per unit length in z that passes between the end points of the integration. As to the boundary conditions, rst consider the patches of conducting paint. The electric eld is perpendicular to the boundaries of these, and hence the model B is parallel to them. This is the magnetic boundary condition at a perfect conductor. As mentioned in the statement of the problem, one can use a patch of conducting paint to simulate a surface on which the magnetic eld is known to lie, thereby reducing the extent of the model. The electric eld is parallel to the edge of the conducting paper, and zero outside it, since E = J/ . Hence, the model magnetic eld is perpendicular to the edge of the paper. Outside the paper, (244) and the vanishing of E would imply that B vanishes also. This awkwardness can be avoided by supposing that we were actually modeling the eld H rather than B, and the paper corresponds to a region of permeability = 1, where B = H. Then, if we suppose that the region outside the paper has very high permeability, the continuity of B at the boundary implies that H is extremely

Princeton University 1999

Ph501 Set 4, Solution 14

52

small outside the paper, which restores consistency of the actual electrical boundary conditions with a class of magnetic boundary conditions. Thus, we conclude that the conducting paper technique is particularly well suited for mapping 2-dimensional magnetic elds in situations bounded by a high permeability material, such as iron. Of course, the actual elds must not be so high that the iron saturates and the eective permeability drops to near unity.

Princeton University

Ph501 Electrodynamics Problem Set 5


Kirk T. McDonald (1999) kirkmcd@princeton.edu http://puhep1.princeton.edu/~mcdonald/examples/

Princeton University 1999

Ph501 Set 5, Problem 1

1. a) A charged particle moves in a plane perpendicular to a uniform magnetic eld B. Show that if B changes slowly with time, the magnetic moment produced by the orbital motion of the charge remains constant. Show also that the magnetic ux through the orbit, = r2 B is constant. These results are sometimes given the fancy name of adiabatic invariants of the motion. b) The Magnetic Mirror. Suppose instead, that the magnetic eld is slightly nonuniform such that Bz increases with z . Then, if the charged particle has a small velocity in the z direction, it slowly moves into a stronger eld. Again, we would expect the ux through the orbit to remain constant, which means that the orbital radius must decrease and the orbital velocity must increase. However, magnetic elds which are constant in time cannot change the magnitude of the velocity, therefore vz must decrease. If Bz increases enough, vz will go to zero, and the particle is trapped by the magnetic eld. Write 2 2 2 v 2 = vz + v = v0 , (1) where v is the orbital velocity and v0 is constant. Use the result of part a) to show that Bz (z ) 2 2 2 vz (z ) v 0 v (0) . (2) Bz (0)

Princeton University 1999

Ph501 Set 5, Problem 2

2. If one pitches a penny into a large magnet, eddy currents are induced in the penny, and their interaction with the magnetic eld results in a repulsive force, according to Lenz law. Estimate the minimum velocity needed for a penny to enter a long, 1-T solenoid magnet whose diameter is 10 cm. You may suppose that the penny moves so that its axis always coincides with that of the magnet, and that gravity may be ignored. The speed of the penny is low enough that the magnetic eld caused by the eddy currents may be neglected compared to that of the solenoid. Equivalently, you may assume that the magnetic diusion time is small.

Princeton University 1999

Ph501 Set 5, Problem 3

3. a) Diamagnetism. We consider a model of an atom in which the distance r from the electron to the nucleus is somehow xed, but the electron is free to orbit the nucleus. Then, if a eld B is applied to the atom, an E.M.F. is induced around the orbit, while B is changing, which generates a magnetic dipole moment m via the resulting motion of the electron. Show that e2 r 2 B, (3) m= 4mc2 where e and m are the charge and mass of an electron, respectively. In bulk matter, with n atoms per unit volume, the magnetization M is then M = nm. The magnetic susceptibility is dened by M = M H. (4)

Since B = H, and also B = H + 4 M = (1 + 4M )H, we see that the diamagnetic permeability obeys < 1. Calculate M = ( 1)/4 for hydrogen gas at S.T.P. and compare with the measured value of 2.24 109 . b) In materials where B = H, we claim that the magnetic energy is Umag = 1 8 B H dVol = 1 8 B 2 dVol 1 2 B M dVol . (5)

Use your analysis from part a) to show that the last term is just the kinetic energy of the electrons motion induced by the eld B.

Princeton University 1999

Ph501 Set 5, Problem 4

4. a) A ip coil is a practical device for measuring magnetic elds. A coil whose axis is the z -axis is ipped by 180 about the x-axis. The coil leads are connected to a charge integrator. Show that charge 2 (6) Q= R is collected in the ip, where is the magnetic ux through the loop before (and after) ipping and R is the resistance of the integrator (plus coil). b) A fancy ip coil is made by winding wire on the surface of a sphere such that the turns are distributed according to dN sin d . (7)

(Recall prob. of set 4.) All turns are parallel to the x-y plane. For this coil, show that Bz dVol, (8)

the integration being over the interior of the sphere. c) The eld component Bz (r, , ) obeys 2 Bz = 0 inside the sphere, and so may be expanded in a series of Legendre functions. However, Bz is not necessarily azimuthally symmetric, so a slight generalization must be made: Bz =
m,n m Am,n rn Pn (cos )eim ,

(9)

m are the associated Legendre polynomials. where n and m are integers, and the Pn 0 Note that Pn (cos ) = Pn (cos ), the ordinary Legendre polynomials. Using this, show that Bz (0, 0, 0), so that the sin ip coil measures Bz at the center of the sphere, no matter how B varies over the sphere!

d) A sin coil is hard to build. Suppose we try to make do with a simple cylindrical coil of radius a and height h. Show that if h = 3a, all eects of the rst, second and third derivatives of the eld vanish. With such a coil, accuracies of 1 in 104 may be achieved. Hint: Expand Bz in rectangular coordinates and note that B = 0, B = 0 and hence 2 B = 0.

Princeton University 1999

Ph501 Set 5, Problem 5

5. A cylinder of dielectric constant rotates with constant angular velocity about its axis. A uniform magnetic eld B is parallel to the axis, in the same sense as . Find the resulting dielectric polarization in the cylinder and the surface and volume charge densities, neglecting terms of order (a/c)2 , where a is the radius of the cylinder. Answer: 1 B r 4c where r is the radial vector out from the axis. P= (10)

This problem can be conveniently analyzed by starting in the rotating frame. Consider also the electric displacement D.

Princeton University 1999

Ph501 Set 5, Problem 6

6. a) Show that the self- and mutual inductances of two circuits obey L11L22 L2 12 by considering the magnetic energy 1 1 2 2 U = L11I1 + L22I2 + L12I1I2 . 2 2 (12) (11)

b) A toroidal coil of N turns has a circular cross-section of radius a; the central radius of the coil is b > a. Show that the self-inductance is L11 = 8N 2 ( b b2 a2) sin1 c2 1 1+
3b2 4 a2

(13)

c) A second circuit in the form of a single loop of radius > a links the toroid; the plane of the second circuit is the same as that of one of the turns of the toroid, and that turn is entirely inside the new circuit. Calculate the mutual inductance L12 between the toroid and the new circuit, and show that relation (11) is obeyed in this example.

Princeton University 1999

Ph501 Set 5, Problem 7

7. a) A coaxial cable consists of a center wire of radius a surrounded by a thin conducting sheath of radius b > a. The region a < r < b is vacuum. Consider a circuit formed by joining the two conductors at to show that the self inductance per unit length is L= 2 c2 1 b + ln 4 a . (14)

Assume the current is distributed uniformly within the center wire. b) Suppose the axis of the sheath is a distance from the axis of the center wire. Calculate the self inductance accurate to terms in ( /b)2 .

Princeton University 1999

Ph501 Set 5, Problem 8

8. a) A long cylinder of radius a has uniform magnetization M perpendicular to its axis. Find the magnetic elds B and H everywhere. the direction of the magnetization. Let z be the axis of the cylinder and x , along its axis. Find the b) Suppose the cylinder is given a uniform velocity, v = v z resulting charge density and electric eld everywhere. You may ignore eects of order (v/c)2. You can check your result by noting that the Lorentz force on a charge at rest with respect to the cylinder should vanish.

Princeton University 1999

Ph501 Set 5, Problem 9

9. An iron ring has a circular cross section of radius a, and average radius b a. However, the ring has a narrow gap from azimuth = 0 to h/b 1; the gap width is w. A toroidal winding of N turns wraps around the ring. Calculate the stored magnetic energy as a function of the current I in the windings and the gap width w in a regime where the permeability of the iron is very large. Calculate the force needed to keep the gap from closing. Suppose the eld in the gap were 15,000 Gauss, near the maximum that is readily achieved in an iron core magnet. Express the force/area that tends to close the gap in terms of atmospheric pressure.

Princeton University 1999

Ph501 Set 5, Problem 10

10

10. Discuss the surface charges and ow of eld energy in a cylindrical wire of radius a of conductivity that carries current I distributed uniformly within the wire. For deniteness, assume the current returns in a hollow conducting cylinder of inner radius b and very large outer radius. Then, the current density J and electric eld E are vanishingly small in the outer conductor, whose constant electrical potential may be taken as zero. Steps in the discussion: Find the magnetic eld B everywhere. Find the electric potential (r, , z ) and electric eld E rst for r < a, and then for a < r < b. Dene (0, 0, 0) = 0 at the center of the wire. Answer: (a < r < b ) = Iz ln(r/b) . a2 ln(a/b) (15)

Find the surface charge density at r = a which is needed to shape the electric eld inside the wire to be along z. When the current rst begins to ow, the electric eld is not yet uniform and free charge heads for the surface of the wire until the desired static surface charge distribution is obtained. A length l of the wire has resistance R = l/a2 and consumes power at the rate I 2R. Show that the Poynting vector S = (c/4 )E B at the surface of the wire provides this power. Thus, according to Poynting, the power ows down the air gap and into the side of the wire. As Sommerfeld says, Electromagnetic energy is transported without losses only in nonconductors. Conductors are nonconductors of energy, which is dissipated in Joule heating. An alternative calculation of the surface charge density may be instructive. Consider rst the question of how a tube of radius a of uniform axial electric eld could be created in the absence of the wire. A capacitor consisting of a pair of circular plates of radius a has a very nonuniform eld between the plates as their separation becomes large. We want the equipotentials to be perpendicular to the axis, and uniformly spaced, which could be approximately achieved by adding a set of conduting rings of radius a, spaced uniformly along the axis with potentials that vary linearly between the two end plates. The charge on a ring would be given by Q = CV , where C is he capacitance of a ring, and V is the desired potential of the ring. The current-carrying wire is a kind of continuum limit of the above procedure. The desired potential inside the wire is (z ) = IRz . For the coaxial geometry of the present problem, calculate the capacitance per unit length between the wire of radius a and the return conductor of radius b. Then calculate the charge per unit length, and the surface charge density, on the wire via Q(z ) = CV (z ).

Princeton University 1999

Ph501 Set 5, Problem11

11

11. Consider an air-core transformer in the form of two coaxial cylinders of length l and radii r1 < r2 l. Each cylinder is wrapped with Ni turns, and the total resistance of coil i is Ri . a) Deduce the currents I1(t) and I2(t) in the coils when the primary coil 1 is driven by voltage V1 (t) = V0 cos t. First, evaluate the self and mutual inductances, L1 , L2 and M , and then solve the coupled circuit equations. Calculate the time-average power dissipated in coil 2. b) Evaluate the Poynting vector S to show that its time average is nonvanishing only for r1 < r < r2 , and that the total Poynting ux 2rl Sr is just the power dissipated in coil 2. What is the direction of S? c) Consider coil 2 as the primary driven by voltage V2 (t) = V0 cos t, and discuss the relation between the Poynting vector and the power dissipated in coil 1.

Princeton University 1999

Ph501 Set 5, Problem 12

12

12. Feynman Disk Paradox. Consider a small coil centered on the origin that carries a current which sets up a magnetic dipole moment m = m z. A ring of radius a in the plane z = 0 has charge Q distributed uniformly on it. The ring is rigidly attached to the coil, but the assembly is free to rotate about the z axis. a) Calculate the initial angular momentum LEM in the electromagnetic eld. Use the multipole expansion for the potential of a ring of charge, pp. 58-59, to show that 2mQ/15ca, r < a, LEM,z = (16) 13mQ/15ca, r > a. b) Now let the current in the coil decrease to zero. Calculate the eld induced at the ring, and the resulting torque to show that Lmech,z = once the moment m has vanished. Hint: Since magnetic eld lines always form loops, the ux through the ring is equal and opposite to that across the plane z = 0 outside the ring. For yet another version of this problem, see http://puhep1.princeton.edu/~mcdonald/examples/feynman_cylinder.pdf mQ , ca (17)

Princeton University 1999

Ph501 Set 5, Problem 13

13

13. Consider particle with charge e and momentum P = Pz + P (P = 0) that is moving on average in the z direction inside a solenoid magnet whose symmetry axis is the z axis and whose magnetic eld strength is Bz . Inside the solenoid, the particles trajectory is a helix of radius R, whose center is at distance R0 from the magnet axis. The longitudinal momentum Pz is so large that when the particle reaches the end of the solenoid coil, it exits the eld with little change in its transverse coordinates. This behavior is far from the adiabatic limit (c.f. Prob. 1) in which the trajectory spirals around a eld line. When the particle exits the solenoid, the radial component of the magnetic fringe eld exerts azimuthal forces on the particle, and, in general, leaves it with a nonzero azimuthal momentum, P . Deduce a condition on the motion of the particle when within the solenoid, i.e., on R, R0 , Pz , P , and Bz , such that the azimuthal momentum vanishes as the particle leaves the magnetic eld region. Your result should be independent of the azimuthal phase of the trajectory when it reaches the end of the solenoid coil. Hint: Consider the canonical momentum and/or angular momentum.

Princeton University 1999

Ph501 Set 5, Solution 1 Solutions

14

1. a) Since the particle moves in the plane perpendicular to the magnetic eld , the velocity v, the eld B and the force F on the particle of mass m and charge q are mutually orthogonal. The orbit is a circle of radius r related by F = qvB mv 2 = , r c (18)

so long as B varies suciently slowly. Then, r= mcv qB (19)

and the magnetic moment due to this orbit is of magnitude = r2 qv qrv mv 2 r2 I = = = . c c 2r 2c 2B (20)

The vector is in the opposite direction to B, which can be considered as an example of Lenz Law. If the eld B varies with time, then an electric eld is induced around the particles orbit as given by Faradays Law: E dl = 2rE = Thus, rB mv B = , (22) 2c 2qB with E in the same direction as v. That is, if the magnetic eld increases, the electric eld causes the particle to accelerate, E= v = The solution to this is v vB qE = . m 2B B, (23) 1d c dt B dS = r2 B . c (21)

(24)

so that v 2/B is constant, and hence the magnetic moment (20) is constant. The ux linked by the orbit, m2c2 v 2 = r2 B = , (25) q 2B is also constant. b) Suppose now that the magnetic eld is constant in time, but varies in space. For example, consider a eld that has azimuthal symmetry about the z axis, and Bz increasing with z . A charged particle with nonzero vz , moves along a kind of helix in this eld.

Princeton University 1999

Ph501 Set 5, Solution 1

15

The magnetic eld at the z coordinate of the particle then varies as dBz dBz (z (t)) = vz . dt dz (26)

If this change is slow, the analysis of part a) holds, and the particles motion varies so 2 (z )/Bz (z ) constant, so that as to keep v
2 2 v (z ) v (0)

Bz (z ) . Bz (0)

(27)

In writing this, we recall from prob. 6, set 4 that for magnetic elds with azimuthal symmetry, Bz (r, z ) Bz (0, z ) r2Bz (0, z )/4 + ..., and we ignore the radial dependence for orbits with small r.
2 2 + v remains constant as well, so we have Of course, v 2 = vz 2 2 2 vz (z ) = v 0 v (0)

Bz (z ) . Bz (0)

(28)

The particle stops moving forward in z at the plane where Bz (z ) =


2 v0 Bz (0) > B (0) . 2 v (0)

(29)

Although the particle has vz = 0 at this plane, its v now equals v (0), so still there is a large Lorentz force in the z direction, and the particle spirals its way back down the z axis. Hence the term magnetic mirror. A eld conguration in which the axial eld strength increases with |z | can trap charged particles near the origin. This is no contradiction to Earnshaws theorem, as a magnetic bottle has no static equilibrium point, but relies on electrodynamics to trap particles with constant, nonzero velocity.

Princeton University 1999

Ph501 Set 5, Solution 2

16

2. The penny has radius a and thickness z . For the motion as stated in the problem, the eddy current will ow in concentric rings about the center of the disk. Therefore, we rst examine a ring of radius r and radial extent r. The magnetic ux through the ring at position z is r2 Bz (0, z ), whose time rate of change is = r2 B z = r2 B v, z (31) (30)

where indicates dierentiation with respect to time, is dierentiation with respect to z , Bz stands for Bz (0, z ), and v is the velocity of the center of mass of the ring. The penny has electrical conductivity . Its resistance to currents around the ring is R= 2r , r z (32)

so the (absolute value of the) induced current is I= using Faradays law. The azimuthal eddy current interacts with the radial component of the magnetic eld to produce the axial retarding force. Close to the magnetic axis, we estimate the radial eld in term of the axial eld according to Br (r, z ) r Br (0, z ) r Bz (0, z ) rB = z, r 2 z 2 (34) rBz v rz E = = , R cR 2c (33)

as can be deduced from the Maxwell equation B = 0, noting that on the magnetic axis Br /r = Bx /x = By /y . Then, the retarding force on the ring is Fz = r2 Br Bz v rz 2rBr I r3 (Bz )2 v rz = . c c2 2c2 (35)

Alternatively, we note that the kinetic energy lost by the penny appears as Joule heating. Hence, for the ring analyzed above, v Fz = dU r3 (Bz )2 v 2rz , = I 2 R = dt 2c2 (36)

using eqs. (32) and (33), which agains leads to eq. (35). The equation of motion of the ring is dFz = r3 (Bz )2 v rz = mv = 2rrz v v, 2c2 (37)

Princeton University 1999

Ph501 Set 5, Solution 2

17

where is the mass density of the metal. We integrate this equation with respect to radius to nd a4(Bz )2 v z = a2z v v, (38) 8c2 After dividing out the common factor a2 z v , we nd v = a2 (Bz )2 . 8c2 (39)

For an estimate, we note that the peak gradient of the axial eld of a solenoid of diameter D is about B0 /D, and the gradient is signicant over a region z D. Hence, on entering a solenoid the jet velocity is reduced by v
2 a2B0 . 8c2 D

(40)

The penny must have initial velocity v0 > v to enter the magnet. A copper penny has a 1 cm, density 10 g/cm3 , electrical resistivity 106 cm, and therefore conductivity 9 1017 Gaussian units. The minimum velocity to enter a 1-T = 104 -G magnet with diameter D = 10 cm is then, vmin 9 1017 (1)2 (104 )2 125 cm/s. 8 (3 1010 )2 10 10 (41)

The case of a sphere rather than a disk has been presented in J. Walker and W.H. Wells, Drag Force on a Conducting Spherical Drop in a Nonuniform Magnetic Field, ORNL/TN6976 (Sept. 1979).

Princeton University 1999

Ph501 Set 5, Solution 3

18

3. a) Assume that the electron of an atom is initially stationary and that the vector r between the electron and the nucleus is at a right angle to the direction of the increasing magnetic eld B = B z. As the magnetic eld is applied, an electric eld is induced around a loop of radius r according to E= rB , 2c (42)

which, as Lenz law decrees, will accelerate the electron to velocity v , given by v= v dt = eE dt = m erB erB dt = , 2mc 2mc (43)

so that its magnetic dipole moment opposes the magnetic eld. From the next to last equality in (20), we have m= evr r0 r 2 B e2 r 2 B z= z, z = 2c 4mc2 4 (44)

where r0 = e2/mc2 = 2.8 1013 cm is the classical electron radius. With n atoms per unit volume, the magnetization is M = nm = The magnetic susceptibility M is related by M M H = so M B M B , 1 + 4M (46) nr0 r2 B. 4 (45)

nr0r2 . (47) 4 Hence, the permeability, = 1 + 4M is less than one. For hydrogen, r is the Bohr radius, a0 = r0 /2 , and, at S.T.P., n 5.4 1019 /cm3 , so M 1.1 1010 , which is of the same order of magnitude as the stated value of 2.24 109 . M b) In a volume V , there are N = nV electrons, and their kinetic energy is nV e2r2 B 2 V 1 T = N mv 2 = = M B, 2 8mc2 2 (48)

using (43) and (45), which is just the second term in the expression (5) for the magnetic energy.

Princeton University 1999

Ph501 Set 5, Solution 4

19

4. a) As the coil ips, the total amount of ux cut by the coil is 2. Then, since the E.M.F. E generated is proportional to the rate of change of ux, the charge integrated over time is Q= I dt = 1 R E dt = 1 cR 1 d dt = dt cR d = 2 . cR (49)

b) The total magnetic ux through the turns, which are perpendicular to the z axis, is = B ( z dx dy )dN. (50)

Since the density of turns obeys dN sin d, and dz = a sin d , where a the radius of the sphere, we have dN dz . Hence, (50) becomes Bz dxdydz = Bz dVol. (51)

c) Inserting the Legendre series (9) into (51), we have


m,n m Am,n rn Pn (cos )eim r2 dr d cos d.

(52)

The integral over the azimuthal angle is


2 0

eim d = 2 m0 .

(53)

The integral over the polar angle is then


1 1 1

Pn (cos ) d cos =

Pn (cos )P0 (cos ) d cos = 2 n0 . a3 . 3

(54)

The radial integral is just


a 0

r2 dr =

(55)

Combining (52-55) then gives 4a3 A0,0 = Bz (0, 0, 0)Vol. 3 (56)

d) The total ux linked by the coil is again given by (50), where now the density of windings is dN = ndz . Thus, = n Bz dVol. (57) In a Taylor expansion of Bz in rectangular coordinates about the center of the coil, the integral of odd-order terms will vanish because the cylinder is symmetrical under reections. Hence, up to third order, the only terms which survive are the zeroth-order term, a2 hBZ (0, 0, 0), and the second-order term, 1 2 d2 Bz dx2 i x2 i dVol.
0

(58)

Princeton University 1999

Ph501 Set 5, Solution 4

20

In current-free regions and static situations, 2 Bz =


i

d2 Bz =0, dx2 i

(59)

so (58) will vanish if the three integrals in the sum are equal. The x1 = x and x2 = y integrals are automatically equal because of the symmetry of the cylinder, which means that for the term to vanish, we need z 2 dVol = 1 2 (x2 + y 2) dVol = 1 2 r2 dVol (60) (61)

Hence, we require that h =

a4h a2h3 = . 12 4 3a.

Princeton University 1999

Ph501 Set 5, Solution 5

21

5. The v B force on an atom in the rotating cylinder is radially outwards, and increasing linearly with radius, so we expect a positive radial polarization. We begin our analysis in the rotating frame, in which any polarization charge density is at rest and causes no additional magnetic eld. Then, P = E , where E and P are the electric eld and dielectric polarization in the rotating frame. If v = r c, then the electric eld in the rotating frame is related to lab frame quantities by E =E+ v B, c (62)

where E is the electric eld due to the polarization that we have yet to nd. Since polarization is charge times distance, in the nonrelativistic limit the polarization is the same in the lab frame and the rotating frame: P = P. The velocity has magnitude v = r, and is in the azimuthal direction. Thus, v B = B r, so that B r . (63) P= E+ c There are no free charges, so the electric displacement is zero: D = 0 = E + 4 P. Thus, E = 4 P. Recalling that = ( 1)/4 , (63) leads to P= The surface charge density is pol = P(a) r= 1 Ba, 4c (66) 1 B r. 4c (65) (64)

where a is the radius of the cylinder. As well as this surface charge density, there is a volume charge density, pol = P = 1 1 rPr = B, r r 2c (67)

so that the cylinder remains neutral over all. Both the surface and volume charge densities are proportional to v (r)/c, and are moving at velocity v (r). Hence, the magnetic eld created by these charges is of order v 2/c2 , and we neglect it in this analysis. This example is perhaps noteworthy in that a nonvanishing, static volume charge density arises in a charge-free, linear dielectric material. In pure electrostatics this cannot happen, since P = E together with D = 0 = E + 4 P imply that pol = P = 0.

Princeton University 1999

Ph501 Set 5, Solution 5

22

We also oer an iterative solution. The axial magnetic eld acts on the rotating molecules to cause a v B force radially outwards. This can be described by an eective electric eld B r. (68) E0 = c This eld causes polarization B r. c Associated with this is the uniform volume charge density P0 = E0 = 0 = P0 = 2B. According to Gauss Law, this charge density sets up a radial electric eld E1 = 20r = 4B r. At the next iteration, the total polarization is B r. (72) c This causes additional charge density 2 , which leads to additional electric eld E2, ... P1 = (E0 + E1 ) = (1 4) At the nth iteration, the polarization will have the form Pn = kn Then, n = Pn = 2kn B, and En+1 = 2n r = 4kn B r. (75) The eective electric eld at iteration n + 1 is the sum of E0 due to the v B force and En+1 due to the polarization charge. Thus, Pn+1 = (E0 + En+1 ) = (1 kn ) But by denition, Pn+1 = kn+1 Hence, kn+1 = (1 4kn ). If this series converges to the value k , then we must have k = (1 4k ), so that 1 = , 1 + 4 4 which again gives (10) for the polarization. k= (79) (80) (78) B r. c (77) B r. c (76) (74) B r. c (73) (71) (69)

(70)

Princeton University 1999

Ph501 Set 5, Solution 6

23

6. a) In the expression (12) for the total magnetic energy in the circuits, let I1 = The total energy is then L2 1 2 L22 12 I2 . U= 2 L11 (82) L12 I2 . L11 (81)

Since the magnetic energy U is also given by B 2 dvol/8 , it must be non-negative. Hence, the factor in parentheses in (82) must be non-negative, and L11 L22 L2 12 . (83)

b) The magnetic eld due to current I in a toroid of N windings is azimuthal, and is conned to the interior. Amp` eres Law gives the magnitude as B (r ) = 2NI , cr (84)

where r is the perpendicular distance from the axis. The self inductance L11 is related by N 1 /cI , where 1 is the ux linked by one turn. Thus, for a toroid of central radius b whose cross section is a circle of radius a, 2N 2 a 2dx a2 x2 . (85) L11 = 2 c x+b a Substituting y = x + b, L11 4N 2 = c2 4N 2 = c2
b+a ba

dy y 2 + 2by + a2 b2 . y

2(b y ) y 2 + 2by + a2 b2 b sin1 2 4a + 3b2 b+a 2by + 2a2 2b2 b2 a2 sin1 2 4a + 3b2 ba 2 8N 1 = (b b2 a2) sin1 . 2 2 c 1 + 3b 2
4a

(86)

c) To calculate the mutual inductance between the two circuits, we note that the second loop links all the ux of the toroidal eld, which we called 1 above. Hence, L12 = 1 L11 = . cI N (87)

If the second circuit has radius R, and is made of a wire of radius r0, then its self inductance is 4R 8R 7 L22 = 2 ln , (88) c r0 4

Princeton University 1999

Ph501 Set 5, Solution 6

24

from p. 115b of the Notes. Hence, for the system of loop plus toroid,
R R ln 8 7 N 2 L22 L11L22 r0 4 = = 1 2 2 L2 L 2(b b a ) sin 11 12

1 1+ 3b 2
4a 2

(89)

The numerator is smallest when R = a, the minimum for which the second loop fully links the toroid. The denominator is largest when b = a and the toroid looks like a donut whose hole has shrunk to zero. Then, L11 L22 L2 12 =
min a ln 8 r0 7 4 4 7

2 sin1

(90)

This expression equals unity when a = 1.06r0 , i.e., when the second loop is also essentially a donut with no hole. However, the expression (88) for the self inductance of a loop was deduced supposing that R r0. Since the general restriction (11) is satised using (88) for any R > 1.06r0 , we infer that (88) is still reasonably accurate for R only a few times r0 .

Princeton University 1999

Ph501 Set 5, Solution 7

25

7. a) In cylindrical coordinates (r, , z ), the magnetic eld is azimuthal in a coaxial cable whose axis is the z axis. When current I ows in the cable, whose solid inner conductor has radius a and whose outer conductor is a cylindrical shell of radius b, the eld strength follows from Amp` eres law as
2 2Ir/a c,

B (r) =

2I/cr, 0,

r a, a r b, r > b.

(91)

The energy per unit length along the cable of this magnetic eld is U= 1 8 B 2 dArea = I2 2c2
a 0

r2 2r dr + a4

b a

2r dr r2

I2 c2

b 1 + ln 4 a

(92)

Since the energy can be expressed in terms of the self inductance L as U = 1 LI 2, we 2 obtain the result (14). Alternatively, we can evaluate the self inductance as L = /cI , where is the magnetic ux per unit length linked by the circuit. The ux linked for a < r < b is clearly (a < r < b) =
b a

B dr =

2I c

b a

2I b dr = ln . r c a

(93)

More care is required when discussing the region r < a. On p. 115a of the Notes we saw that a consistent procedure for an extended current distribution is to average the ux linked by the various lamentary currents. In the present case, consider rst a lament of area r dr d at (r , ). We can dene the surface through which the ux is to be calculated as that portion of the shell of radius r that connects (r , ) with the point (r , 0), plus the plane = 0 between r and a. Since the eld is azimuthal, no ux is linked on the shell; all laments on the same shell link the same ux. Thus, (r < a) = 2I 1 c a2
a 0

2r dr
r

dr

r 2I = a2 a2 c

a 0

r dr

r2 a2

2I 4c

(94)

Combining (93-94) and dividing by cI , we again arrive at (14). b) It appears impossible to make an accurate estimate of the self inductance when the outer cylinder is o center by either of the methods used in part a). The reason is that the currents are no longer uniformly distributed over the surfaces of the cylinders, so it is hard to calculate the magnetic eld properly. A solution can be given for the closely related problem in which the inner conductor, as well as the outer conductor, is a cylindrical shell. Then, we know from transmission line analysis (Lecture 13) that LC = 1/c2 , where C is the capacitance per unit length. With some eort we then nd that L=
2 2 b ln , c2 a b2 a 2

(95)

to O( 2 /b2 ). See my note An O-Center Coaxial Cable (Nov. 21, 1999). http://puhep1.princeton.edu/~mcdonald/examples/coax.pdf

Princeton University 1999

Ph501 Set 5, Solution 7

26

Here we illustrate what happens if we follow the approachs of part a), assuming the currents are uniformly distributed over the two cylinders. If the center of the outer cylinder is at (r, ) = ( , 0), then the surface of that cylinder follows b2 = r2 + 2 2 r cos , (96) or r() = cos + b2
2

sin2 b + cos

2b

sin2 .

(97)

We rst calculate the self inductance via the energy method. Inside the outer cylinder the magnetic eld is still given by the rst two lines of eq. (91), but with r = b replaced by r() from eq. (97). Outside the cylinder the eld is not quite zero because the magnetic eld vectors from the currents in the inner and outer cylinders have slightly dierent magnitudes and directions. The vector from the center of the outer cylinder, ( , 0) to a point (r, ) has magnitude r r cos , and makes angle ( /r) sin to r. Hence, the magnetic eld from the current in the outer cylinder is B 2I cr sin cos , , 1 r r (98)

and the total magnetic eld outside the outer cylinder is Boutside so its magnitude is Boutside 2I /cr2 . The magnetic eld energy per unit length along the axis is now U =
a r2 2 r () r dr 2 1 I2 B 2 dArea = 2 r dr + d + d 8 2c2 0 a4 r2 0 a 0 2 2 2 2 d I2 b 2 d ln 1 + sin + + cos 2c2 2 a b 2b2 2 0 b2 0 2 2 2 I2 b 2 2 2 + + cos d ln sin cos + 2c2 2 a b 2b2 2b2 b2 0 b I2 1 1 + ln = 2 = LI 2. c 4 a 2 r ( ) 2

2I (sin , cos ), cr2

(99)

r dr r4

(100)

Hence, we would conclude from the energy method that there is no change in the inductance to second order. We contrast this with a calculation of the ux linked by the o-center coax. The contribution for r < a is again given by (94). For r > a but inside the o-center outer cylinder, the magnetic eld is still B = (0, 2I/cr). The ux through the region

Princeton University 1999

Ph501 Set 5, Solution 7

27

r() > r > a varies with azimuth, so we average over laments on the outer cylinder: (r() > r > a) =
2 r () dr 2I 1 2 2I 2 b = d d ln 1 + cos 2 sin2 c 2 0 r 2c 0 a b 2b a 2 2 2I 2 b d ln + cos 2 sin2 2 cos2 2c 0 a b 2b 2b 2 2I b = ln 2 . (101) c a 2b

Combining (94) and (101), we nd that the self inductance is now L= to O( 2 /b2 ). Comparing with the result (95), we infer that the calculation via the linked ux is more accurate than that via the energy method when we use the incorrect assumption of uniform current distributions. 2 c2
2 1 b + ln 2 , 4 a 2b

(102)

Princeton University 1999

Ph501 Set 5, Solution 8

28

8. a) Since there are no free currents in the problem, H = 0 and we can dene a magnetic scalar potential such that H = . As the cylinder is very long, we approximate the problem as 2-dimensional: = (r, ) in cylindrical coordinates (r, , z ). The source of the magnetic scalar potential is the imagined magnetic charges associated , the volume charge density M = M = 0. with the magnetization. Since M = M x However, at the surface of the cylinder at r = a, there is a density given by M = M r = M cos . The potential is continuous at the boundary r = a, and Gauss law tells us that 4M = 4M cos = Hr (r = a+ ) Hr (r = a ) = (r = a+ ) (r = a ) + . (104) r r (103)

The potential can be expanded as a harmonic series, but only the term in cos will contribute in view of (104). Thus, = Hr cos , r a, 2 H ar cos , r a, (105)

satises continuity of the potential at r = a. Then, (104) also tells us that H = 2M . Inside the cylinder we have (r < a) = 2Mx, = 2M, H(r < a) = 2M x B(r < a) = H + 4M = 2 M. Outside the cylinder there is no magnetization, and (r > a) = 2Ma2 H (r > a ) = B (r > a ) = cos , r (109) (110) (106) (107) (108)

2Ma2 ). (cos r + sin r2

b) In case of a moving cylinder, the analysis of part a) holds in the rest frame of the cylinder. When the cylinder has velocity v = v z in the lab frame, there appears to be an electric eld in the lab frame related by E = v v B B, c c (111)

where we ignore terms of order v 2/c2 , so the magnetic eld B in the lab frame is the same as the eld B given by (108) and (110) in the rest frame. Regarding the sign in (111), we note that a charge which is at rest in the lab frame is moving with velocity v in the rest frame of the magnetized cylinder, and so in the latter frame experiences a Lorentz force v/c B.

Princeton University 1999 Thus,

Ph501 Set 5, Solution 8

29

v v ), = 2M (sin E(r < a) = 2M y r + cos c c 2Mva2 ). (sin r cos E (r > a ) = cr2 There is an electric charge density on the surface of the cylinder given by = 1 Mv sin . Er (r = a+ ) Er (r = a ) = 4 c

(112) (113)

(114)

This can be thought of as arising from a polarization P related to the moving magnetization by v (115) P = M. c See sec. 87 of Becker for a discussion of how M and P form a relativistic tensor.

Princeton University 1999

Ph501 Set 5, Solution 9

30

9. The magnetic induction B is related to the magnetic eld H by B = H, where is the permeability. In the gap, = 1. The normal component of the magnetic induction is continuous across the boundaries of the gap, since B = 0. Thus, Hgap = Bgap = Biron = Hiron. For a large permeability iron, the magnetic eld Hiron is negligible. The magnetic eld H at the center of the toroid is related by Amp eres law as H dl = Hgapw + Hiron (2b w) = With the neglect of the small quantity Hiron , we nd Hgap = Bgap = Biron The magnetic energy is U= 1 8 B H dVol a2w 8 4NI cw
2

(116)

4NI . c

(117)

4NI . cw

(118)

2 2 a2N 2 I 2 . c2 w

(119)

The force tending to close the gap is F = 2 2a2 N 2 I 2 dU = . dw c2 w 2 (120)

The pressure can also be calculated via the Maxwell stress tensor as Pgap = If Bgap = 15, 000 Gauss, then Pgap = 9 106 dyne/cm2 = 9 atmospheres. (122)
2 Bgap . 8

(121)

Princeton University 1999

Ph501 Set 5, Solution 10

31

10. The current density associated with a uniform current I in a wire of radius a whose axis is the z axis is I . J = 2z (123) a Ohms law gives the electric eld inside the wire as E= I J , = 2 z = IRz a (124)

where is the conductivity, and R = 1/a2 is the resistance per unit length of the wire. The electric potential inside the wire is therefore, (r < a) = IRz, where we dene (0, 0, 0) = 0. For the region a < r < b, we suppose the potential satises separation of variables: (a < r < b ) = f (r )g (z ). Continuity of the potential at r = a is satised by the form (a < r < b) = f (r)IRz. Substituting (127) into Laplaces equation, 2 = 0, we nd that 1 d df r = 0, r dr dr so f has the general solution f = A + B ln r. (129) The boundary conditions on the potential at r = a and b now require that f (a) = 1 and f (b) = 0. Hence, f = ln(r/b)/ ln(a/b), and (a < r < b) = IRz ln(r/b) ln(r/b) = IRz . ln(a/b) ln(b/a) (130) (128) (127) (126) (125)

The surface charge density q at the surface of the wire is q = 1 1 (r = a+ ) (r = a ) Er (r = a+ ) Er (r = a ) = + 4 4 r r IRz . (131) = 4a ln(b/a)
, IRz

The electric eld is E = = r < a, IRz r/r ln(b/a) + IR ln(r/b) z/ ln(a/b), a < r < b, 0, b < r. (132)

Princeton University 1999

Ph501 Set 5, Solution 10

32

Figure 1: The solid curves show lines of Poynting ux S, and the dashed lines are the electric eld E in the region between the wire and the outer conductor. Because the tangential component of E is continuous at the boundary r = a, for r < a, the eld lines for r > a are bent towards positive and E = IRz z . For |z | < b the eld lines leave positive surface charges at r = a and end on negative surface charges also at r = a; in loop circuits (b > L) this is the general behavior. From Electrodynamics by A. Sommerfeld (Academic Press, 1952), p. 129.

The magnetic eld follows from Amp` eres law: B (r) = The Poynting vector is then, r < a, c 2 2 2 S= E B = I R ln(b/r) r/2r ln(a/b) I Rz z/2r ln(b/a), a < r < b, 4 0, b < r. (134) The Poynting vector is radially inwards at the surface of the wire, and the energy ux per unit length there is 2aS (r = a) = I 2R. That is, the Poynting ux energy the wire through its surface provides the I 2R power loss to Joule heating. The Poynting ux crossing a plane at constant z is Sz dArea = I 2 Rz 2 ln(b/a)
b a

2Ir/a2 c, r a, 2I/cr, a r,

(133)

2 r/2a2, I Rr

2r dr = I 2Rz. r2

(135)

Since the ux is zero at z = 0, we interpret (135) as indicating that the total Poynting ux crossing a plane at constant z equals the power dissipated by the wire between 0 and z . This ux exists in the region a < r < b, i.e., in the air (or vacuum) between the conductors, rather than in the conductors themselves.

Princeton University 1999

Ph501 Set 5, Solution 10

33

For the alternative calculation of the surface charge density, we note that the capacitance per unit length between the inner and outer conductors is C= 1 , 2 ln(b/a) (136)

so the charge per unit length needed to support the potential (a, z ) = IRz is Q(z ) = C(z ) = and the corresponding surface charge density is = as previously found in eq. (131). This argument helps us understand how the charge distribution and electric eld in the central region of the wire is insensitive to the physical details of the ends of the wire. The capacitance per unit length might be dierent from the expression (136) for a few wire diameters in z from the ends of the wire, but it is quite accurate over most of the length of the wire. Hence, we are less surprised that the potential (130) was obtained without ever specifying the boundary conditions at the ends of the wire. Those boundary conditions only aect the potential very near the ends of the wire, and the potential over most of the wire must have the form (130) in any case. The potential (130) can be thought of as a kind of zero-frequency mode of the cavity between the inner and outer conductors. This cavity more has a natural behavior at the ends, found by inserting zend into eq. (130). We readily see that this radial potential distribution would hold if the ends of the cable are terminated naturally in plates of uniform conductivity, so Er jr 1/r, and (r) ln r. If the coaxial cable transmits energy from a source (battery) at one end to a load (resistor) at the other, there is a net momentum dVol S/c2 stored in the elds, which is very small due to the factor 1/c2 . If the cable is an isolated system, then it also has an equal and opposite mechanical momentum. See, http://puhep1.princeton.edu/~mcdonald/examples/hidden.pdf Q IRz = , 2a 4a ln(b/a) (138) IRz , 2 ln(b/a) (137)

Princeton University 1999

Ph501 Set 5, Solution 11

34

11. a) If long coil 1 carries steady current I1, then the magnetic eld inside that coil is axial with magnitude 4N1 I1 B1 = , (139) cl by an application of Amp eres law, ignoring end eects. Outside the coil, the magnetic eld is zero. The ux linked by coil 1 is therefore,
2 1 = N1 r1 B1 = 2 2 4 2N1 r1 I 1 = cL1 I1, cl

(140)

so the self inductance of coil 1 is L1 = Similarly, the self inductance of coil 2 is L2 =


2 2 r2 4 2N2 . 2 c l 2 2 4 2N1 r1 . 2 c l

(141)

(142)

The mutual inductance can be calculated via the ux linked in coil 2 when coil 1 carries current I1 . Since the magnetic eld due to current I1 is zero outside coil 1, which is inside coil 2, we have
2 B1 = 12 = N2 r1 2 I1 4 2 N1N2 r1 = cMI1 , cl

(143)

2 4 2N1 N2 r1 . M= c2 l Since r2 > r1 , we have L1 L2 > M 2 .

so the mutual inductance is

(144)

In solving the coupled circuit equations in the presence of an oscillatory driving voltage at frequency , we use complex notation, and divide out the common factor eit. Then the symbols I1 and I2 are complex numbers such that the real current is Re I1eit , etc. The coupled equations are 2M = I1R1 + iI1L1 + iI2M, V0 = I1R1 + I1 L1 + I 1M = I2R2 + iI2L2 + iI1M. 0 = I2R2 + I2 L2 + I These are readily solved as I1 = I2 (R2 + iL2 )V0 , M 2 ) + i (R1 L2 + R2L1 ) iMV0 = 2 , R1 2 (L1 L2 M 2 ) + i(R1 L2 + R2 L1 ) R2 1 2 (L1 L2 (147) (148) (145) (146)

The time-average power dissipated in coil 2 is then, P2 =


2 |I 2 | R2 2 M 2 R2 V02 . = 2 (L L M 2 )]2 + 2 (R L + R L )2 2 2 [R2 1 2 1 2 2 1 1

(149)

Princeton University 1999

Ph501 Set 5, Solution 11

35

b) To calculate the Poynting vector S, we need the electric and magnetic elds. The (complex) magnetic eld is
4 (N1 I1 + N2 I2 )/cl,

Bz (r) =

4N2 I2/cl, 0,

r < r1 , r 1 < r < r2 , r < r2 .

(150)

The electric eld is azimuthal, as follows from Faradays law: E = 1 d r i r Bz 2r dr = Bz r dr rc 0 2rc dt 0 2 r < r1 , 2ir(N1 I1 + N2 I2 )/c l, 2 2 2 = 2 i ( r N I + r N I ) /c lr, r 2 2 1 < r < r2 , 1 1 1 2 2 2 2i (r1 N1 I1 + r2 N2 I2)/c lr, r2 < r,

(151)

The Poynting vector is radial, and positive if both E and Bz are positive. Its timeaverage value is Sr = (c/8 )ReE Bz . For r < r1 , E Bz is pure imaginary, so Sr = 0 here. Since Bz = 0 for r > r2 , Sr = 0 here also. The remaining region gives S r ( r 1 < r < r2 ) 2i 2 r2 N1 N2 c 4N2 I2 N1 I1 + r2 N2 I2 ) Re 2 (r1 = 122 Im(I1 I2) 8 c lr cl cl r 2 r1 N1 N2 MR2 V02 = 2 2 2 2 2 c2 l2r [R2 1 (L1 L2 M )] + (R1 L2 + R2 L1 ) 2 M 2 R2 V02 = 2 2 2 2 2 4lr [R2 1 (L1 L2 M )] + (R1 L2 + R2 L1 ) 1 P2 . = (152) 2rl =

Since 2rl Sr is the power transported by the electromagnetic eld across the cylinder of radius r and length l, we interpret the power consumed in the outer coil as owing from the inner, driven coil. c) If, instead, coil 2 is driven, then the solutions to the coupled equations are obtained from (147-148) by swapping indices 1 and 2. Likewise, the power consumed in coil 1 is obtained from (149) by the same swap of indices. The expressions (150-151) for the electric and magnetic elds in terms of the currents remain the same, as does the rst line of (152) for the Poynting vector. However, in the rest of (152) we must swap indices 1 and 2, and note the sign change that occurs in I1 . Thus, we nd S r ( r 1 < r < r2 ) = 1 P1 . 2rl (153)

Again, power ows from the driven coil to the load coil.

Princeton University 1999

Ph501 Set 5, Solution 12

36

12. a) The eld angular momentum is given by LEM = r Peld dVol = 1 4c r (E B) dVol = 1 4c [(r B)E (r E)B] dVol.

(154) From the symmetry of the problem, we infer that the angular momentum will be along the z axis, and that the electric and magnetic eld are independent of azimuth in spherical coordinates (r, , ). Thus, we desire LEM,z = 1 2c
0

r3 dr

1 1

d cos (Br Ez Er Bz ).

(155)

The magnetic eld due to magnetic dipole m z is 2 cos r + sin 3 cos rz m = m. r3 r3 The components we need are B= Br = 2mP1 (cos ) , r3 and Bz = cos Br sin B = 2mP2 (cos ) . r3 (156)

(157)

The electric eld can be gotten from the electric potential of a charged ring, p. 59 of the Notes with cos 0 = 0:
Q Q
r a

n n

n r a n a r

Pn (0)Pn (cos ), r < a, Pn (0)Pn (cos ), r > a.

(158)

Since Pn (0) = 0 for odd n, only even n terms contribute to the potential. The electric eld components are Er E
Q r r < a, ar n n a Pn (0)Pn (cos ), = = Q n a r r2 n (n + 1) r Pn (0)Pn (cos ), r > a. Q 1 ar = = Q r r 2

(159)

n n

n r 1 Pn (0)Pn (cos ), a n a 1 Pn (0)Pn (cos ), r

r < a, r > a.

(160) (161)

E = 0, using the fact that

dPn (cos ) 1 = Pn (cos ), (162) d m where Pn (cos ) is an associated Legendre polynomial. See eq. (3.39) of Jackson. We also need Ez = cos Er sin E , for which it is useful to note two recurrence relations (Gradshetyn and Ryzhik, 8.731.2 and 8.735.2): cos Pn (cos ) = (n + 1)Pn+1 (cos ) + nPn1 (cos ) , 2n + 1 1 sin Pn (cos ) = n cos Pn (cos ) nPn1 (cos ) n(n + 1) = [Pn+1 (cos ) Pn1 (cos )]. 2n + 1 (163)

(164)

Princeton University 1999 Then Ez = so that LEM,z = mQ = c mQ = c mQ = c =


0 a r3 dr 14 0 ar

Ph501 Set 5, Solution 12

37

Q Q
r2

ar

n n (n

r a

Pn (0)Pn1 (cos ),
a r n

r < a,

+ 1)

Pn (0)Pn+1 (cos ), r > a.

(165)

1 2c

r3 dr
n

1 1

d cos (Br Ez Er Bz )
a r 2

n
2

Pn (0)
a r 2 5 2 2

1 1

d [P2 ()Pn () P1 ()Pn1 ()]


1 1

a 3 0 r

3 a r

dr r15
1 dr ar 42

n (n a r 2 3

+ 1) 1 2
a r

Pn (0)
2 3 2 5

d [P1 ()Pn+1 () P2 ()Pn ()]

3 a r

dr r15

3
dr a r4

1 2

a 4 r 15a 0 2 dr 3 a r2

dr 2 + 3a 5

2mQ , 15ac 13mQ , 15ac

r < a, r > a.
1 1

(166) Pm ()Pn () d = (167)

where we have used the facts that P0 (0) = 1, P2 (0) = 1/2, and 2 mn /(2n + 1). Altogether, mQ , LEM,z = ac

b) As the magnetic moment m goes to zero, the electromagnetic angular momentum vanishes. But, the consequent change in the ux through the charged ring results in an azimuthal electric eld E around the ring, which causes a torque that increases the mechanical angular momentum: aQ d dLmech,z = aQE = . dt 2ac dt This integrates to Q Q a 2r dr Bz (169) initial = 2c 2c 0 If we use (157) for Bz , the result diverges. However, this form does not correctly account for the ux inside the small coil at the origin. We avoid this issue by noting that, since B = 0, the magnetic ux through the loop of radius a is the negative of the ux across the plane z = 0 outside the loop. In that plane, cos = 0, and since P2 (0) = 1/2, we have Lmech,z,nal = Lmech,z,nal = Q c
a

(168)

r dr Bz =

mQ c

mQ dr = = LEM,z,initial. 2 r ac

(170)

Remark: Equation (169) can be given another interpretation. The magnetic ux can be expressed in terms of the vector potential: initial = B dS = A dl = 2aAinitial, . (171)

Princeton University 1999 Thus,

Ph501 Set 5, Solution 12

38

Lmech,z,nal = (r Pnal)z = aPnal, = QAinitial, Since Pinitial, = 0 = Anal, , we can write r P+ QA c = constant.
z

(172)

(173)

This is the z component of the canonical angular momentum of a charged particle in an electromagnetic eld. Hence, another view of the Feynman disk paradox is that it illustrates the conservation of canonical angular momentum.

Princeton University 1999

Ph501 Set 5, Solution 13

39

13. The key to this problem is conservation of canonical momentum, P + eA/c, where A is the vector potential (in Gaussian units). It turns out to be even more eective to consider the canonical angular momentum, which is L = r (P + eA/c). We want P = 0 outside the magnet. This implies Lz = rP = 0 also. Therefore, we need r(P + eA/c) = 0 inside the magnet. A solenoid magnet with eld Bz has vector potential A = rBz /2. To see this, recall that the integral of the vector potential around a loop is equal to the magnetic ux through the loop: 2rA = r2 Bz . For a particle with average momentum in the z direction, its trajectory inside the magnet is a helix whose center is at some radius RG (called R0 in the statement of the problem) from the magnetic axis. The radius RB (called R in the statement of the problem) of the helix can be obtained from F = ma:
2 mv v = e Bz , RB c

(174)

so RB =

eBz . cP

(175)

The direction of rotation around the helix is in the z direction (Lenz law). Since the canonical angular momentum is a constant of the motion, we can evaluate it at any convenient point on the particles trajectory. In particular, we consider the point at which the trajectory is closest to the magnetic axis. As shown in Fig. 2, this point obeys r = RG RB , and so Lz = (RG RB ) P + eBz eBz 2 (RG RB )2 = R2 . G RB 2c 2c (176)

2 Note that R2 G RB is the product of the closest and farthest distances between the trajectory and the magnetic axis.

Hence, the canonical angular momentum vanishes for motion in a solenoid eld if and only if RG = RB , i.e., if and only if the particles trajectory passes through the magnetic axis. We also see that if the trajectory does not contain the magnetic axis, the canonical angular momentum is positive; while if the trajectory contains the magnetic axis, the canonical angular momentum is negative.

Princeton University 1999

Ph501 Set 5, Solution 13

40

RG RB RG P + BR RG B R Magnetic Axis P a) z L > 0 RG

+ R B

RB RG RG b) z L < 0 - R B < 0

Figure 2: The projection onto a plane perpendicular to the magnetic axis of the helical trajectory a charge particle of transverse momentum P . The magnetic eld Bz is out of the paper, so the rotation of the helix is clockwise for a positively charged particle. a) The trajectory does not contain the magnetic axis, and Lz > 0. b) The trajectory contains the magnetic axis, and Lz < 0.

Princeton University

Ph501 Electrodynamics Problem Set 6


Kirk T. McDonald (2001) kirkmcd@princeton.edu http://puhep1.princeton.edu/~mcdonald/examples/

Princeton University 2001

Ph501 Set 6, Problem 1

1. a) The intensity of sunlight at the Earths orbit is 1.4 106 erg/s/cm2 . What size chunk of earth ( 5 g/cm3 ) could be levitated without orbiting, but at the radius of the Earths orbit? b) Newtons Rings. Explain briey whether a dark or bright spot appears at the center, when viewing the reected and transmitted fringe patterns in the apparatus sketched on the left. Ignore multiple reections inside the glass in both parts b) and c).

c) Lloyds Mirror. Explain whether a dark or bright spot appears at the base of the screen in the apparatus sketched on the right.

Princeton University 2001

Ph501 Set 6, Problem 2

2. Carry out the derivation of Fresnels equations by matching the elds at the dielectric boundary, as discussed on p. 143 of the Notes. Deduce the four ratios: E0r , E0i E0t E0i (1)

for Ei polarized parallel and perpendicular to the plane of incidence. The derivation of Fresnels equations from Maxwells equation was rst performed by Helmholtz.

Princeton University 2001 3. Fresnels Rhomb.

Ph501 Set 6, Problem 3

Linearly polarized light can be converted to circularly polarized light, and vice versa, with Fresnels rhomb: a piece of glass cut in the shape of a rhombic prism. If the glass has index of refraction n = 1.5, show that the angle must be 50.2 or 53.3 . The eect is based on the phase change of totally internally reected light. Hint: write E0r = E0i ei , with ( ) for E ( ) to the plane of incidence, and show that tan 2 cos i sin2 i 1/n2 = . sin2 i (3) (2)

Princeton University 2001

Ph501 Set 6, Problem 4

4. (a) More amplitude analysis. On pp. 141-142 of the Notes, we considered reection and transmission at a dielectric boundary, using the amplitudes i, r and t, which are proportional to the electric elds of the incident, reected, and transmitted waves, respectively.

We found that if i = 1, then |r | = and |r|2 =


2

sin2 (1 2) , sin2(1 + 2 ) tan2 (1 2 ) , tan2( 1 + 2 )

for E to the plane of incidence,

(4)

for E

to the plane of incidence.

(5)

Can we deduce relations for the phases of r and t, and not merely their amplitudes? See sec. 33-6 of Vol. I of the Feynman Lectures on Physics. Consider now an inverse situation:

If i = 1, then conservation of energy tells us that |r| + |t| = |r | + |t | . We can also consider something even more peculiar:
2 2 2 2

(6)

Princeton University 2001

Ph501 Set 6, Problem 4

This can be regarded as the time reversal of the original situation. But, we also recognize this as the superposition of two more ordinary congurations:

In particular, if a wave of amplitude r were incident from side 1, then the transmitted wave would have amplitude rt in terms of our original denitions. Show that this implies that r is real, while t = t . Hint: rst deduce that r = r and |t | = |t|. Then, multiply the relation 1 = r2 + tt by its complex conjugate.... (b) Dielectric Slab. Consider a plate of thickness d of a dielectric with index of refraction n2 , surrounded by a medium of index n1 = 1. A wave of unit amplitude is incident from below. Multiple reections occur whose interference leads to the reected and transmitted waves, being the sums of the amplitudes at the dashed wavefronts shown in the gure.

Show that the waves corresponding to a ray and its next higher-order neighbor have a phase dierence 2 due to the dierent path lengths they have traveled, where = 2d cos 2 , 2 (7)

while the phase lag for the rst transmitted ray compared to the case of no plate is , where 2d cos 1 = . (8) 1 Dene the total reected and transmitted amplitudes to be R and T , respectively.

Princeton University 2001

Ph501 Set 6, Problem 4

Sum the partial amplitudes, and use the results of part (a) to show that R= r(1 e2i ) , 1 r2 e2i and T = (1 r2 )ei( ) , 1 r2 e2i (9)

which obey energy conservation: |R|2 + |T |2 = 1. The ratio of the reected to transmitted amplitudes is r(1 e2i ) R 2iei r sin = = T (1 r2 )ei( ) 1 r2 (10)

so there is a phase dierence of /2 between amplitudes R and T . An experiment that would be sensitive to this phase dierence could involve a second beam, incident on the beam splitter at angle 1 , but from the opposite side of the beam splitter from the original input beam. Then, the transmitted part of the rst beam would interfere with the reected part of the second beam. If the two input beams are in phase at, say, the midplane of the beam splitter, they would have a phase dierence of at the surface of the splitter onto which the second beam is incident. So, when considering the interference of the two beams, the phase found in eq. (10) would drop out, and we should say that there is an eective phase dierence of 90 between the reected and transmitted amplitudes in a beam splitter of nite thickness. This 90 phase dierence plays an important role when comparing the classical and quantum behavior of a beam splitter: http://puhep1.princeton.edu/~mcdonald/examples/bunching.pdf An argument due to Feynman [Chaps. 30-31 of Vol. 1 of the Feynman Lectures on Physics; pp. 282-285, Lecture 23 of the Notes] shows that R is i times a positive number, and hence that r is negative. Then, eqs. (4) and (5) lead to r= and r= sin(1 2 ) , sin(1 + 2 ) tan(1 2 ) , tan(1 + 2 ) for E to the plane of incidence, (11)

for E

to the plane of incidence.

(12)

The expressions (9) for R and T are valid even if n1 > n2. Then, for large enough 1, we expect total internal reection. In the Notes, we found for this case that cos 2 = i (n1 /n2 )2 sin2 1 1, so eq. (7) gives = Thus, T = d 2id (n1 /n2 )2 sin2 1 1 i . 2 as d . (14) (13)

(1 r2 )ed/ ei 0 1 r2 e2d/

(15)

Princeton University 2001

Ph501 Set 6, Problem 4

But for nite thickness d the transmitted amplitude is nonzero, even though we found no wave motion in medium 2 which traveled normal to the boundaries. This phenomenon is called tunneling.

Princeton University 2001 5. (a) Antireection Coatings

Ph501 Set 6, Problem 5

Suppose the slab of dielectric of prob. 4 separates media of indices n1 and n3.

Show that the reected and transmitted amplitudes obey r12 + r23e2i t12t23ei , T = , (16) 1 + r12r23e2i 1 + r12 r23e2i where r12 is the amplitude for a single reection at boundary 1-2, etc. Consider the special case of normal incidence. Show that if n2 = n1 n3 and d = 2 /4, then R = 0, which is a prescription for an antireection lens coating. R= Show also that if d = 2/2 the R is independent of n2 , and if in addition n1 = n3 then R vanishes. (b) Dielectric Mirrors Can we make a good mirror by applying an appropriate dielectric coating on a plate of glass? Not with only two layers, but consider a multilayer mirror. For example, if a medium 4 exists beyond medium 3, then the reection at the 2-3 boundary could be described by r23 + r34e2i3 t23t34ei3 , T23 = , (17) R23 = 1 + r23r34e2i3 1 + r23r34 e2i3 t12t23t34ei(2 +3 ) T = , etc . (18) (1 + r23 r34e2i3 )(1 + r12 R23e2i2 ) Then, for a stack of n 2-3 pairs, 1-2-3-2-3- -3-2-1, T = |t12|2 |t23|2n eni(2 +3 ) t12t23t32t23 t32t21eni(2 +3 ) = , big mess big mess (19)

where the big mess is not small if 2 = 3 = /4, since in that case 1 + r23 r32e2i3 = 2 , etc. Since |t23|2n 0 for large n, T 0 and R 1. 1 + r23 The prescription given here for a multilayer dielectric mirror works well for only a narrow range of angles of incidence and a narrow range of wavelengths, since we require that 2 = 3 = /4. Better prescriptions can be given that maintain very high reectivity over a large range of parameters. See, for example, J.P. Dowling, Science 282, 1841, (1998).1
1

http://puhep1.princeton.edu/~mcdonald/examples/optics/dowling_science_282_1841_98.pdf

Princeton University 2001

Ph501 Set 6, Problem 6

6. In 1890, O. Wiener carried out an experiment that can be said to have photographed electromagnetic waves. a) A plane wave is normally incident of a perfectly reecting mirror. A glass photographic plate is placed on the mirror at a small angle . The polarization of the wave is parallel to the line of intersection of the mirror and the plate. The photographic emulsion is almost transparent ignore attenuation and reection in it and in the glass. When the plate is developed a striped pattern is observed.

Calculate the electromagnetic elds E and B for y > 0, where y = 0 is the surface of the mirror. Predict the position and spacing of the dark stripes that appear on the developed negative plate. b) Repeat the discussion for waves incident at 45 . That is, calculate E and B for y > 0, and predict the pattern of blackening on the negative.

Distinguish the case of E and

to the plane of incidence.

Princeton University 2001

Ph501 Set 6, Problem 7

10

7. Two airplanes are ying at distance d apart at height h above the ocean whose dielectric constant is . One plane sends signals to the other. Both airplanes have short vertical antennae. Ignore the curvature of the Earth. Show that the ratio of the intensity of the signal reected o the ocean to that of the direct signal is ( 1)d2 + 4 h2 2 h d6 . (d2 + 4h2 )3 ( 1)d2 + 4 h2 + 2 h In addition to facts about plane waves you need to know that The intensity of spherical broadcast waves falls o as 1/r2 . Over small spatial regions (except close to the source) the spherical waves can be considered as plane waves. The amplitude (E eld) of a broadcast wave varies linearly with the projection perpendicular to the line of sight of the motion (acceleration) of the charges that cause the wave (p. 141 of the Notes). Likewise, the current I excited in a receiving antenna varies as the projection of E onto the antenna. The power of the received signal is, of course, I 2R, where R is the resistance in the receiving antenna.
2

(20)

Princeton University 2001

Ph501 Set 6, Problem 8

11

8. (a) Plasma with a dc magnetic eld. Consider the Earths ionosphere to be a plasma of uniform density with a static, uniform magnetic eld B0 (the Earths eld) in the +z direction. Discuss the propagation of circularly polarized plane radio waves parallel (or antiparallel) to B0. The response of an ionized electron of charge e and mass m at position r to the wave of angular frequency is described by r m r + e B0 = eEei(kzt) , c where for circularly polarized waves the electric eld amplitude can be written ). E = E0 ( x iy Show that r = where eE , m ( B ) (22) (21)

(23)

eB0 , mc and that this implies a dielectric constant for the plasma of B =

(24)

= 1

2 p , ( B )

(25)

where the plasma frequency p is given by 2 p = 4Ne2 m (26)

for a plasma of number density N per cm3. + iy (called left-handed in optics although Show that for waves of circular polarization x a photograph of the electric eld vector would show it to behave like a right-handed screw), B vgroup = 2vphase = 2c . (27) p [See pp. 146a-d of the Notes for a discussion of group and phase velocity.] It turns out that B p 107 Hz in the ionosphere. Estimate the dierence in arrival times for signals of 105 and 2 105 Hz originating simultaneously (in lightning ashes) at the opposite side of the Earth. This illustrates the whistler or chirp eect as higher frequency waves arrive rst. iy ? What is the fate of waves of polarization x

Princeton University 2001 (b) Reections and Mirages

Ph501 Set 6, Problem 8

12

In the ionosphere the density of ionized electrons actually increases with height: the lower atmosphere is screened from the Sun by the upper. Hence, the (frequencydependent) index of refraction decreases with height, since n( ) 1 ( p / )2, where p is the plasma frequency.

Suppose at the bottom of the atmosphere, where n = 1, a radio wave propagates upwards with angle i to the vertical. Use Snells law to show that the wave is reected back downwards if the electron density rises until p = cos i at some height. Mirages are a similar phenomenon in which higher temperatures in the air close to the Earths surface result in lower density (N = RT /P ) at lower height, and hence lower 2 2 index at lower height for optical frequencies (n 1 + 2 p /( 0 )), so downward going light rays can be reected upwards near the surface.

Princeton University 2001

Ph501 Set 6, Problem 9

13

9. A plane electromagnetic wave of angular frequency is normally incident on a good conductor that occupies the region z > 0. Show that the Poynting vector S evaluated at z just greater than zero is equal to the power (per unit area to z ) lost to Joule heating in the conductor.

Princeton University 2001

Ph501 Set 6, Problem 10

14

10. A plane electromagnetic wave of angular frequency is normally incident on a thin conducting sheet of thickness a d, the skin depth. Ignoring reection, show that the relative transmitted intensity is T = 1 where is the conductivity. Use an energy argument as in prob. 8. Extending the argument to show that the relative reected intensity R is (2a/c)2 . A trick derivation is to note that a sheet of unit area and thickness a has resistance R= 1 1 l = = A a 1 a (29) 4 a, c (28)

to the induced currents that ow in the plane of the sheet. Hence, power V 2/R if absorbed is a wave of voltage V per unit length crosses the sheet. But, the power carried by a plane wave is V 2 /Rvac where Rvac = 4/c = 377 . Thus, the fractional power absorbed is Rvac/R = 4a/c.

Princeton University 2001 11. Total Internal Reection

Ph501 Set 6, Problem 11

15

A wave of frequency is incident at angle i on the boundary between dielectrics of indices n1 > n2 . Find the time-averaged Poynting vector of the transmitted wave when sin i > n2 /n1 , i.e., when total internal reection occurs.

Verify that the transmitted surface wave satises the wave equation and Maxwells equations.

Princeton University 2001 12. The grating accelerator

Ph501 Set 6, Problem 12

16

In optics, a reective grating is a conducting surface with a ripple. For example, consider the surface dened by 2x z = a sin . (30) d The typical use of such a grating involves an incident electromagnetic wave with wave vector k in the x-z plane, and interference eects lead to a discrete set of reected waves also with wave vectors in the x-z plane.

Consider, instead, an incident plane electromagnetic wave with wave vector in the y -z plane and polarization in the x direction: ei(ky ykz zt), Ein = E0 x (31)

d), this leads to a reected where ky > 0 and kz > 0. Show that for small ripples (a wave as if a = 0, plus two surface waves that are attenuated exponentially with z . What is the relation between the grating wavelength d and the optical wavelength such that the x component of the phase velocity of the surface waves is the speed of light, c? In this case, a charged particle moving with vx c could extract energy from the wave, which is the principle of the proposed grating accelerator [R.B. Palmer, A Laser-Driven Grating Linac, Part. Accel. 11, 81-90 (1980)].2

http://puhep1.princeton.edu/~mcdonald/examples/accel/palmer_pa_11_81_80.pdf

Princeton University 2001

Ph501 Set 6, Problem 13

17

13. A radiofrequency quadrupole (RFQ) is a device for focussing beams of charged particles. The electric eld in this device can be approximated as that derived from the quasistatic potential (x, y, t) = E0 2 (y x2) sin t, 2d (32)

where d is a length and is the frequency of the eld. The magnetic eld is ignored in this approximation. While the approximate elds do not satisfy Maxwells equations, there is little error for |x|, |y | , the wavelength of the radiofrequency waves. Deduce the equations of motion for a particle of charge e and mass m in the radiofrequency quadrupole. Consider solutions of the form x(t) = f (t) + g (t) sin t (33)

where g f and both f and g are slowly varying compared to sin t. The parameters may be assumed to satisfy the conditions that such solutions exist. Complete the solution for the particular case that x(0) = 0, y (0) = 0, z (0) = 0, x (0) = v00 , y (0) = 0, z (0) = v0, (34) (35) (36)

1. At what distance along the z -axis is the rst image of the beam spot, with 0 i.e., where the initially diverging beam is brought back to the z -axis?

Princeton University 2001 14. Pulsar Timing

Ph501 Set 6, Problem 14

18

The distance from the Earth to a pulsar can be estimated by observing the dispersion of the radio-frequency pulses as they cross the interstellar medium. a) Suppose the medium is a plasma of N electrons/cm3. What is the index of refraction n( ) where is the angular frequency of a wave? b) The pulsar emits a short pulse that contains a broad range of frequencies. We observe the pulse in a receiver that is tuned to a narrow band about an adjustable central frequency . We measure the time dierence t between the arrival of two components of the pulse, centered at frequencies and + , where . This can be done in a single receiver if the pulsar has a precise pulse rate as is the case. Pulsars are the most accurately periodic macroscopic phenomenon ever observed! Which component, or + , arrives rst, and by how much, as a function of the Earth-pulsar distance L? Use the following representative values to calculate the distance L to pulsar 1913+16: = 2000 MHz, / = 0.01, N = 0.04 electrons/cm3, and |t| = 0.004 s. A pulsar tidbit: Many pulsars occur in binary systems, including one such system where the direction to the Earth lies very close to the plane of the orbit. By observing the small general-relativistic pulse delays that occur when one of the binary partners occults the other, the eccentricity of the orbit can be determined to remarkable accuracy. The data indicate that the orbit has radius REarthSun and a small eccentricity corresponding to being out of round by less than 1 cm! The orbit is round to one part in 1013 , which makes it the roundest object ever measured and its at the other end of the galaxy (F. Camilo, Princeton Ph.D. thesis, 1994).

Princeton University 2001

Ph501 Set 6, Solution 1 Solutions

19

1. a) For a black chunk of earth of radius r levitated at distance RE = 1.5 1013 cm in sunlight of intensity I = 1.4 106 erg/s/cm2 , FG = Hence r= 3IR2 3 1.4 106 (1.5 1013 )2 E = 1.2 105 cm, 4cGMS 4 3 1010 5 6.7 108 2 1033 (38) I GMS m 4r3 GMS = = Frad = r2 . 2 2 RE 3RE c (37)

which is less than a wavelength of light! b) The center of the reected spot is dark, being the interference between the reection o the bottom surface of the lens and the top surface of the glass plate. There is a 180 phase dierence between these two cases. (The reections o the top of the lens and bottom of the glass plate give a background intensity that is independent of position.) The center of the transmitted spot is bright, as there is no phase shift during transmission across a dielectric boundary. c) The fringe pattern is dark at the base of the screen, due to interference of the direct and reected rays. The latter undergo a 180 phase shift on reection.

Princeton University 2001

Ph501 Set 6, Solution 2

20

2. We consider plane waves of the form E = E0ei(krt) , B n H = = k E=

E, k

(39)

/c = n/c in media with dielectric constant , index n = , and where k = permeability . At a boundary between two dielectrics, the perpendicular components of D and B, and the parallel components of E and H are continuous. The incident and reected waves are in medium 1, and the transmitted wave is in medium 2. The unit normal . Then the boundary conditions are vector pointing into medium 2 is labeled n n2 1 (E 0 i + E 0 r ) n 1 i E0 i + k r E0 r ) n n1 (k (E 0 i + E 0 r ) n n1 r E0 r ) n (k i E 0 i + k 1 Of course, Snells law tells us that n1 sin 1 = n2 sin 2 . (44) n2 2 , E0 t n 2 t E0 t n , = n2 k , = E0 t n n2 , = k t E0 t n 2 = (40) (41) (42) (43)

i, k r (a) Polarization perpendicular to the plane of incidence (the plane containing k t). and k

Relation (40) is satised identically. Both relations (41) and (42) yield E0i + E0r = E0t. Relation (43) tells us that n2 n1 sin 1 cos 2 n1 (E0i E0r ) cos 1 = E0t cos 2 = E0t , 1 2 2 sin 2 (46) (45)

Princeton University 2001 and hence,

Ph501 Set 6, Solution 2

21

E0i E0r = Adding (45) and (47), we nd that

1 sin 1 cos 2 E0t. 2 cos 1 sin 2

(47)

2n1 cos 1 2 sin 2 cos 1 E0t = 1 E0i n1 cos 1 + n2 cos 2 sin(1 + 2 )


2

if 1 = 2 = 1.

(48)

Then, (45) leads to n1 cos 1 E0t E0r = 1= E0i E0i n1 cos 1 +


1 n 2 2 1 n 2 2

cos 2 cos 2

sin(1 2) sin(1 + 2 )

if 1 = 2 = 1.

(49)

(b) Polarization parallel to the plane of incidence.

Relation (40) leads to n2 n2 1 (E0i E0r ) sin 1 = 2 E0t sin 2 , 1 2 which simplies to E0i E0r = 1 n2 sin 1 E0t E0t 2 n1 sin 2 if 1 = 2 = 1. (51) (50)

using Snells law. Similarly, relation (42) leads to E0i + E0r = Adding (51) and (52), we nd that E0t 2n1 cos 1 2 sin 2 cos 1 4 sin 2 cos 1 = = 1 E0i n2 cos 1 + n1 cos 2 sin 1 cos 1 + sin 2 cos 2 sin 2 1 + sin 22
2

cos 2 E0t. cos 1

(52)

2 sin 2 cos 1 = sin(1 + 2 ) cos(1 2 )

if 1 = 2 = 1.

(53)

Princeton University 2001

Ph501 Set 6, Solution 2

22

Combining this with (52), we have E0r E0i = n1 cos 2 cos 2 E0t 1 = cos 1 E0i n1 cos 1 +
1 n 2 2 1 n 2 2

cos 1 cos 2

2 sin 2 cos 2 1 sin 1 cos 1 + sin 2 cos 2 sin 2 cos 2 sin 1 cos 1 sin(1 2) cos(1 + 2 ) = = sin 1 cos 1 + sin 2 cos 2 sin(1 + 2 ) cos(1 2 ) tan(1 2 ) = if 1 = 2 = 1. tan(1 + 2 )

(54)

(c) Normal Incidence. Taking the limit of either polarization as 1 0 and 2 0, we nd 2n1 2n1 E0t = 1 E0i n2 + n1 n1 + n2
2

if 1 = 2 = 1,

(55)

n1 E0r = E0i n1 +

1 n 2 2 1 n2
2

n1 n2 n1 + n2

if 1 = 2 = 1.

(56)

Princeton University 2001

Ph501 Set 6, Solution 3

23

3. For a linearly polarized wave, E and E are in phase, while for a circularly polarized wave their phase dierence is = 90 . In Fresnels rhomb, there are two internal reections, each of which causes phase changes and for light polarized perpendicular and parallel to the plane of incidence, respectively. Hence, if = 45 (57)

at each reection, we will achieve the desired conversion of linearly into circularly polarized light. In case of total internal reection where media 1 and 2 have indices n1 = n and n2 = 1, we use Snells law to write sin 2 = n sin 1 , and cos 2 = 1 n2 sin2 1 = in sin2 1 1/n2 . (58)

Then, eqs. (49) and (54) can be written as E0r E0i sin 1 in sin2 1 1/n2 cos 1 n sin 1 sin(1 2 ) = = sin(1 + 2) sin 1 in sin2 1 1/n2 + cos 1 n sin 1 = and E0r E0i = tan( 1 2 ) tan(1 + 2 ) cos 1 i sin2 1 1/n2 cos 1 + i sin2 1 1/n2 , (59)

cos 1 i sin2 1 1/n2 cos 1 in sin2 1 1/n2 sin 1 n sin 1 = cos 1 + i sin2 1 1/n2 cos 1 in sin2 1 1/n2 + sin 1 n sin 1 = cos 1 i sin2 1 1/n2 cos 1 + i sin2 1 1/n2 cos 1 cos 1 i sin2 1 1/n2 + 1 cos 1 (1 1/n2 1) + i sin2 1 1/n2 cos 1 (1 1/n2 1) + i sin2 1 1/n2 cos 1 in2 sin2 1 1/n2 cos 1 + in2 sin2 1 1/n2 . (60) cos 1 cos 1 + i sin2 1 1/n2 1

Both eqs. (59) and (60) have the form a2 b2 2iab a ib ei , = a + ib a 2 + b2 (61)

Princeton University 2001 so tan =

Ph501 Set 6, Solution 3

24

2ab 2(a/b) = a 2 b2 1 (a/b)2 2 tan(/2) = tan 2(/2) = , 1 tan2 (/2) a tan(/2) = . b

(62)

and hence, Thus, tan( /2) = and

(63)

sin2 1 1/n2 , cos 1

and

tan( /2) = n2

sin2 1 1/n2 , cos 1 (64)

tan( /2 /2) = =

tan( /2) tan( /2) 1 + tan( /2) tan( /2) sin2 1 1/n2 1 n2 2 1/n2 1 cos 1 1 + n2 sin cos 2 1 cos 1 sin2 1 1/n2 . sin2 1 (65)

The angle of incidence, 1 , is the same as angle shown in the gure for Fresnels rhomb. Thus, condition (57) implies that cos sin2 1/n2 = tan 22.5 = ( 2 1), 2 sin or (3 2 2) sin4 = (1 sin2 )(sin2 1/n2 ), (4 2 2) sin4 (1 + 1/n2 ) sin2 + 1/n2 = 0, 1 + 1/n2 (1 + 1/n2 )2 4(4 2 2)/n2 2 sin = . 2(4 2 2) (66)

(67) (68) (69)

For n = 1.5, we nd = 50.2 and 53.3 .

Princeton University 2001

Ph501 Set 6, Solution 4

25

4. (a) Since the time reversed case is the superposition of the two cases shown below the former, we conclude that (70) i = 1 = r2 + tt , and 0 = rt + tr . From eq. (71) we learn that r = r. Combining this with conservation of energy, eq. (6), we nd that |t | = |t | . Given these relations, we can now write r = r0ei , where r0 and t0 are real and obey
2 r0 + t2 0 = 1.

(71) (72)

(73)

t = t0 ei ,

and

t = t0 ei ,

(74)

(75)

Inserting relations (74) into eq. (70), we nd


2 2i i( + ) e + t2 . 1 = r0 0e

(76)

Multiplying eq.(76) by its complex conjugate yields


4 2 2 1 = r0 + t4 0 + 2r0 t0 cos(2 ).

(77)

In view of relation (75), we must have 2 = + . Then, eq. (76) can be rewritten as
2 2i + t2 = e2i. 1 = (r0 0 )e

(78)

(79)

Hence, = n, where n is an integer, from which we conclude that r (and r ) is real. Further, + = 2n, which implies that t =t , according to the denitions (74). (b) The phase dierence 2 is that between points a and b in the gure (82) (81) (80)

Princeton University 2001

Ph501 Set 6, Solution 4

26

Namely, 2 =

2l sin 1 2 2d , 2 cos 2 1

(83)

where l = 2d tan 2 . The wavelengths are related by n1 1 = n2 2, so eq. (83) becomes 2 = using Snells law. The phase lag for the rst transmitted ray is that between points a and b in the gure below: n1 sin 1 tan 2 cos 2 2 2d 1 2 cos 2 n2 = 4d cos 2 , 2 (84)

We note that l = d(tan 1 tan 2 ), so that = 2d 2d 2l sin 1 + 1 2 cos 2 1 cos 1

Princeton University 2001 =

Ph501 Set 6, Solution 4

27

2d sin 1 tan 2 2d 2d sin 1 tan 1 2d + 2 cos 2 1 1 cos 1 1 = , 2d cos 1 . 1

(85)

with = The total reected amplitude is

(86)

R = r + tr t e2i + tr 3 t e4i + ... = r |t|2 re2i


n=0

(r2 e2i)2 (87)

= r

(1 r )re 1 r2 e2i
2

2i

r(1 e ) , 1 r2 e2i
2i

using relations (72) and (82). Similarly, the total transmitted amplitude is T = tt ei( )
n=0

(r2 e2i )2 =

(1 r2 )ei( ) . 1 r2 e2i

(88)

For a discussion of tunneling via the Poynting vector, see D. Mugani, Opt. Comm. 175, 309 (2000).3

http://puhep1.princeton.edu/~mcdonald/examples/EM/mugnai_oc_175_309_00.pdf

Princeton University 2001

Ph501 Set 6, Solution 5

28

5. (a) The reected amplitude for the 3-layer medium is R = r12 + t12r23t21e2i + t12r23r21 r23t21e4i + ... = r12 + t12t21r23e2i
j =0 j

r21r23e2i

= r12 + t12t21

r23 e2i . 1 r21 r23e2i

(89)

2 From prob. 4, r21 = r12, and t12t21 = |t12|2 = 1 r12 , so eq. (89) becomes 2 ) R = r12 + (1 r12

r23e2i 1 + r12r23 e2i (90)

= Similarly,

r12 + r23 e2i . 1 + r12r23 e2i

T = t12t23ei + t12e3i + t12r23r21r23 r21t23e5i + ... = t12t23ei + t12r23r21t23e3i


j =0

r21r23 e2i

= t12t23ei + t12t23 = t12t23ei =

r21 r23e 1 r21r23 e2i r12 r23e2i 1 1 + r12r23e2i (91)

3i

t12t23ei . 1 + r12r23 e2i

At normal incidence the reected amplitude at the a-b boundary is rab = na nb . na + nb (92)

If d = 2 /4, then = 2d/ 2 = /2, and R= r12 r23 = r12 r23 , 1 r12r23 (93) (94)

(n1 n2 )(n2 + n3 ) (n2 n3 )(n1 + n2) 2(n1 n3 n2 2) = , (n1 + n2 )(n2 + n3) (n1 + n2 )(n2 + n3 ) and R = 0 if n2 = n1 n3 . If d = 2 /2, then = , and R= (n1 n2 )(n2 + n3) + (n2 n3 )(n1 + n2 ) n1 n3 r12 + r23 = , = 1 + r12r23 (n1 + n2 )(n2 + n3 ) + (n1 n2 )(n2 n3 ) n1 + n3

(95)

which is independent of n2 , and vanishes if media 1 and 3 are the same.

Princeton University 2001

Ph501 Set 6, Solution 6

29

6. a) The photographic plate intersects the mirror along the line x = y = 0. The incident wave of frequency and polarization along the z axis has electric and magnetic elds ei(kyt), Ei = E0z ei(kyt) , Bi = E0 x (96)

where k = c = 2/, c is the speed of light, and is the wavelength. The reected wave has electric eld with a 180 phase change so as to satisfy the boundary condition that the tangential electric eld vanish at the surface of the mirror. Hence, ei(kyt) , ei(kyt) . Br = E0 x (97) Er = E0z The total elds are standing waves: E = Ei + Er = Re(2iE0 z sin kyeit ) = 2E0 z sin ky sin t, it cos kye cos ky cos t. B = Bi + Br = Re(2E0 x ) = 2E0 x (98) (99)

The photographic plated is exposed by energy transfer between the electromagnetic elds and the emulsion. Recall that a magnetic eld cannot change the energy of a charged particle, while an electric eld can. We conclude that it is the electric eld (98) whose spatial dependence will determine the pattern of exposure of the photograph. Since the electric eld energy depends on E 2 , the pattern of exposure will actually follow the time average E 2 sin2 ky 1 cos 2ky . We also recall that the developed photographic negative would be black everywhere if it were unexposed. Exposure due to strong electric elds will result in transparent regions on the negative. The blackest stripes on the negative appear where the electric eld energy vanishes, i.e., at y = n/2. For a plate making angle to the x axis, y = s sin , where distance s is measured from the edge of the plate in contact with the mirror. Hence, the black stripes on the negative appear at n . (100) s= 2 sin If the incident wave had polarization along the x axis, the striping on the negative would be parallel to the x axis with periodicity /2. b) Now, the incident wave has a 45 angle of incidence, and the plane of incidence is the x-y plane. We rst consider the case of polarization perpendicular to the plane of incidence. Then, the incident wave vector is k ), xy (101) ki = ( 2 and the electromagnetic elds are Ei = E0 zei(kx/
2ky/ 2t)

,
i(kx/ 2ky/ 2t)

(102) . (103)

E0 i Ei = )e ( x+y Bi = k 2

Princeton University 2001 The reected wave has

Ph501 Set 6, Solution 6

30

k ), kr = ( x+y 2
2+ky/ 2t)

(104)

and the electromagnetic elds are zei(kx/ Er = E0 ,


i(kx/ 2+ky/ 2t)

(105) . (106)

E0 r Er = )e ( xy Br = k 2 The total elds are the waves:

ky z sin ei(kx/ 2t) E = Ei + Er = Re 2iE0 2 ky = 2E0 z sin sin(kx/ 2 t), (107) 2 ky ky cos + iy sin ei(kx/ 2t) B = Bi + Br = Re 2E0 x 2 2 ky ky sin sin(kx/ 2 t) . (108) cos cos(kx/ 2 t) + y = 2E0 x 2 2 This is a travelling wave in the x direction, modulated in y by sin ky/ 2. Then,

ky E 2 sin2 1 cos 2ky, 2 so the dark stripes appear on the plate at positions 2n s= . 2 sin For polarization parallel to the plane of incidence,

(109)

(110)

E0 )ei(kx/ 2ky/ 2t), x+y Ei = ( (111) 2 i Ei = E0 Bi = k zei(kx/ 2ky/ 2t), (112) E0 )ei(kx/ 2ky/ 2t) , (113) xy Er = ( 2 r Er = E0 z ei(kx/ 2ky/ 2t) , Br = k (114) ky ky sin y cos ei(kx/ 2t) E = Ei + Er = Re 2E0 ix 2 2 ky ky sin sin(kx/ 2 t) + y cos cos(kx/ 2 t) , (115) = 2E0 x 2 2 ky z cos ei(kx/ 2t) B = Bi + Br = Re 2E0 2 ky cos cos(kx/ 2 t). = 2E0 z (116) 2

Princeton University 2001 Thus,

Ph501 Set 6, Solution 6

31

ky ky E 2 sin2 + cos2 = 1, 2 2 so the photographic plate would be uniformly exposed.

(117)

Princeton University 2001

Ph501 Set 6, Solution 7

32

7. Since the intensity I E 2 , and I 1/r2 for a wave that emanates from a localized source, we must have E 1/r for the electric eld of the wave. Hence, if we label the eld strength of the source at airplane a as Ea , the eld strength at airplane b is Eb = Ea , d (118)

where d is the distance between the two airplanes, taken to be at the same height h.

As shown in the gure, the reected wave makes angle of incident 1 with respect to the surface of the ocean. The reected angle is, of course, also 1 , while the transmitted angle 2 obeys Snells law, sin 2 = d/2l n1 sin 1 sin 1 = = = n2 d (d2 + 4h2 ) , (119)

in the approximation that index n1 = 1 for air. For later use, we note that tan 1 = and tan 2 = sin 2 1 sin 2
2

d , 2h d ( 1)d2 + 4 h2 .

(120)

(121)

The amplitude of the wave emitted at airplane a which is reected by the ocean is smaller than that of the direct wave by the factor cos = sin 1. Likewise, the amplitude of the currents excited in the antenna on airplane b by a wave that makes angle is smaller by the factor cos than that due to the direct wave. Since the antenna on airplane a is vertical, the polarization of the emitted wave is in a vertical plane, which is also the plane of incidence of the wave with the ocean. Upon reection, the wave suers a loss of amplitude described by the ratio Er Ei = = tan 2 tan 1 1 tan 1 tan 2 tan(1 2 ) = tan(1 + 2 ) tan 2 + tan 1 1 + tan 1 tan 2 ( 1)d2 + 4 h2 2 h ( 1)d2 + 4 h2 + 2 h , (122)

Princeton University 2001

Ph501 Set 6, Solution 7

33

according to the Fresnel equation (54), and eqs. (120)-(121). Also, the path length of the reected wave between airplanes a and b is 2l, so the amplitude of the reected wave has fallen o by factor 1/2l. Altogether, the excitation in antenna b due to the reected wave is proportional to ER = Ea cos 1 Er 2l Ei cos , (123)

while that due to the direct wave is proportional to 1 ED = Ea . d Therefore, the ratio of the intensity of the reected to the direct signal is
2 ER d2 IR tan2 (1 2 ) 4 = = sin 1 2 ID ED 4l2 tan2 (1 + 2 )

(124)

( 1)d2 + 4 h2 2 h d6 . = 2 2 (d2 + 4h2 )3 ( 1)d + 4 h + 2 h

(125)

Princeton University 2001

Ph501 Set 6, Solution 8

34

8. (a) The equation of motion of an ionized electron of chage e and mass m in a circularly polarized plane wave is r )ei(kzt). m r + e B0 x iy z = eE0( c We seek solutions of a similar form: )ei(kzt) . x iy r = r0 ( Inserting eq. (127) into (126) we nd ) i m 2 r0( x iy r0 m 2 and hence, r0 = where eE0 , m ( B ) (130) eB0 ix ) = eE0( ), r 0 ( y x iy c (128) (129) (127) (126)

eB0 ) = eE0( ), ( x iy x iy c

eB0 . (131) mc Note that for propagation antiparallel to the direction of the magnetic eld, B is a negative number. B = Since r measures the separation of electrons from positive ions, the resulting polarization density is Ne2 P = Ner = (132) E E, m ( B ) and the dielectric constant is

= 1 + 4 = 1

2 4Ne2 p =1 , m( B ) ( B )

(133)

where the square of the plasma frequency is given by 2 p = For radio waves with B p ,

4Ne2 . m

(134)

2 p . B

(135)

The phase velocity of the plane waves is related by vphase = c c = = . k n (136)

Princeton University 2001

Ph501 Set 6, Solution 8

35

Comparing eqs. (135) and (136) we see that the phase velocity is imaginary for waves iy , which means that these waves are attenuated rapidly. Only with polarization x +i y propagate in the ionosphere, and their phase velocity the waves with polarization x is B vphase = c . (137) p For propagation opposite to the direction of the magnetic eld, the situation is reversed, iy survive. For the surviving waves, the wave and only wave with polarization x vector is related to frequency by k= c
+

p c

, B

(138)

and so the group velocity is given by 1 B d = = 2c vgroup = = 2vphase . dk dk/d p For waves with 105 Hz and B p 107 Hz, vphase c/10. The dierence in arrival times for pulses centered on frequencies 1 = 105 and 2 = 2 105 Hz from the opposite side of the Earth (d = 2 109 cm, which used to be the denition of a centimeter) is t = d d p d 2 = 1 vg,1 vg,2 2c 2 B 1 2 109 107 = 21 2 3 1010 2 105 107 0.1 s. (139)

(140)

(b) For the radio wave to be reected back downwards, there must be some height h such that (h) = 90 . According to Snell, ni sin i = n(h) sin (h), so with ni = 1, sin (h) = 1, and n(h) = 1 ( p (h)/ )2, we need (142) (141)

p (h) = cos i .

Princeton University 2001

Ph501 Set 6, Solution 9

36

9. Inside the good conductor the plane wave has the form 2 z i(zt) E = E0 e e , = , H= c

2 (1 + i) z E,

(143)

where is the conductivity and is the permeability. Hence, the time-averaged Poynting vector is c 2 c 2 z. (144) S = Re(E H ) = |E0| e2z 8 8 The power lost per unit area to Joule heating is P 1 |E 0 |2 Re( E E ) dz = 2 0 2 0 2 c |E0 | c 2 2 = = |E0| = S(z = 0) . 4 2 8 =

J E dz =

e2z dz =

|E 0 |2 4 (145)

Princeton University 2001

Ph501 Set 6, Solution 10

37

10. Ignoring reections, the incident power is either absorbed or transmitted, so recalling prob. 8 we can write Sout = Sin Joule heating = Sin c |E0|2 a |E0 |2 , 8 2 d. The relative transmitted intensity is (147)
a 0

J E dz =

c |E 0 |2 |E 0 |2 8 2

a 0

e2z/d dz (146)

where the approximation holds since a T =

Sout 4a = 1 . Sin c

To analyze the reected intensity, we rst consider the reected and transmitted amplitudes. In particular, we focus on the magnetic eld H, since its transverse component is continuous across a metallic boundary. Furthermore, since the thickness of the sheet is much less than the skin depth, the magnitude H is essentially unchanged from one side of the sheet to the other. That is, Hi + Hr Ht , (148)

where i, r and t indicate incident, reected and transmitted, respectively. We can deduce Ht from the transmitted intensity ratio T = |Ht |2 / |Hi |2 , 2a |Ht | = |Hi | T |Hi | 1 c Thus, from eq. (148) Hr and 2a Hi , c
2

(149)

(150)

2a |Hr |2 R= 2 c |Hi |

(151)

Princeton University 2001

Ph501 Set 6, Solution 11

38

11. According to Snells law, the angle 2 of the transmitted wave obeys the formal relation n1 sin 2 = sin 1 . (152) n2 For the stated conditions, sin 2 > 1. Then, cos 2 = 1 sin2 2 = i n2 1 sin2 1 1. 2 n2 (153)

We take the x axis to be normal to the 1-2 boundary, and the y axis along the boundary in plane of incidence.

Formally, the transmitted wave vector kt has components n2 n2 n1 2 2 cos 2 = i sin 2 = sin 1 . (154) n2 ky = kx = 1 sin 1 n2 i, c c c c The space-time dependence of the transmitted wave is therefore ei(kt rt) = ex ei(ky yt) , (155)

which describes a surface wave that propagates in the +y direction at phase velocity c/(n1 sin 1) < c, and whose amplitude is signicant only for x < 1/ . For incident electric eld perpendicular to the plane of incidence, the Fresnel equation (48) and eq. (155) tell us that Et = Et z= 2 sin 2 cos 1 E0i zex ei(ky yt). sin(1 + 2) 1 Bt = i Bt , c t c (156)

We can nd the magnetic eld via Faradays equation: Et = so that (157)

c Ez c Ez c i y Et . Bt = i x +i y = n1 sin 1 x y x

(158)

The electric eld (156) satises E = 0 since Ez does not depend on z . Similarly, the magnetic eld (158) satises B = 0, and hence Maxwells equations are satised. The wave equation is also obeyed by eq. (156) since
2 )Et = 2 Et = ( 2 ky 2 2 n2 n2 2 2 Et E = . t c2 c2 t2

(159)

Princeton University 2001

Ph501 Set 6, Solution 11

39

The time-averaged Poynting vector of the transmitted wave is St = Now, |sin(1 + 2 )|2 = = = = |sin 1 cos 2 + cos 1 sin 2|2 2 sin2 1 |cos 2| + cos2 1 sin2 2 sin2 1 (sin2 2 1) + (1 sin2 1 ) sin2 2 sin2 2 sin2 1 n2 = sin2 2 1 2 , n2 1 c c 4 sin2 2 cos2 1 . Re(Et Bt ) = |E0i |2 e2x n1 sin 1 y 8 8 |sin(1 + 2 )|2 (160)

(161)

so that St =

c n1 2 2x 2 . y 2 sin 1 cos 1 |E0i | e n 2 1 2 2


n1

(162)

As expected, the Poynting vector is parallel to the y axis, so no energy is transmitted into medium 2. However, the formal result (162) is that a non-negligible energy ows in the thin layer near the bounding surface of medium 2. For the case of the electric eld parallel to the plane of incidence, the Fresnel equation (53) is E0t (x = 0) 2 sin 2 cos 1 = (163) E0i sin 1 cos 1 + sin 2 cos 2 However, the transmitted electric eld cannot be only in the x direction, as this would not satisfy E = 0. There must be a y component as well, as we found for the magnetic eld when the electric eld was perpendicular to the plane of incidence. Hence, we expect that c Aex ei(ky yt) , i y Et = n1 sin 1 x where A is chosen to satisfy eq. (163) at x = 0. That is,
2 2 2 (2n2 = 1 sin 1 n2 ) |A|

(164)

4 sin2 2 cos2 1 2 2 |E0i | |sin 1 cos 1 + sin 2 cos 2 |


2 2 4 n1 2 sin 1 cos 1
2

n2

= =

sin 1 (1 sin 1 ) +
2 2

n2 1 n2 2

sin

n2 1 ( n1 2 2

sin 1 1)
2 2

|E0i |2 (165)

4 cos2 1 (1
n2 2 )((1 n2 1

n2 1 ) sin2 n2 2

1 1)

|E0i| .

Faradays law (157) gives us the magnetic eld as Bt = i c Etx Ety y x


x i(ky yt) . = n2 e z 2 Ae

(166)

Princeton University 2001

Ph501 Set 6, Solution 11

40

The time-averaged Poynting vector of the transmitted wave is St c c 2 2x y Re(Et Bt ) = n1 n2 2 sin 1 |A| e 8 8 |E0i |2 e2x y n1 sin 1 cos2 1 c . = 2 2 2 n n n 2 2 2 1 2 1+ 1 sin 1 1 2 1 2 2 2 sin 1 1 =
n1 n2 n2

(167)

Perhaps the most interesting feature of the surface wave for the case of polarization in the plane of incidence is that the electric eld (164) includes a component along the direction of propagation of the wave, and the wave velocity is less than c. A charged particle moving along with this wave would experience a continual force in the direction of motion, and would therefore be accelerated. A related concept for particle acceleration by surface waves will be explored in prob. 12.

Princeton University 2001

Ph501 Set 6, Solution 12

41

12. The interaction between particle beams and diraction gratings was rst considered by Smith and Purcell, Phys. Rev. 62, 1069 (1953),4 who emphasized energy transfer from the particle to free electromagnetic waves. The excitation of surface waves by particles near conducting structures was rst discussed by Pierce, J. Appl. Phys. 26, 627-638 (1955),5 which led to the extensive topic of wakeelds in particle accelerators. The presence of surface waves in the Smith-Purcell eect was noted by di Francia, Nuovo Cim. 16, 61-77 (1960).6 A detailed treatment of surface waves near a diraction grating was given by van den Berg, Appl. Sci. Res. 24, 261-293 (1971).7 Here, we construct a solution containing surface waves by starting with only free waves, then adding surface waves to satisfy the boundary condition at the grating surface. If the (perfectly) conducting surface were at, the reected wave would be ei(ky y+kz zt) . Er = E0x (168)

However, the sum Ein + Er does not satisfy the boundary condition that Etotal must be perpendicular to the wavy surface (30). Indeed, ei(ky yt) sin kz z 2iakz E0 x ei(ky yt) sin kx x, [Ein + Er]surface = 2iE0 x where the approximation holds for a d, and we have dened kx = 2/d. (169)

Hence, we require additional elds near the surface to cancel that given by (169). For z 0, these elds therefore have the form ei(ky yt) eikx x eikx x . E = akz E0 x This can be decomposed into two waves E given by ei(kx x+ky yt). E = akz E0 x (171) (170)

Away from the surface, we suppose that the z dependence of the additional waves can be described by including a factor eikz z . Then, the full form of the additional waves is ei(kx x+ky y+kz zt). E = akz E0 x (172)

The constant kz is determined on requiring that each of the additional waves satisfy the wave equation, 1 2 E 2 E = 2 . (173) c t2 This leads to the dispersion relation
2 2 kx + ky + kz2 =
4 5 6 7

2 . c2

(174)

http://puhep1.princeton.edu/~mcdonald/examples/accel/smith_pr_92_1069_53.pdf http://puhep1.princeton.edu/~mcdonald/examples/accel/pierce_jap_26_627_55.pdf
http://puhep1.princeton.edu/~mcdonald/examples/accel/toraldo_di_francia_nc_16_61_60.pdf

http://puhep1.princeton.edu/~mcdonald/examples/accel/vandenberg_asr_24_261_71.pdf

Princeton University 2001

Ph501 Set 6, Solution 12

42

The component ky of the incident wave vector can be written in terms of the angle of incidence in and the wavelength as ky = Combining (174) and (175), we have 1 cos in kz = 2i 2 d
2

2 sin in .

(175)

(176)

For short wavelengths, kz is real and positive, so the reected wave (168) is accompanied by two additional plane waves with direction cosines (kx , ky , kz ). But for long enough wavelengths, kz is imaginary, and the additional waves are exponentially attenuated in z. When surface waves are present, consider the elds along the line y = 0, z = /2kz . Here, the incident plus reected elds vanish (see the rst form of (169)), and the surface waves are ei(kx xt). (177) E = akz e |kz |/2kz E0 x The phase velocity of these waves is vp = d = c. kx (178)

When d = , the phase velocity is c, and kz = iky according to (176). The surface waves are then, 2a cos in (/2) tan in ei(kx xt) . E0 x (179) e E = d A relativistic charged particle that moves in, say, the +x direction remains in phase with the wave E+ , and can extract energy from that wave for phases near . On average, the particles energy is not aected by the counterpropagating wave E . In principle, signicant particle acceleration can be achieved via this technique. For a small angle of incidence, and with a/d = 1/2 , the accelerating eld strength is equal to that of the incident wave.

Princeton University 2001

Ph501 Set 6, Solution 13

43

13. This problem was abstracted from T.P. Wangler, Strong focusing and the radiofrequency quadrupole accelerator, Am. J. Phys. 64, 177 (1996).8 The electric eld in the RFQ can be obtained from the potential via E = , so Ex = Ey The equations of motion are x = x eE0 sin t, d m y eE0 y = sin t, d m z = 0. (182) (183) (184) x E0 sin t, d y = E0 sin t. d (180) (181)

Then, z (t ) = z 0 + v 0 z t = v 0 t for the particular case specied. For the x motion, we consider the form (33), x = f + g sin t + g cos t, + g x = f sin t + 2 g cos t 2g sin t. The x equation of motion now yields f + g sin t eE0 + 2 g f cos t = g + 2 g + sin t. d m (188) (186) (187) (185)

Since g is both small and slowly varying by hypothesis, we neglect the terms involving g and g , leaving 2g + f eE0 sin t + g eE0 sin2 t. (189) f d m d m should In this, the coecent of the rapidly varying term sin t should vanish, and f 2 be the average of the term in sin t. The rst condition tells us that g= eE0 f, m2 d (190)

which combines with the (averaged) second condtion to give a dierential equation for f: 2 = 1 eE0 f. (191) f 2 md
8

http://puhep1.princeton.edu/~mcdonald/examples/accel/wangler_ajp_64_177_96.pdf

Princeton University 2001 Thus,

Ph501 Set 6, Solution 13

44

f A cos t + B sin t, Together we have

where

eE0 . 2md

(192)

x(t) (A cos t + B sin t) 1

eE0 sin t . m2 d

(193)

The particular initial conditions (34)-(36) are satisifed by x (t ) eE0 v0 0 sin t 1 sin t . m 2d eE0 m 2 d (194)

For this to be consistent we must have that 1. (195)

Then, the beam returns to the z -axis at time t = /, corresponding to distance z = v0/. The argument is similar for the y motion. The opposite sign of the electric eld leads to eE0 g=+ f, (196) m 2 d and so eE0 sin t . (197) y (t) (C cos t + D sin t) 1 + m 2 d The particular initial conditions (34)-(36), however, require that both C and D vanish. Experts will recognize that the dimensionless quantity eE0 , mc (198)

where c is the speed of light, is a useful invariant of the eld. In terms of this invariant the condition of validity of the solution is 2d 1. (199)

If d is a characteristic aperture of the RFQ, we earlier required that d so the quasistatic approximation to the elds would be valid. Hence, the invariant eld strength cannot be too large in the RFQ. The physical meaning of the invariant is that it is the ratio of the energy gain over distance /2 to the electron rest energy mc2: = eE0 eE0/2 = . mc mc2 (200)

Thus, the RFQ should not impart relativistic transverse motion to the particles if it is to function as described above.

Princeton University 2001

Ph501 Set 6, Solution 14

45

14. a) We ignore the interstellar magnetic eld (see prob. 7 for a discussion of the eect of such elds), so the usual analysis of waves incident on free electrons applies: m r = eEei(kzt) , eE , m2 e2 E p = Ner = N = E, m 2 2 4Ne2 p = 1 + 4 = 1 = 1 , m 2 2 where the plasma frequency p is given by r= 2 p = 4Ne2 c2 4Ne2 = 4Nr0 c2 , = 2 m mc (201) (202) (203) (204)

(205)

where r0 = e2/mc2 = 2.8 1013 cm is the classical electron radius. Then, n( ) = = 1 2 p . 2 (206)

b) The propagation time for a wave of frequency over distance L is t ( ) = n( )L , c (207)

so the propagation-time dierence for frequencies separated by is t = L 2 dt p = . d cn 2 (208)

The higher frequency takes longer to arrive. For the example that = 2000 MHz, / = 0.01, N = 0.04 electrons/cm3, and |t| = 0.004 s, we nd that
2 13 (3 1010 )2 1.3 108 , 2 p = 4Nr0 c = 4 0.04 2.8 10

(209) (210)

n=

2 p = 2

1.3 108 1, 4 1018

and the distance to the pulsar is L = cnt 4 1018 2 10 1 0 . 004 100 3.7 1020 cm 400 light years. 3 10 8 2 1 . 3 10 p (211)

Princeton University

Ph501 Electrodynamics Problem Set 7


Kirk T. McDonald (2001) kirkmcd@princeton.edu http://puhep1.princeton.edu/~mcdonald/examples/

Princeton University 2001

Ph501 Set 7, Problem 1

1. Polarization Dependence of Emissivity Deduce the emissive power P of radiation of frequency into vacuum at angle to the normal to the surface of a good conductor at temperature T , for polarization both parallel and perpendicular to the plane of emission. Recall Kirchhos law of heat radiation (as claried by Planck, The Theory of Heat Radiation, chap. II, especially sec. 28) that P h 3/c2 = K (, T ) = h/kT , A e 1 (1)

where P is the emissive power per unit area per unit frequency interval (emissivity) and E0r 2 A = 1 R = 1 (2) E0i is the absorption coecient (0 A 1), c is the speed of light, h is Planks constant and k is Boltzmanns constant.

Princeton University 2001 2. Rayleigh Resistance

Ph501 Set 7, Problem 2

A circular wire of conductivity and radius a d, where d( ) is the skin depth, carries current that varies as I (t) = I0 eit . As in prob. 9, set 6, consider the timeaveraged Poynting vector at the surface of the wire. Relate this to the Joule loss I 2R to show that R( ) = a R0 , 2d where R0 = 1 a2 is the dc resistance per unit length. (3)

Princeton University 2001 3. Telegraphers Equation

Ph501 Set 7, Problem 3

Deduce the dierential equation for current (or voltage) in a two-conductor transmission line that is characterized by resistance R (summed over both conductors), inductance L, capacitance C and leakage conductivity K , all dened per unit length. The leakage conductivity describes the undesirable current that ows directly from one conductor to the other across the dielectric that separate them according to Ileakage = KV, (4)

where V (x, t) is the voltage between the two conductors, taken to be along the x axis.

Deduce a relation among R, L, C and K that permits distortionless waves of the form exf (x vt) (5)

to propagate along the transmission line. Give expressions for v and in terms of R, L and C ; relate to the transmission line impedance dened by Z = V/I in the limit that R and K vanish.

Princeton University 2001

Ph501 Set 7, Problem 4

4. Transmission Line Impedance a) Consider a two-wire transmission line with zero resistance (and zero leakage current) that lies along the z axis in vacuum. We dene the line impedance Z by V = ZI , where I (z, t) is the current in each of the wires (+I in one, I in the other), and V (z, t) is the voltage dierence between the two wires. Show that Z is real ( V and I are in phase), and that Z = L/C , where C and L are the capacitance and inductance per unit length. b) Impedance Matching A transmission line of impedance Z1 for z < 0 is connected to a line of impedance Z2 for z > 0. A wave Vi ei(k1 zt) is incident from z = . Show that the reected wave Vr ei(k1 z+t) (z < 0) and the transmitted wave Vt ei(k2 zt) (z > 0) obey Vr Z2 Z1 Vt 2Z2 = , and = . (6) Vi Z2 + Z1 Vi Z2 + Z1 Note the boundary conditions on I and Z at z = 0. Since Z1 and Z2 are real, the transmission line of impedance Z2 could be replaced by a pure resistance R = Z1 and no reection would occur. Even when line 2 is present, we can avoid a reection by a kind of antireection coating as discussed in prob. 5, set 6. Another way to deduce this is by consideration of the complex impedance Z (z ) = V (z )/I (z ) where V and I are the total voltage and total current. If the line for z < z0 < 0 were replaced by a source whose impedance is exactly Z (z0), then the waves for z > z0 would be unchanged. Show that Z (l) = Z1 Z2 iZ1 tan k1 l . Z1 iZ2 tan k1 l (7)

2 Also show that when l = 1 /4 then Z (l) = Z1 /Z2 , which is real. Hence, the source 2 at z = l could be a transmission line of impedance Z0 = Z1 /Z2 . That is, a quarter-wave section of impedance Z1 = Z0 Z2 matches lines of impedances Z0 and Z2 with no reections.

c) Impedance Matching with Resistors The quarter-wave matching of part b) works only at a single frequency. Impedance matching of transmission lines over a broad range of frequencies can be accomplished with appropriate resistors, as illustrated in the following examples. The key to an analysis is that a transmission line of impedance Z acts on an input signal like a pure resistance of R = Z which is connected to ground potential. Thus, a resistor of value R = Z1 Z2 matches signals that moves from a line of impedance Z1 into one of impedance Z2 < Z1 , as shown below

Princeton University 2001

Ph501 Set 7, Problem 4

However, if Z1 < Z2 , then a matching circuit (for signals moving from line 1 into line 2) can be based on a resistor R = Z1 Z2 /(Z2 Z1) the connects the junction to ground, as shown below.

It might be desirable to have a circuit that matches lines 1 and 2 no matter which direction the signals are propagating. This could be accomplished with resistors R1 = Z1 (Z1 Z2 ) and R2 = Z2 Z1 /(Z1 Z2 ) as shown below, assuming Z1 > Z2 .

Another type of matching problem involves lines of a single impedance Z , where it is desired to split the signal into two parts with ratio A between the currents and voltages. This could be accomplished with resistors R1 = Z/A and R2 = AZ as shown below.

In case of a 1:1 split, then R1 = R2 = Z . In this case, a reectionless split could also be accomplished with three identical resistors of value R = Z/3 arranged as in the gure below. If these resistors are mounted in a box with three terminals, the split is accomplished properly no matter how the lines are connected.

Princeton University 2001

Ph501 Set 7, Problem 5

5. In the manufacture of printed circuit boards it is common to construct transmission lines consisting of a wire separated from a ground plane by a layer of dielectric. Estimate the transmission line impedance for two typical congurations: a) Wire over Ground

The wire has diameter d, centered at height h d above the conducting ground plane. The other space above the ground plane is lled with a dielectric of constant (= 4.7 for G-10, a berglass-epoxy composite often used in circuit boards). Estimate the capacitance C per unit length, and from this show the transmission line impedance is 60 4h . (8) Z ln d b) Stripline

A conducting strip of width w is at height h above the conductor. It may be dicult to estimate the capacitance for w < h, so consider the case that w h to show that h Z 377 . (9) w

Princeton University 2001 6. O-Center Coaxial Cable

Ph501 Set 7, Problem 6

A coaxial transmission line has inner conductor of radius a and outer conductor of radius b, but the axes of these two cylinders are oset by a small distance b. Deduce the capacitance and inductance per unit length, and the impedance Z , accurate to order 2 /b2 . The (relative) dielectric constant and permeability of the medium between the two conductors both unity. The relevant frequencies and conductivities are so large that the skin depth is small compared to .

Princeton University 2001 7. Cavity Q

Ph501 Set 7, Problem 7

a) A rectangular cavity with conducting walls of length x = y = a, z = l is excited in the (1,1,0) mode: Ez = E0 sin y it x sin e , a a Ex = Ey + Hz = 0. (10)

Calculate the time-averaged force on each of the six faces. b) The cavity Q (quality factor) is dened by Q= Show that Q= stored energy . energy lost per cycle (11)

volume al , 2d(a + 2l) d surface area

(12)

where d is the skin depth. The approximate version of eq. (12) provides a reasonable estimate for the Q of the lowest mode of any cavity.

Princeton University 2001 8. Cavity Line Broadening

Ph501 Set 7, Problem 8

According to the denition of cavity Q given in prob. 4, 0 dU = U, dt 2Q (13)

where U is the averaged energy stored in the cavity elds, whose angular frequency is 0 . Thus, if the cavity were left to itself, the energy would die away: U (t) = U0e 0 t/2Q . (14)

Since the eld energy U is proportional to the square of the electric eld E , we have E (t) E0 e0 t/4Q ei0 t . This is not the behavior of a pure frequency 0 . Perform a Fourier analysis of the electric eld: E (t ) = E eit d, (16) (15)

supposing that the cavity is turned on at t = 0, which implies that E = Show that this leads to U |E |2 ( 0 )2 1 , + ( 0 /4Q)2 (18) 1 2
0

E (t)eit dt.

(17)

which has the form of a resonance curve. The damping due to resistive wall losses gives a nite width to the resonance: FWHM = 0 /2Q. Thus, the relation Q= 0 2 (19)

gives additional meaning to the concept of cavity Q. That is, the Q is a measure of the sharpness of the cavity frequency spectrum.

Princeton University 2001 9. Rayleigh-Jeans Law

Ph501 Set 7, Problem 9

10

One of the most signicant uses of a cavity was the measurement of the spectrum of the waves inside when the walls were red hot. Maxwell told us that if thermal equilibrium holds, each cavity mode carries energy kT (considering a mode as a kind of oscillator with two polarizations). Here, T is the temperature and k is Boltzmanns constant. Show that the number of modes per unit interval of angular frequency in a cubical cavity of edge a is a3 2d dN = , (20) 2 c3 for frequencies such that the mode indices l, m, n are all large compared to one. The famous hint is that each mode (l, m, n) corresponds to a point on a cubical lattice. Then, the energy spectrum of the cavity radiation would be dE = 8a3 2 kT d a3 2kT d = , 2 c3 c3 (21)

where = /2 is the ordinary frequency. The Rayleigh-Jeans expression (21) implies that the total energy of the cavity radiation grows arbitrarily large as one include the contributions at high frequency. Such behavior was, of course, not observed in the laboratory. Planck saw that this requires a rather fundamental change in our thinking...

Princeton University 2001 10. Right Circular Cavity

Ph501 Set 7, Problem 10

11

A simple mode of a right circular cavity is sketched below:

We expect that the electric eld of angular frequency has the form E= zE (r)eit . Plugging in to the wave equation 2 E = we see that 1 2E , c2 t (23) (22)

1 rE 2 + 2 E = 0, r r r c which we recognize as Bessels equation of order zero, r . E J0 c

(24)

(25)

Alternatively, ignore the cylindrical walls initially, and simply consider the cavity to be a parallel plate capacitor. This suggests that the time-dependent electric eld of angular frequency has the form Ez = E0 eit . (26) Show, however, that the time dependence of the displacement current induces an azimuthal magnetic eld ir E0 eit . H = (27) 2c Than, Faradays law tells us that a correction to E is induced by the time variation of H... Follow this logic enough to demonstrate the rst and second corrections to Ez , which form the rst terms of the series (1)n r 2n r it Ez = E0eit = E0J0 e . (28) 2 2c c n=0 (n!) J0 (x) oscillates, with zeros at x = 2.405, 5.520, 8.654, .... Hence, the boundary condition that Ez = 0 at r = a is satised if a = 2.405, 5.520, 8.654, ..., (29) c which describes an important class of modes of a right circular cavity.

Princeton University 2001

Ph501 Set 7, Problem 11

12

11. RF Cavity with Fields that vary Linearly with Radius A simple rf cavity is a right circular cylinder of radius a and length d (see the preceding problem), for which the TM0,1,0 mode has electromagnetic elds Ez (r, , z, t) = E0 J0(kr) cos t, B (r, , z, t) = E0 J1(kr) sin t, where ka = 2.405 is the rst zero of the Bessel function J0. Such a cavity is potentially interesting for particle acceleration in that the electric eld points only along the axis and is independent of z , so that a large fraction of the maximal energy eEd could be imparted to a particle of charge e as it traverses the cavity. However, such cavities are not useful in practice for at least two reasons: the particles must pass through the cavity wall to enter or exit the cavity and thereby suer undesirable scattering; the magnetic eld does not vary linearly with radius, and so acts like a nonlinear lens for particles whose motion is not exactly parallel to the axis. Practical accelerating cavities have apertures (irises) of radius b in the entrance and exit surfaces, so that a beam of particles can pass through without encountering any material. In this case, the electric eld can no longer be purely axial. Deduce the simplest electromagnetic mode of a cavity with apertures for which the transverse components of the electric and magnetic elds vary linearly with radius. Deduce also the shape of the wall of a perfectly conducting cavity that could support this mode. Consider a cavity of extent d < z < d, with azimuthal symmetry and symmetry about the plane z = 0, that could be a unit cell of a repetitive structure. This implies that either Ez = 0 at (r, z ) = (0, d) and (0, d), or Ez /z = 0 at these points. (30) (31)

Princeton University 2001 12. Reex Klystron

Ph501 Set 7, Problem 12

13

The gure below shows a slice through a kind of cylindrical cavity used in generation of high strength radio frequency elds, the so-called reex klystron.

This is something like a piece of (vacuum) coaxial cable of length h, inner radius a and outer radius b terminated with a conducting plate on the right, but with a small gap d between the termination plate and the center conductor on the left. An electron beam is made to pass along the axis of the cavity through small holes. The beam is modulated at the cavity resonant frequency, and transforms its energy to the cavity eld if it crosses the gap 180 out of phase with the cavity eld. Estimate the lowest resonant frequency of the cavity.

Princeton University 2001 13. Guide Loss

Ph501 Set 7, Problem 13

14

Consider propagation of waves in the lowest TE mode of a rectangular waveguide with edges a < b, as shown in the gure below. The walls have conductivity and the interior of the guide is at vacuum.

Due to Joule losses in the walls, the intensity of the propagating eld dies out like ez where power loss per unit length along guide = . (32) power transmitted down the guide For waves of wavelength show that 1 + 2ba 1 c 4 = 4 a c 1
2b 2b 2 2

(33)

which could be minimized to nd the best choice for .

Princeton University 2001 14. Loop Coupling

Ph501 Set 7, Problem 14

15

A common way of feeding waves into a guide is shown in the gure below. The center conductor of a coaxial cable is bent into a semicircle of radius r and grounded on the guide wall. Then, for waves with r it is a good approximation that the current it I0 e is constant over the loop.

a) Explain briey why essentially no power is radiated into a TM mode by this coupler. b) If r b < a, where a and b are the lengths of the edges of the guide, show that the power radiated into the lowest TE mode is P = 4 2 k a r I c 0 kg b 2a
4

(34)

in each direction, independent of the position h of the coupling loop.

Princeton University 2001

Ph501 Set 7, Solution 1 Solutions

16

1. This solution is adapted from Born and Wolf, Principles of Optics, 7th ed., sec. 14.2. (see also Landau and Lifshitz, The Electrodynamics of Continuous Media, sec. 67), and nds application in the calibration of the polarization dependence of detectors for cosmic microwave background radiation (E.J. Wollack, Princeton Ph.D. dissertation, 1994, Appendix C.1.1; C. Herzog, Princeton U. Generals Expt., 1999). In eq. (2) we need the Fresnel equations of reection that E0r E0i =

sin(t i ) , sin(t + i )

E0r E0i

tan(t i ) , tan(t + i )

(35)

where i, r, and t label the incident, reected, and transmitted waves, respectively. The solution is based on the fact that eq. (1) holds separately for each polarization of the emitted radiation, and is also independent of the angle of the radiation. This result is implicit in Plancks derivation of Kirchhos law of radiation, and is stated explicitly in Reif, Fundamentals of statistical and thermal physics, sec. 9.14. That law describes the thermodynamic equilibrium of radiation emitted and absorbed throughout a volume. The emissivity Pv and the absorption coecient A can depend on the polarization of the radiation and on the angle of the radiation, but the denitions of polarization parallel and perpendicular to a plane of emission, and of angle relative to the normal to a surface element, are local, while the energy conservation relation P = A K (, T ) is global. A ray of radiation whose polarization can be described as parallel to the plane of emission is, in general, a mixture of parallel and perpendicular polarization from the point of view of the absorption process. Similarly, the angles of emission and absorption of a ray are dierent in general. Thus, the concepts of parallel and perpendicular polarization and of the angle of the radiation are not well dened after integrating over the entire volume. Thermodynamic equilibrium can exist only if a single spectral intensity function K (, T ) holds independent of polarization and of angle. All that remains is to evaluate the reection coecients R and R for the two polarizations at a vacuum-metal interface. These are well known, but we derive them for completeness. To use the Fresnel equations (35), we need expressions for sin t and cos t . The boundary condition that the phase of the wave be continuous across the vacuum-metal interface leads, as is well known, to the general form of Snells law: ki sin i = kt sin t , where k = 2/ is the wave number. Then, cos t = 1
2 ki sin2 i . 2 kt

(36)

(37)

Princeton University 2001

Ph501 Set 7, Solution 1

17

To determine the relation between wave numbers ki and kt in vacuum and in the conductor, we consider a plane wave of angular frequency = 2 and complex wave vector k, E = E0 ei(ktrt), (38) which propagates in a conducting medium with dielectric constant , permeability , and conductivity . The wave equation for the electric eld in such a medium is 2 E 2E 4 E , = 2 2 c t c2 t (39)

where c is the speed of light. We nd the dispersion relation for the wave vector kt on inserting eq. (38) in eq. (39):
2 = kt

2 4 +i . 2 c c2

(40)

For a good conductor, the second term of eq. (40) is much larger than the rst, so we write 1+i 2 2 kt (1 + i) = = , (41) c d d(1 i) where d= c 2 (42) = 1 = and = 0 we

is the frequency-dependent skin depth. Of course, on setting obtain expressions that hold in vacuum, where ki = /c.

We see that for a good conductor |kt | ki , so according to eq. (37) we may take cos t 1 to rst order of accuracy in the small ratio d/. Then the rst of the Fresnel equations becomes E0r E0i =

(d/)(1 i) cos i 1 cos i sin t / sin i 1 (ki /kt ) cos i 1 = , (43) cos i sin t / sin i + 1 (ki /kt ) cos i + 1 (d/)(1 i) cos i + 1

and the reection coecient is approximated by R = E0r E0i


2

4d cos i = 1 2 cos i .

(44)

For the other polarization, we see that E0r E0i so that R R 1 E0r = E0i cos(i + t) E0r cos(i t ) E0i 4d sin2 i cos i cos i (d/)(1 i) sin2 i , cos i + (d/)(1 i) sin2 i 4d 2 =1 cos i cos i . (45)

(46)

Princeton University 2001

Ph501 Set 7, Solution 1

18

An expression for R valid to second order in d/ has been given in Stratton, Electromagnetic Theory, sec. 9.9. For i near 90 , R 1, but eq. (46) for R is not accurate. 1, eq. (45) becomes Writing i = /2 i with i E0r E0i For i = /2, R = 1, and R
,min

i (d/)(1 i) , i + (d/)(1 i) 2)/(5 + 2) = 0.58 for i = 2 2d/.

(47)

= (5

Finally, combining eqs. (1), (2), (44) and (46) we have P and 4d cos h , 3 eh/kT 1 P 4d h , cos eh/kT 1
3

(48)

P = cos2 P d/ .

(49)

for the emissivities at angle such that cos

The conductivity that appears in eq. (48) can be taken as the dc conductivity so long as the wavelength exceeds 10 m. If in addition h kT , then eq. (48) can be written 4d kT cos 4d kT , P 3 P , (50) 3 cos in terms of the skin depth d.

Princeton University 2001

Ph501 Set 7, Solution 2

19

2. Inside a conductor the magnetic eld at angular frequency is related to the electric eld by c E, (1 + i)k (51) H= d where c is the speed of light, is the permeability, and d = c/ 2 is the skin depth. Hence, the time-averaged Poynting vector is S = c2 , |E |2 k 8d (52)

is very nearly normal to the where for waves just inside the surface of the conductor k surface. Since d, the electric eld is very small inside the conductor, and Amperes law applied to a loop of radius a gives 2aH0 or 4 I, c (53)

c 2I E0 . H0 = (1 + i)k ac d 2d |I | |E0 | , ac2 S0

(54)

Thus,

(55)

and eq. (52) gives d . |I |2 k (56) 4a2c2 In terms of the eective resistance R per unit length of the wire, the average power dissipated per unit length is d 2 1 2 |I | R = 2a S0 |I | . 2 2ac2 Hence, we identify the eective resistance at frequency as d 2 1 a 1 a R = = = = R0 , ac2 ac 2 2 2ad 2d a2 2d (57)

(58)

where R0 = 1/a2 is the dc resistance (per unit length) of the wire to longitudinal currents.

Princeton University 2001

Ph501 Set 7, Solution 3

20

3. Referring to the sketch, Kirchos rule for the circuit of length dz shown by dashed lines tells us V (x) I (Rdx) V (x + dx) (Ldx) I = 0, t or V I =L + IR. x t (59)

Next, the charge dQ that accumulates on length dx of the upper wire during time dt is (Cdx)dV in terms of the change of voltage dV between the wires, which also can be written in terms of currents as Q = (Cdx)dV = (I (x) I (x + dx) Ileakage)dt, so I V =C + KV. (60) x t

Together these imply the desired wave equation 2I I 2I + KRI. = LC + ( RC + KL ) x2 t2 t We seek solutions of the form I = ex f (x vt), for which I = vex f , t 2I = v 2ex f , t2 (62) (61)

and

(63)

while I = ex f + ex f , x so 2I = 2 ex f 2ex f + ex f . x2 (64)

Inserting these into the wave equation we nd 2 f 2f + f = v 2LCf v (RC + KL)f + KRf. (65)

Princeton University 2001

Ph501 Set 7, Solution 3

21

This should be true for an arbitrary function f , so the coecients of each derivative of f must separately be equal: v= 1 , LC = KR, and 2 = RC + KL = 2 RCKL, v (66)

where we have used the rst two relations in obtaining the second form of the third. In general, a + b = 2 ab; this only holds when a = b. So we deduce the desired condition RC = KL, for distortionless telegraphy. With this condition, we can re-express as =R C . L (68) (67)

Finally, we relate this to the impedance Z = V/I when R = 0 = K . For this, we suppose that V = V0 f (x vt) and I = I0f , where V0 and I0 can be related by either of the rst-order dierential equations above. We quickly nd that V0 = vLI0, so Z = L/C . Then, R (69) = , Z once we have arranged that RC = KL. Remark: This problem was solved by O. Heaviside (1887) who argued that longdistance telegraph lines (including trans-Atlantic cables) should be designed to be distortionless. Previous cables were fairly far from this ideal. However, long cables are expensive so there was considerable hesitation to abandon the large existing capital investment and implement the proposed improvements. Indeed, the editor of the journal that published Heavisides papers was red for being too sympathetic to Heavisides views that were initially quite unpopular with industry. Heaviside, who was unemployed for most of his life, could not be red! Large-scale implementation of distortionless telegraphy occured only after 1900 following vigorous advocacy by M. Pupin of the U.S.A., for whom the physics building of Columbia U. is named. A typical cable has RC KL. It costs a lot to reduce RC , although this was the direction of industry prior to Heaviside. He noted that one shouldnt even try to reduce leakage K , so long as the signal is not attenuated until it is undetectable and the distortion-free condition makes it much easier to detect small signals. Rather one should increase the inductance, or leakage, or both! This counterintuitive result did not sit well with industry leaders, who, needless to say, were little guided by partial dierential equations.

Princeton University 2001

Ph501 Set 7, Solution 4

22

4. a) For a two-wire transmission line with negligible resistance, the voltage V between the wires is related to the current I in each of the wires by I V =L , z t which follows from Kirchhos law applied to a short length of the line, and 1 I V = , t C z (71) (70)

which follows from charge conservation, where C and L are the capacitance and inductance per unit length. For a wave of frequency moving in the +z direction, the waveforms are and I = I0 ei(kzt) . (72) V = V0 ei(kzt) , Substituting these forms into eqs. (70)-(71) we nd kV = LI, and hence, L V = Z. (74) I C Since C and L are real numbers, the impedance Z is real in this case. This implies that V0 /I0 is also real, and so the current and voltage are in phase. b) When a transmission line of impedance Z1 that occupies z < 0 is connected to a line of impedance Z2 for z > 0, the current and voltage will be continuous at the boundary z = 0. An incident wave of frequency from a source at z = has current Ii ei(k1 zt) . This results in a reected wave Ir ei(k1 z+t) for z > 0 and a transmitted wave Itei(k2 zt) for z > 0. Of course, Vi = Z1 Ii , Vr = Z1 Ir , and Vt = Z2 It. Continuity of the current at z = 0 tells us that Ii Ir = It , and hence Vi Vr = Z1 Vt , Z2 (75) and V = kI , C (73)

since a positive value for Ir corresponds to current owing in the z direction. Continuity of the voltage at z = 0 tells us that Vi + Vr = Vt . Equations (75) and (76) yield Vt = 2Z2 Vi , Z2 + Z1 and Vr = Z2 Z1 Vi . Z2 + Z1 (77) (76)

Princeton University 2001

Ph501 Set 7, Solution 4

23

At position z = l the total voltage is V (l) = Vi ei(k1 lt) + Vr ei(k1 l+t) = Vi eit eik1 l + Vi eit (Z2 + Z1 )eik1 l + (Z2 Z1 )eik1 l Z2 + Z1 Vi eit = Z2 (eik1 l + eik1 l ) Z1 (eik1 l eik1 l ) Z2 + Z1 2Vi eit = (Z2 cos k1 l iZ1 sin k1 l) , Z2 + Z1 = and (again noting that positive Ir implies a negative current) the total current is I (l) = Ii ei(k1 lt) Ir ei(k1 l+t) = = Vi it ik1 l Z2 Z1 ik1 l e e e Z1 Z2 + Z1 Z2 Z1 ik1 l e Z2 + Z1

(78)

Vi eit (Z2 + Z1 )eik1 l (Z2 Z1 )eik1 l Z1 (Z2 + Z1 ) Vi eit = Z1 (eik1 l + eik1 l ) Z2 (eik1 l eik1 l ) Z1 (Z2 + Z1 ) 2Vi eit = (Z1 cos k1 l iZ2 sin k1 l) , Z1 (Z2 + Z1 ) Hence, we nd the impedance Z ( l ) = Z2 cos k1 l iZ1 sin k1 l Z2 iZ1 tan k1 l V ( l ) = Z1 = Z1 . I ( l ) Z1 cos k1 l iZ2 sin k1 l Z1 iZ2 tan k1 l

(79)

(80)

2 /Z2 is a real number, which When l = 1 /4 = /2k1 , then tan k1 l and Z (l) = Z1 permits the region /2k1 < z < 0 of impedance Z1 to be an antireection matching 2 /Z2 and one of impedance Z2 . section between a line of impedance Z0 = Z1

c) A signal propagates in line 1 without reection is that line is terminated in (real) impedance Z1 . Thus, for the circuit

We need Z1 = R + Z2 , since the resistor is in series with line 2 (as viewed from line 1). A proper match is possible so long as Z1 > Z2 . The currents are the same in lines 1 and 2, so the transmitted voltage is V2 = V1 Z2 /Z1 . However, this is smaller than the transmitted voltage (77) for an unmatched line, because the matching resistor dissipates more power than is lost to the reection at an unmatched junction. If Z1 < Z2 , then a proper match to line 1 can be obtained with a resistor R in parallel with line 2, as shown below,

Princeton University 2001

Ph501 Set 7, Solution 4

24

provided the eective resistance of R and Z2 is again Z1 , i.e., 1 1 1 = + Z1 R Z2 R= Z1 Z2 . Z2 Z1 (81)

Here, the transmitted voltage is the same as that in line 1, but the transmitted current is less: I2 = I1Z1 /Z2 . To match lines 1 and 2 for signals moving in either direction, a combination of the above two circuits can be used, as shown below for the case that Z1 > Z2 :

Signals emanating from line 1 must be terminated in impedance Z1 ; hence, Z1 = R1 + R2 Z2 . R2 + Z2 (82)

Likewise, signals from line 2 must be terminated in impedance Z2 ; hence, Z2 = From eq. (82) we have Z1 Z2 = R1 R2 + R1Z2 R2 (Z1 Z2 ), while eq. (83) gives Z1 Z2 = R1 R2 R1 Z2 + R2 (Z1 Z2 ), Adding eqs. (84) and (84) we nd Z1 Z2 = R1 R2 , while subtracting gives R1 = R2 Solving these, we nd R1 = Z1 (Z1 Z2 ), and R2 = Z2 Z1 . Z1 Z2 (88) Z1 Z2 . Z2 (86) (85) (84) R2 (R1 + Z1 ) . R1 + R2 + Z1 (83)

(87)

For signals emanating from line 1, the transmitted voltage is V2 = V1 1 Z1 Z2 , Z1 (89)

Princeton University 2001

Ph501 Set 7, Solution 4

25

while for signals emanating from line 2, the transmitted voltage is V1 = V2 Z1 1 Z2 Z1 Z2 . Z1 (90)

In both cases, the transmitted voltage is less than the incident. For a reectionless split of the signal in a line of impedance Z we case use the circuit:

where the desired ratio of currents (and voltages) is A= Hence, R2 = AR1 + (A 1)Z. (92) Also, the two output lines must combine to terminate the input line in impedance Z , which tells us that (R1 + Z )(R2 + Z ) . (93) Z= R1 + R2 + 2Z Substituting eq. (92) in (93) we nd R1 = AZ, and R2 = Z . A (94) I1 R1 + Z = . I2 R2 + Z (91)

For a 50/50 split, A = 1 and R1 = R2 = Z . If we perform this split with three identical resistors in the symmetric conguration

the matching condition is Z =R+ and hence, R= Z . 3 (96) R+Z , 2 (95)

Princeton University 2001

Ph501 Set 7, Solution 5

26

5. As shown on p. 156 of the Notes, the inductance and capacitance per unit length of any two-conductor transmission line are related by LC = c2 , (97)

where c is the speed of light in vacuum and the medium outside the conductors is lled with a dielectric of constant . [The permeability is taken to be unity, and the frequency of the waves of interest is high enough that the skin depth is small compared to the transverse size of the conductors.] Thus, the impedance of the transmission line can be expressed as 30 L = = . (98) Z= C cC C a) Wire over Ground The capacitance of the wire over ground is twice the capacitance of the wire plus its image wire (since C = Q/V and V in the present example is 1/2 that for the case of two wires):

Recall prob. 11b, Set 3 to obtain the exact solution: C= 2 ln


2h+ 4h2 d2 d

h 2 ln 4d

(99)

Here, the presence of the dielectric medium leads to the factor in the numerator. Using the approximate form in eq. (99), which holds for d h, we nd 60 4h Z ln . d (100)

The approximate result can be obtained quickly as follows. When d h, the charge on the wire is distributed nearly uniformly, so we may use the result that the potential for a uniformly charged wire embedded in a medium of dielectric constant varies as V (r) = 2(Q/ ) ln r. Thus, the potential dierence between the wire of radius d/2 and its image are distance 2h due to charge Q per unit length on the wire is V 2(Q/ ) ln[2h/(d/2)]. In evaluating the capacitance, we suppose that the image wire

Princeton University 2001

Ph501 Set 7, Solution 5

27

has charge Q per unit length, which doubles the potential dierence between the two wires. Finally, we recall that the voltage dierence between the wire and the ground plane is 1/2 that between the wire and its image. Hence, V = 2(Q/ ) ln 4h Q = , d C (101)

which leads to the approximate result of eq. (99). b) Stripline If the strip width w is large compared to height h, then the capacitance per unit length is roughly w , (102) C 4h as follows from Gauss law, D = 4free, and the stripline impedance is Z 120 h 377 h = . w w (103)

In practical circuit boards, w h, and we expect the impedance to be between the estimates (103) and (100). If we use the exact form of eq. (99) and take d = h = w, we nd = , (104) C= 2.6 2 ln(2 + 3) and we estimate a lower bound on the impedance to be 80 Z . (105)

There does not appear to be a closed-form analytic solution to the present problem, but many numerical algorithms exist. See, for example, http://www.ideaconsulting.com/strip.htm This program estimates the impedance of a stripline with h = w embedded in a thick dielectric medium to be 110 (106) Z .

Princeton University 2001

Ph501 Set 7, Solution 6

28 L/C to MKSA units by

6. We use Gaussian units, and convert the impedance Z = noting that 1/c = 30, where c is the speed of light.

We dont need to calculate both the capacitance C per unit length and the inductance L per unit length, since in the case of a (perfectly conducting) transmission line they are related by (107) LC = 2 , c where the dielectric constant and the permeability are unity in the present case. The assumed smallness of the skin depth permits us to approximate the present transmission line as perfectly conducting. We rst present two calculations of the capacitance (secs. a and b), and then a calculation of the inductance (sec. c) as illustrations of various possible techniques. a) The Capacitance Via the Image Method It is expedient to use the image method for 2-dimensional cylindrical geometries. Recall that in the case of a wire of charge q per unit length at distance b from a ground conducting cylinder of radius a, as shown in the gure, one can think of an image wire of charge q at radius a2/b.

To apply this to the present problem, sketched in the gure below, note that the image wires of charge q per unit length are both located to the left of the center of the inner conductor, say at distances ra and rb .

For the inner cylinder to be an equipotential, we must have rb = a2 , rb b2 , ra + (108)

and the outer cylinder is also an equipotential provided rb + = (109)

Princeton University 2001

Ph501 Set 7, Solution 6

29

noting the oset by between the inner and outer cylinder. Combining eqs. (108) and (109), and noting that ra 0 as 0, we nd ra = b2 a 2 2 (b2 a2 2)2 4a2 2 . 2 (110)

The capacitance is related by C = q/V , where V = Vb Va is the potential dierence between the two cylinders. Recall that the potential at distance r from a wire of charge q per unit length is 2q ln r + constant. We evaluate the potentials at the points where the cylinders are closest to one another: Va = 2q ln(a ra) 2q ln(rb a) = 2q ln using eq. (108), and Vb = 2q ln(b ra ) 2q ln(rb b + ) = 2q ln using eq. (109). Then, V = 2q ln a 1+ b ra . (113) b2 /(r ra + b ra = 2q ln , (112) b a + ) b a2 /r a ra ra = 2q ln , a aa (111)

When combined with eq. (110), this is an exact solution for any < b a. In particular, as b a, then ra a, and the cylinders touch with the result that V = 0. Here, we suppose that b2 a 2 2 + = ra so that b a, and expand /ra to second order: (b2 a2 2 )2 4a2 2 b2 a 2 b2 2 , 2 2 2a2 a2 a (b a 2 ) b2 2 2 1 2 . 1+ ra a b a2 1 q b V 2 ln a (114)

(115)

The capacitance per unit length is therefore, C= using eq. (113). The inductance per unit length now follows from eq. (107): 2 b 2 L = 2 ln 2 , c a b a2 and the impedance is Z= 2 2 b L ln 2 C c a b a2 = 60 ln 2 b 2 a b a2 . (118) (117)
2

(116)

b2 a2

Princeton University 2001

Ph501 Set 7, Solution 6

30

Remark: The exact expression (113) is often written in a dierent fashion, which is convenient for large , but perhaps less useful for small . The exact version of (114) leads to b2 + a2 2 + (b2 + a2 2 )2 4a2 b2 1+ = , (119) ra 2a2 which in turn leads to C= q = V
b2 +a2 2 +

2 ln

(b2 +a2 2 )2 4a2 b2 2ab

1
2 b2 2 2 cosh1 a + 2ab

(120)

b) Capacitance Via Series Expansion of the Potential The image method can be deduced by an application of series expansion techniques for the electrostatic potential. In this section, we explore a direct use of such techniques. A full solution is long, and when we leave o some steps at the end, we get an answer that is not quite correct. We dene the electrostatic potential to be zero on the inner conductor, (r = a) = 0, (121)

and V on the outer conductor whose surface is approximately given by r = b + cos , (r = b + cos ) = V. The potential is symmetric about = 0: ( ) = ( ), so terms in sin n cannot appear in the series expansion of the potential: (r, ) = A0 ln r +
n=1

(122)

(123)

An r n +

Bn cos n. rn

(124)

The capacitance C per unit length is, of course, given by C = Q/V , where the charge Q per unit length on the inner conductor is given by
2 2

Q = 2a
0

() d = 2a
0

Er (a, ) a d = 4 2 A0 . 2V

2 0

(a, ) A0 d = . r 2

(125)

Thus, C= (126)

Applying the boundary condition (121) to the general form (124), we have 0 = A0 ln a +
n=1

An an +

Bn cos n. an

(127)

Princeton University 2001

Ph501 Set 7, Solution 6

31

Likewise, the boundary condition (122) yields V = A0 ln(b + cos ) +


n=1

An (b + cos )n +

Bn cos n. (b + cos )n

(128)

With considerable eort, the terms in eq. (128) of the form cosl cos m can be expressed as sums of terms in the orthogonal set of functions cos n. Then, eqs. (127) and (128) can be combined to yield the Fourier coecients An and Bn . Thus, subtracting eq. (127) from (128) and using the approximation (140), we have V = A0 ln b cos 2 cos2 + a b 2b2 + F (An, Bn , ) (129)

IF the integral of F with respect to vanished, then integrating eq. (129) yields V = A0 ln and the capacitance would be C= A0 1 b 2V 2 ln a
2 4b2

2 b 2 , a 4b

(130)

(131)

However, we the presence of terms like A1 cos2 in F means that we cannot expect its integral to vanish, and eq. (131) is not quite correct. c) Calculation of the Inductance The calculation of the inductance is complicated by the fact that the currents in this problem are distributed over surfaces, rather than owing in lamentary wires. We would like to use the relation, = cLI, (132) where I is the total (steady) current owing down the inner conductor (and back up the outer conductor), and is the magnetic ux per unit length linked by the circuit. From Amperes law, with the assumption that the currents are uniformly distributed on the inner and outer conductors, the azimuthal component B of the magnetic eld in the region between the two conductors is given by B (r) = 2I . cr (133)

If the cable were truly coaxial, the ux would be simply


b

0 =

B dr =

2I b ln , c a

(134)

and the corresponding inductance would be L0 = 2 b . ln c2 a (135)

Princeton University 2001

Ph501 Set 7, Solution 6

32

Then, from eq. (107) the capacitance would be C0 = 1 , 2 ln(b/a) (136)

as is readily veried by an electrostatic analysis, and the transmission line impedance would be L0 b 2 b (137) = ln = 60 ln . Z0 = C0 c a a However, because the outer conductor is o center with respect to the inner, we cannot simply use eq. (134). We can segment the currents on the conductors into laments of azimuthal extent d, and calculate the ux () linked the circuit element dened by the segments centered on angle on the inner and outer conductors. Then, the eective inductance of the whole cable can be estimated from eq. (132) using the average of (): rmax () d, a 0 0 a 0 (138) using (133) and (134). The result holds only to the extent that the current distribution is independent of azimuth, as discussed in sec. d. However, there will be a small azimuthal dependence to the current in this problem, so we will not obtain a completely correct result. L= 1 2cI () d = 1 2cI d B (r) dr = 1 c2 ln
2 2 rmax () 2

To complete the analysis, we need rmax (), the maximum radius about the center of the inner conductor of magnetic eld lines that are linked by the segment of the outer conductor at azimuthal angle . Assuming the currents is uniformly distributed over the inner and outer conductors, the magnetic eld between the two conductors is entirely due to the current in the inner conductor, and the eld is purely azimuthal about the axis of the inner conductor as given by eq. (133). Then, the geometry shown in the gure tells us that (139) rmax () = b + cos .

Princeton University 2001

Ph501 Set 7, Solution 6

33

This relation is exact to the extent that the currents are uniformly distributed; however, this is not actually the case in the present problem. To use relation (139) in eq. (138), we approximate ln rmax () cos b + cos b = ln = ln + ln 1 + a a a b b cos 2 cos2 ln + , (140) a b 2b2

which leads to L

2 b 2 ln . c2 a 4b2

(141)

This result happens to agree with the result implied by sec. b, but diers somewhat from the more accurate result of sec. a. d) The Magnetic Flux Linked by a Distributed Circuit The magnetic ux through a lamentary circuit (one in which the conductors are idealized as wires) is well dened as = B dS, (142)

where the integral is taken over any surface bounded by the circuit. However, when the conductors of the circuit are distributed and have a nite cross sectional area A, then eq. (142) is not well dened. We wish to show that a consistent denition of the ux through a distributed circuit is obtained by segmenting the conductors into a large number of circuits each with very small cross sectional area Ai, and dening = 1 A Ai i ,
i

(143)

where the magnetic ux through subcircuit i is given by eq. (142). We are interested in a denition of ux that gives consistency to the relation (132) in the context of circuit analysis. In particular, if the circuit has total resistance R, and the magnetic ux is changing, then we desire Faradays law to be written as IR = E = 1 d , c dt (144)

which is the same form as holds for each of the lamentary subcircuits: Ii Ri = Ei = 1 di . c dt (145)

We suppose that the current owing in subcircuit i is related to the total current according to Ai Ii = I, (146) A

Princeton University 2001

Ph501 Set 7, Solution 6

34

in which case the resistance of subcircuit i is given by Ri = Then, we can combine eqs. (145)-(147) as I=
i

A R. Ai

(147)

Ii =

1 c

1 1 di = Ri dt cRA

Ai
i

di . dt

(148)

Hence, the denition (143) leads to the desired relation (144) for the distributed circuit.

Princeton University 2001

Ph501 Set 7, Solution 7

35

7. a) The force on the cavity walls can be evaluated via the Maxwell stress tensor. Recall that for a good conductor the electric eld is perpendicular to a conducting surface, and the magnetic eld is parallel. Also, the Maxwell stress associated with a perpendicular eld E is +E 2/8 , while that with a parallel eld H is H 2 /8 . That is, the total, time-averaged force on a face of the cavity is given by 1 F = 2 |E | |H | dArea. 8 face
2 2

(149)

A positive value of F corresponds to an inward force. The electromagnetic elds of the (1,1,0) mode are Ex = 0, Ey = 0, Ez = E0 sin Hx Hy Hz using eq. (10), Faradays law E = and the wave equation 2 E = which latter tells us that 2 1 2H , c2 t2 (157) 1 H , c t (156) y it x sin e , a a y it x E0 cos e = i sin , a a 2 E0 y it x sin e = i cos , a a 2 = 0, (150) (151) (152) (153) (154) (155)

2 2 = . a2 c2

(158)

The force on each of the four faces perpendicular to the x or y axes is the same by the symmetry of the problem, and can be calculated using the face at, say, x = 0 to be F = This force is outwards. Likewise, the force on the two faces perpendicular to the z axis is the same, and is F = 1 16
a a

1 16

dy
0 0

dz |Hy | =

2 alE0 . 64

(159)

dx
0 0

dy (|Ez |2 |Hx |2 |Hy |2) = 0.

(160)

b) Turning to the cavity Q, we rst calculate the time-averaged energy U in the cavity: U = 1 2 dVol
2 a2lE0 |E |2 + |H |2 = . 8 32

(161)

Princeton University 2001

Ph501 Set 7, Solution 7

36

The energy lost per cycle into the walls is the time-averaged power loss P times the period T = 2/ . The power P lost in the cavity walls can be calculated by evaluating the component of the (time-averaged) Poynting vector perpendicular to the walls: P = = c 8 S
to walls

dArea =

c 8

Re(E H )

to walls

dArea (162)

Re(E H ) dArea.

If the elds were actually those specied by eqs. (150)-(155), which assume perfect conductors, the power lost to the walls would be zero. For a good, but not perfect, conductor, it is an excellent approximation to suppose the cavity magnetic eld is that given by the perfect-conductor approximation, eqs. (153)-(155), but to take the electric eld near the conducting walls as having a small parallel component given by E where
at the walls

d (1 i) nH , 2c

(163)

c d= (164) 2 is the outward is the skin depth at frequency for the walls of conductivity , and n normal vector. This relation follows from the 4th Maxwell equation, evaluated just inside the surface of the conductor where J = E, H = the curl of which yields, 4 H , (166) c2 t where the approximations are valid for a good conductor. [This diusion equation, due to Lord Kelvin, was the basis of time-dependent electrodynamics in the era shortly before Maxwell claried that if = 0 then waves can propagate with speed c without diusionlike distortion. It is amazing from a modern perspective that the rst telegraph systems were successfully constructed using eq. (166) as their theoretical model.] 2 H Inside the conductor, and for waves of frequency and wave vector k of the form ei(kxt) , eq. (166) becomes 4i k2 , (167) c2 so that 4 1 + i 1+i . (168) k c d 2 Then, the Maxwell equation (165) becomes H ik H ik n 4 E, c (169) 1 E 4 1 E 4 4 J+ = E+ E, c c t c c t c (165)

Princeton University 2001

Ph501 Set 7, Solution 7

37

which can also be written as eq. (163). Inserting eq. (163) into (162), we nd the general expression P = d 16 H
2

dArea.

(170)

Evaluating this for the present example, we nd P


into walls

2 dE0 (a2 + 2al). 32

(171)

The energy lost per cycle is P T = 2 P / , so the cavity Q is Q= al volume U = = . P 2 2d(a + 2l) d surface area (172)

Princeton University 2001 8. Given the electric eld

Ph501 Set 7, Solution 8

38

E (t) = E0 e0 t/4Q ei0 t , its Fourier components are given by E = 1 E0 E (t)eit dt = 2 0 2 iE0 1 = , 2 0 + i 0 /4Q
0

t > 0,

(173)

ei( 0 +i0 /4Q)t dt

(174) where we ignore the oscillatory contribution associated with the limit t . The Fourier analysis of the stored energy U therefore behaves as U |E |2 1 . ( 0)2 + ( 0 /4Q)2 (175)

Princeton University 2001

Ph501 Set 7, Solution 9

39

9. The electric eld of a standing wave mode of angular frequency inside a cubical cavity of edge a has components of the form Ex = E0 cos etc. The wave equation, 2 E = yields the dispersion relation k= 2 l + m2 + n2 = . a c (178) 1 2E , c2 t (177) my nz it lx sin sin e , a a a (176)

This leads to the interpretation that a mode (l, m, n) has a wave vector k whose components are (l, m, n). (179) a The modes populate a cubical lattice in the rst octant of k-space, with /a as the lattice constant. For l, m, and n large, the number of modes in interval d about frequency = kc is equal to (a/)3 times the volume of a shell of thickness dk = d/c in the rst octant of k-space times two since there are two possible polarization of the electric eld for each set of indices (l, m, n). Thus, a3 2 d a3 1 2 ( d/c ) = . (180) 4 ( /c ) 3 8 2 c3 Jeans contribution to this result was to note that only indices in the rst octant correspond to physical modes, and therefore Rayleighs original calculation was to be divided by 8. dN = 2

Princeton University 2001

Ph501 Set 7, Solution 10

40

10. Given the zeroth order electric eld of a right circular cavity, Ez,(0) = E0 eit , (181)

we integrate the 4th Maxwell equation around a loop of radius r in the x-y plane to nd the rst correction to the (initially zero) magnetic eld, H(1) dl = 2rH,(1) = so that 1 c Ez,(0) ir2 dS = E0 eit , t c (182)

ir E0eit. (183) 2c Next, we consider a loop in the x-z plane that includes the z axis and the line x = r, for which Faradays law tells us that H,(1) = E(1) dl = hEz,(1) so that 1 = c H,(1) 2 hr2 E0 eit , dS = 2 t 4c (184)

2 r 2 Ez,(1) = 2 E0 eit . 4c We now iterate, rst using a loop of radius r in the x-y plane to nd H(2) dl = 2rH,(2) = so that 1 c Ez,(1) i 3 r4 dS = E0 eit , 3 t 4c

(185)

(186)

i 3 r3 E0 eit . (187) 3 8c Again, we consider a loop in the x-z plane that includes the z axis and the line x = r, for which Faradays law tells us that H,(2) = E(2) dl = hEz,(2) so that Ez,(2) Thus, Ez = E0 eit 1
(1)n 2 r2 4 r4 it + ... = E e 0 2 4c2 16c4 n=0 (n!)

1 = c

H,(2) 4 hr4 dS = E0 eit , 4 t 16c

(188)

4 r4 = E0 eit . 4 16c
2n

(189)

r 2c

= E0J0

r it e . c (190)

Princeton University 2001

Ph501 Set 7, Solution 11

41

11. We seek a standing wave solution where, say, the time dependence of Ez is cos t. The cavity is symmetric about the plane z = 0, so we expect the z dependence of Ez to have the form cos kn z , where kn =

(2n 1)/2d, if Ez (0, d) = Ez (0, d) = 0, n/d,


if Ez (0, d)/z = Ez (0, d)/z = 0.

(191)

We can combine these two cases in the notation kn = (2n n0 ) n0 = 1, if Ez (0, d) = Ez (0, d) = 0, , where 2d n = 2, if E (0, d)/z = E (0, d)/z = 0. 0 z z (192)

where n = 1, 2, 3, ... Our trial solution, Ez (r, z, t) = fn (r) cos kn z cos t, must satisfy the wave equation 2Ez 1 2Ez 1 = c2 t2 r r rfn r
2 kn

(193)

2 fn = 0. c2

(194)

This is the dierential equation for the modied Bessel function of order zero, I0(Kn r), where 2 2 2 2 2 2 Kn = kn 2 = (2n n0 ) , (195) c 2d the free-space wavelength at frequency is = 2c/ , and I0(x) = 1 + (x/2)2 + (x/2)4 (x/2)6 + + (2!)2 (3!)2 (196)

In the special case of kn = 0, eq. (194) reverts to that for the ordinary Bessel function J0 , and the elds (30)-(31) are obtained. Since this form cannot exist in a cavity with apertures, we ignore it in further discussion. A Fourier series for Ez with nonzero kn is then

Ez (r, z, t) =
n=1

an I0(Kn r) cos kn z cos t.

(197)

The radial component of the electric eld is obtained from E= so that Er (r, z, t) = 1 r r = 2 an kn
n

1 rEr Ez + = 0, r r z

(198)

rI0(Kn r) dr sin kn z cos t (199)

1(Kn r) sin kn z cos t, an kn I


n

Princeton University 2001

Ph501 Set 7, Solution 11

42

using the fact that d(xI1 )/dx = xI0, and where


2 4 1(x) = 2I1 (x) = 1 + (x/2) + (x/2) + I x 1!2! 2!3!

(200)

The azimuthal component of the magnetic eld is obtained from ( E ) = so that B (r, z, t) =
2 dI0 (Kn r) kn r c an I1(Kn r) cos kn z sin t n dr 2 r 1(Kn r) cos kn z sin t, an I = n

Er Ez 1 B = , z r c t

(201)

(202)

using the fact that I0 (x) = I1(x). We desire that the transverse elds Er and B vary linearly with r. According to eqs. (199)-(200) and (202), this requires that Kn = 0. The simplest choice is n = 1, n0 = 1, so that kn = /2d and d = /4. The elds are z Ez = E0 cos cos t, 2d r z Er = E0 sin cos t, 4d 2d r z E0 cos sin t. B = 4d 2d (203) (204) (205)

The cavity length is 2d = /2, and Ez vanishes on axis at the ends of the cavity. This conguration is called the mode in accelerator physics. Since Er (z = d) = 0, this mode cannot exist in a structure with conducting walls at the planes z = d; apertures are required. The electric eld is perpendicular to the walls of a perfectly conducting cavity. Expressing the shape of the walls as r(z ), we then have Ez z dr 4d = , = cot dz Er r wd which integrates to the form r 2 = b2 4d
2

(206)

ln sin

z , 2d

(207)

where b is the radius of the apertures at z = d. Near z = d, the prole is a hyperbola. Since r as z 0, no real cavity can support the idealized elds (203)-(205). However, it turns out that a cavity with maximum radius a = 0.4d has a Fourier expansion (197) where a2 = 0.15a1 , so the elds can be a good approximation to eqs. (203)-(205) in real devices.

Princeton University 2001

Ph501 Set 7, Solution 11

43

We can obtain additional formal solutions in which Kn = 0 for any value of n, and for n0 either 1 or 2. However, these solutions are not really distinct from eqs. (203)(205), but are simply the result of combining any number of /2 cells into a larger structure. Such multicell -mode structures are dicult to operate in practice, because the strong coupling of the elds from one cell to the next makes the useful range of drive frequencies extremely narrow. The main application of -mode cavities is for so-called rf guns, in which a half cell has a surface at z 0 suitable for laser-induced photoemission of electrons, which are then accelerated further in one or a few more subsequent cells. See K.T. McDonald, Design of the Laser-Driven RF Electron Gun for the BNL Accelerator Test Facility, IEEE Trans. Electron Devices, 35, 2052 (1988).1

http://puhep1.princeton.edu/~mcdonald/examples/EM/mcdonald_ieeeted_35_2052_88.pdf

Princeton University 2001

Ph501 Set 7, Solution 12

44

12. An estimate of the lowest rf frequency of the reex klystron cavity can be made via an equivalent LC circuit:

1 = LC

1 L(C1 + C2 )

(208)

where C1 is the capacitance between the left termination plate and the disc of radius a at the left end of the center conductor, C2 is the capacitance between the inner and outer conductor, and L is the self inductance between the inner and outer conductor. The capacitances C1 and C2 are in parallel, and so are added to yield the total capacitance C . C1 is estimated by the usual parallel-plate formula, C1 a2 Area = . 4 height 4d (209)

C2 is estimated by the capacitance of length h d of a coaxial cable, C2 hd . 2 ln b/a (210)

The main interest in this type of cavity is for small gap d, so we write C = C1 + C2 a2 2dh 1+ 2 4d a ln b/a . (211)

L is estimated by the inductance of length h d h of a coaxial cable, L c2 [ C 2h ln b/a hd , c2 2 /(h d)] (212)

recalling that the product of the capacitance per unit length and the inductance per unit length of a transmission line (in vacuum) is 1/c2 . The lowest cavity frequency is then estimated to be c 1 a LC 2d . h(1 + 2dh/a2 ln b/a) ln b/a (213)

For d small enough, we can neglect the capacitance C2 , and eq. (213) simplies to c 1 a LC 2d h ln b/a (d a2/h). (214)

Princeton University 2001

Ph501 Set 7, Solution 12

45

This result is small compared to = c/h, the resonant frequency of a terminated coaxial cable of length h, which shows that it is possible to obtain low cavity frequencies without large cavity size. The book Klystrons and Microwave Triodes by Princetonian D.R. Hamilton (Dover, 1966) quotes (p. 75) a numerical analysis of the reex klystron (dating from 1934) as claiming that = 0.3c/a when b/a = 3, h/a = 3, and d/a = 0.32. Equation (213) yields 0.44c/a for these values, in reasonable agreement. Note that the simple coaxial cavity is not the limit of the reex klystron cavity as d 0, since C1 in that case. Rather, the coaxial cavity obtains when C1 and d are both set to zero, in which case eq. (213) gives the estimate c/h, which is a factor of smaller than the exact result.

Princeton University 2001

Ph501 Set 7, Solution 13

46

13. The electric eld of a TE mode in a rectangular waveguide of edges a < b has the form ny i(kg zt) mx sin e Ex = E0 cos , (215) a b mx ny i(kg zt) , (216) Ey = E0 sin cos e a b Ez = 0, (217) where the guide wave number kg is found from the wave equation to obey m 2 n 2 + . (218) a b The lowest frequency mode has indices m = 0, n = 1, for which only Ex is nonzero.
2+ w = c kg

The magnetic eld of this mode follows from Faradays law, 1 H iH E = = , c t c so that Hx = 0, ckg y E0 sin ei(kg zt), Hy = b ic y i(kg zt) E0 cos e Hz = , b b and kg = 2 2 2 = 2 c b c c 8
2

(219)

(220) (221) (222)

1 2b

(223)

The time-averaged power transmitted down the guide follows from the Poynting vector, P
transmitted

Sz dx dy =

Ex Hy dx dy = .

kg 2 c2 ab E0 16 (224)

c 2 abE0 = 1 16 2b

To nd the time-averaged power loss in the walls, we recall the argument of prob. 4, which led to the general result (170), 2 d P lost in walls = H dArea, (225) 16 where d = c/ 2 is the skin depth. For the present example, the power lost into the walls per unit length along the guide is P
lost

= 2 =

d 16

a 0

|Hz (x, 0, z )|2 dx +


2 c2 kg 2 2 2

b 0

[|Hy (0, y, z )|2 + |Hz (0, y, z )|2 ] dy = d 2 c2 2 b a + E0 8 2 b2 2 (226)

d 2 c a+ E 8 0 2 b2

2 2

+ 2b

c b 2 b2 2
2

bc = 16

2a c 2 E0 1 + b

Princeton University 2001

Ph501 Set 7, Solution 13

47

Finally, the attenuation factor is given by P lost 1 = = P trans a c 1+ b 1


2a 2b 2b 2 2

c 4 = 4 a

2a 1 1+ b c 1

2b 2b

2 2

(227)

Princeton University 2001

Ph501 Set 7, Solution 14

48

14. a) The current in the semicircular loop creates a magnetic eld that has a component along the axis of the guide. Since TM modes have no longitudinal magnetic eld, the eld of the loop does not couple to these modes. b) It is derived on pp. 170-171 of the Notes that an oscillatory, transverse current distribution J inside a waveguide excites a waveguide mode with normalized electric eld E0 to strength 2 (228) E = Z E0 J E0 dArea, c where Z = k/kg for TE modes and Z = kg /k for TM modes. Comparing with prob. 9, the lowest TE mode in the present problem (0 < x < a, 0 < y < b < a) has x 2 , E0 = sin y (229) ab a 2 dArea = 1. where the normalization condition is E0 Since the electric eld of this mode is independent of y , the excitation of this mode by the loop current will be independent of the position of the loop in y . Taking to measure the angle around the semicircular loop, we have J E0 dArea =
0 2

r d I0E0 sin = rI0


0

2 ab

d sin
0

r sin sin a (230)

r 2 I0 a ab

d sin2 =

r I0 , a 2ab

2 2

where the approximation holds for r kg =

a. The wave numbers are related by


2

2 2 2 = k 1 2 c a 2a

(231)

According to eqs. (228)-(229), the strength of the eld in the lowest TE mode is E= x 2 k 2 r2 I0 , y sin c kg a ab a (232)

The (time-averaged) power in the lowest TE mode follows from the Poynting vector, P
trans

= =

Sz 4 k c kg

c k 2 r2 2 dx dy = E dx dy = 8Z 4c kg a 2 4 4 r I0 a 4 r 2a I0 = . 2 b 2a b c 2a 1 2a

2 I0 ab

(233)

This calculation holds for TE waves excited in either direction down the guide. As usual, to convert this result from Gaussian to SI units, simply replace 4/c by 377 .

Princeton University

Ph501 Electrodynamics Problem Set 8


Kirk T. McDonald (2001) kirkmcd@princeton.edu http://puhep1.princeton.edu/~mcdonald/examples/

Princeton University 2001

Ph501 Set 8, Problem 1

1. Wire with a Linearly Rising Current A neutral wire along the z -axis carries current I that varies with time t according to

I (t ) =

(t 0), is a constant.

(1)

t (t > 0),

Deduce the time-dependence of the electric and magnetic elds, E and B, observed at a point (r, = 0, z = 0) in a cylindrical coordinate system about the wire. Use your expressions to discuss the elds in the two limiting cases that ct r and ct = r + , where c is the speed of light and r. The related, but more intricate case of a solenoid with a linearly rising current is considered in http://puhep1.princeton.edu/~mcdonald/examples/solenoid.pdf

Princeton University 2001

Ph501 Set 8, Problem 2

2. Harmonic Multipole Expansion A common alternative to the multipole expansion of electromagnetic radiation given in the Notes assumes from the beginning that the motion of the charges is oscillatory with angular frequency . However, we still use the essence of the Hertz method wherein the current density is related to the time derivative of a polarization:1 . J=p The radiation elds will be deduced from the retarded vector potential, A= 1 c 1 [ J] dVol = r c ] [p dVol, r (3) (2)

which is a solution of the (Lorenz gauge) wave equation 2 A 1 2A 4 = J. 2 2 c t c (4)

Suppose that the Hertz polarization vector p has oscillatory time dependence, p(x, t) = p (x)eit . Using the expansion + ... r = Rr n of the distance r from source to observer, (6) (5)

show that A = i ei(kRt) cR p (r ) 1 + r n 1 ik + ... R dVol , (7)

where no assumption is made that R

source size or that R

= 2/k = 2c/ .

Consider now only the leading term in this expansion, which corresponds to electric dipole radiation. Introducing the total electric dipole moment, P
1

p (r ) dVol ,

(8)

Some consideration of the related topics of Hertz vectors and scalars is given in the Appendix of http://puhep1.princeton.edu/~mcdonald/examples/smallloop.pdf

Princeton University 2001 use

Ph501 Set 8, Problem 2

1 E c t to show that for an observer in vacuum the electric dipole radiation elds are B=A and B= B = k2 E = k2 ei(kRt) i P, 1+ n R kR

(9)

(10)

ei(kRt) 1 i (P n ) + [3( n n P) n P] 2 2 . (11) R k R kR Alternatively, deduce the electric eld from both the scalar and vector potentials via E = in both the Lorenz and Coulomb gauges. For large R, Bfar k while for small R, Bnear ik ( n P)eit , 2 R Enear 3( n P) n P it e , 3 R (14)
2e i(kRt)

1 A , c t

(12)

P, n

, Efar Bfar n

(13)

Enear, and the electric eld Enear has the shape of the static dipole eld Thus, Bnear of moment P, modulated at frequency . Calculate the Poynting vector of the elds of a Hertzian oscillating electric dipole (10)(11) at all points in space. Show that the time-averaged Poynting vector has the same form in the near zone as it does in the far zone, which conrms that (classical) radiation exists both close to and far from the source. Extend your discussion to the case of an oscillating, point magnetic dipole by noting that if E(r, t) and B(r, t) are solutions to Maxwells equations in free space (i.e., where the charge density and current density J are zero), then the dual elds E (r , t ) = B (r , t ), are solutions also. B (r , t ) = E (r , t ), (15)

Princeton University 2001 3. Rotating Electric Dipole

Ph501 Set 8, Problem 3

An electric dipole of moment p0 lies in the x-y plane and rotates about the x axis with angular velocity .

Calculate the radiation elds and the radiated power according to an observer at angle to the z axis in the x-z plane. towards the observer, so that n n . Dene n z = cos , and let l=y

Show that Brad = p0 k 2 ei(krt) i (cos y l ), r Erad = p0 k 2 ei(krt) ), (cos l+i y r (16)

where r is the distance from the center of the dipole to the observer. ), the radiation is linearly polarized, Note that for an observer in the x-y plane ( n=x while for an observer along the z axis it is circularly polarized. Show that the (time-averaged) radiated power is given by d P c 2 4 = p k (1 + cos2 ), d 8 0 P =
4 2cp2 2p2 4 0k = 03 . 3 3c

(17)

This example gives another simple picture of how radiation elds are generated. The eld lines emanating from the dipole become twisted into spirals as the dipole rotates. At large distances, the eld lines are transverse...

Princeton University 2001 4. Magnetars

Ph501 Set 8, Problem 4

The x-ray pulsar SGR1806-20 has recently been observed to have a period T of 7.5 s = 8 1011 . See, C. Kouveliotou et al., An and a relatively large spindown rate T X-ray pulsar with a superstrong magnetic eld in the soft -ray repeater SGR1806-20, Nature 393, 235-237 (1998).2 Calculate the maximum magnetic eld at the surface of this pulsar, assuming it to be a standard neutron star of mass 1.4M = 2.8 1030 kg and radius 10 km, that the mass density is uniform, that the spindown is due to electromagnetic radiation, and that the angular velocity vector is perpendicular to the magnetic dipole moment of the pulsar. Compare the surface magnetic eld strength to the so-called QED critical eld strength m2c3 /eh = 4.4 1013 gauss, at which electron-positron pair creation processes become highly probable.

http://puhep1.princeton.edu/~mcdonald/examples/EM/kouveliotou_nature_393_235_98.pdf

Princeton University 2001

Ph501 Set 8, Problem 5

5. Radiation of Angular Momentum Recall that we identied a eld momentum density, Peld = and angular momentum density Leld = r Peld. (19) S U = k , 2 c c (18)

Show that for oscillatory sources, the time-average angular momentum radiated in unit solid angle per second is (the real part of) d L 1 3 n B ) B ( n E)]. = r [E( dt d 8 Thus, the radiated angular momentum is zero for purely transverse elds. In prob. 2, eq. (11) we found that for electric dipole radiation there is a term in E with 1/r2 . Show that for radiation by an oscillating electric dipole p, En d L ik 3 = ( n p)( n p ). dt d 4 (21) (20)

If the dipole moment p is real, eq. (21) tells us that no angular momentum is radiated. However, when p is real, the radiation is linearly polarized and we expect it to carry no angular momentum. Rather, we need circular (or elliptical) polarization to have radiated angular momentum. The radiation elds (16) of prob. 2 are elliptically polarized. Show that in this case the radiated angular momentum distribution is k3 d L = p2 sin l, dt d 4 0 and P d L = z. dt (22)

[These relations carry over into the quantum realm where a single (left-hand) circularly polarized photon has U = h , p = h k, and L = h . For a another view of waves that carry angular momentum, see http://puhep1.princeton.edu/~mcdonald/examples/oblate_wave.pdf

Princeton University 2001

Ph501 Set 8, Problem 6

6. Oscillating Electric Quadrupole An oscillating linear quadrupole consists of charge 2e at the origin, and two charges e each at z = a cos t.

Show that for an observer in the x-z plane at distance r from the origin, Erad = 4k 3 a2e sin(2kr 2t) sin cos l, r (23)

n . This radiation is linearly polarized. where l=y Show also that the time-averaged total power is P = 16 6 4 2 ck a e . 15 (24)

Princeton University 2001

Ph501 Set 8, Problem 7

7. A single charge e rotates in a circle of radius a with angular velocity . The circle is centered in the x-y plane.

The time-varying electric dipole moment of this charge distribution with respect to the origin has magnitude p = ae, so from Larmors formula (prob. 2) we know that the (time-averaged) power in electric dipole radiation is PE 1 = 2a2e2 4 . 3c3 (25)

This charge distribution also has a magnetic dipole moment and an electric quadrupole moment (plus higher moments as well!). Calculate the total radiation elds due to the E1, M1 and E2 moments, as well as the angular distribution of the radiated power and the total radiated power from these three moments. In this pedagogic problem you may ignore the interference between the various moments. Show, for example, that the part of the radiation due only to the electric quadrupole moment obeys a4 e2 6 d PE 2 = (1 cos4 ), d 2c5 Thus, PE 2 v2 12a2 2 , = PE 1 5c2 c2 where v = a is the speed of the charge. (27) PE 2 = 8a4e2 6 . 5c5 (26)

Princeton University 2001

Ph501 Set 8, Problem 8

8. Radiation by a Classical Atom a) Consider a classical atom consisting of charge +e xed at the origin, and charge e in a circular orbit of radius a. As in prob. 5, this atom emits electric dipole radiation loss of energy the electron falls into the nucleus! Calculate the time to fall to the origin supposing the electrons motion is nearly circular at all times (i.e., it spirals into the origin with only a small change in radius per turn). You may ignore relativistic corrections. Show that tfall = a3 , 2 4r0 c (28)

where r0 = e2/mc2 is the classical electron radius. Evaluate tfall for a = 5 109 cm = the Bohr radius. b) The energy loss of part a) can be written as dU e2a 4 e2 = Pdipole = dt c a or a c
3

v c

U T

v c

(29)

v dipole energy loss per revolution energy c

(30)

For quadrupole radiation, prob. 5 shows that v quadrupole energy loss per revolution energy c
5

(31)

Consider the Earth-Sun system. The motion of the Earth around the Sun causes a quadrupole moment, so gravitational radiation is emitted (although, of course, there is no dipole gravitational radiation since the dipole moment of any system of masses about its center of mass is zero). Estimate the time for the Earth to fall into the Sun due to gravitational radiation loss. What is the analog of the factor ea2 that appears in the electrical quadrupole moment (prob. 5) for masses m1 and m2 that are in circular motion about each other, separated by distance a? Also note that in Gaussian units the electrical coupling constant k in the force law F = ke1e2/r2 has been set to 1, but for gravity k = G, Newtons constant. The general relativity expression for quadrupole radiation in the present example is PG2 =
2 32 G m2 1 m2 a4 6 5 c 5 (m 1 + m 2 )2

(32)

[Phys. Rev. 131, 435 (1963)]. The extra factor of 4 compared to E2 radiation arises because the source term in the gravitational wave equation has a factor of 16 , rather than 4 as for E&M.

Princeton University 2001

Ph501 Set 8, Problem 9

10

9. Why Doesnt a Steady Current Loop Radiate? A steady current in a circular loop presumably involves a large number of electrons in uniform circular motion. Each electron undergoes accelerated motion, and according to prob. 5 emits radiation. Yet, the current density J is independent of time in the limit of a continuous current distribution, and therefore does not radiate. How can we reconcile these two views? The answer must be that the radiation is canceled by destructive interference between the radiation elds of the large number N of electrons that make up the steady current. Prob. 5 showed that a single electron in uniform circular motion emits electric dipole radiation, whose power is proportional to (v/c)4 . But the electric dipole moment vanishes for two electrons in uniform circular motion at opposite ends of a common diameter; quadrupole radiation is the highest multipole in this case, with power proportional to (v/c)6. It is suggestive that in case of 3 electrons 120 apart in uniform circular motion the (time-dependent) quadrupole moment vanishes, and the highest multipole radiation is octupole. For N electrons evenly spaced around a ring, the highest multipole that radiates in the N th, and the power of this radiation is proportional to (v/c)2N +2. Then, for steady motion with v/c 1, the radiated power of a ring of N electrons is very small. Verify this argument with a detailed calculation. Since we do not have on record a time-dependent multipole expansion to arbitrary order, return to the basic expression for the vector potential of the radiation elds, A(r, t) = 1 c [ J] 1 dVol r cR [J] dVol = 1 cR J(r , t = t r/c) dVol , (33)

where R is the (large) distance from the observer to the center of the ring of radius a. For uniform circular motion of N electrons with angular frequency , the current density J is a periodic function with period T = 2/ , so a Fourier analysis can be made where J (r , t ) = with J m (r ) = Then, A(r, t) = etc. The radiated power follows from the Poynting vector, c 2 c 2 dP = R |B|2 = R | A|2 . d 4 4 (37)
m m= T 0

Jm (r )eimt ,

(34)

1 T

J(r , t )eimt dt . Am (r)eimt ,

(35) (36)

However, as discussed on p. 181 of the Notes, one must be careful in going from a Fourier analysis of an amplitude, such as B, to a Fourier analysis of an intensity that

Princeton University 2001

Ph501 Set 8, Problem 9

11

depends on the square of the amplitude. Transcribing the argument there to the present case, a Fourier analysis of the average power radiated during one period T can be given as d P d 1 T dP cR2 T cR2 T 2 |B| dt = B Bm eimt dt = dt = T 0 d 4T 0 4T 0 m 2 2 T 1 cR cR Bm B eimt dt = Bm Bm = 4 m T 0 4 m= cR2 = 2
m=0

|Bm |

dPm . m=0 d

(38)

That is, the Fourier components of the time-averaged radiated power can be written cR2 cR2 cR2 dPm 2 2 Am |2 , = |Bm | = | Am | = |imk n d 2 2 2 points from the center of the ring to the observer. where k = /c and n Evaluate the Fourier components of the vector potential and of the radiated power rst for a single electron, with geometry as in prob. 5, and then for N electrons evenly spaced around the ring. It will come as no surprise that a 3-dimensional problem with charges distributed on a ring leads to Bessel functions, and we must be aware of the integral representation im 2 imiz cos e d. (40) J m (z ) = 2 0 Use the asymptotic expansion for large index and small argument, (ex/2)m Jm (mx) 2m (m 1, x 1), (41) (39)

to verify the suppression of the radiation for large N . This problem was rst posed (and solved via series expansions without explicit mention of Bessel functions) by J.J. Thomson, Phil. Mag. 45, 673 (1903).3 He knew that atoms (in what we now call their ground state) dont radiate, and used this calculation to support his model that the electric charge in an atom must be smoothly distributed. This was a classical precursor to the view of a continuous probability distribution for the electrons position in an atom. Thomsons work was followed shortly by an extensive treatise by G.A. Schott, Electromagnetic Radiation (Cambridge U.P., 1912), that included analyses in term of Bessel functions correct for any value of v/c. These pioneering works were largely forgotten during the following era of nonrelativistic quantum mechanics, and were reinvented around 1945 when interest emerged in relativistic particle accelerators. See Arzimovitch and Pomeranchuk,4 and Schwinger.5
3 4

http://puhep1.princeton.edu/~mcdonald/examples/EM/thomson_pm_45_673_03.pdf http://puhep1.princeton.edu/~mcdonald/examples/EM/arzimovitch_jpussr_9_267_45.pdf 5 http://puhep1.princeton.edu/~mcdonald/accel/schwinger.pdf

Princeton University 2001 10. Spherical Cavity Radiation

Ph501 Set 8, Problem 10

12

Thus far we have only considered waves arising from the retarded potentials, and have ignored solutions via the advanced potentials. Advanced spherical waves converge on the source rather than propagate away so we usually ignore them. Inside a cavity, an outward going wave can bounce o the walls and become an inward going wave. Thus, a general description of cavity radiation should include both kinds of waves. Reconsider your derivation in Prob. 1, this time emphasizing the advanced waves, for which t = t + r/c is the advanced time. It suces to consider only the elds due to oscillation of an electric dipole moment. If one superimposes outgoing waves due to oscillating dipole p = p0 eit at the origin with incoming waves associated with dipole p, then we can have standing waves and zero total dipole moment. Suppose all this occurs inside a spherical cavity with perfectly conducting walls at radius a. Show that the condition for standing waves associated with the virtual electric dipole is cot ka = 1 ka ka c min = 2.74 , a (42)

where k = /c. [A quick estimate would be max = 2a min = c/a.] By a simple transformation, use your result to nd the condition for standing magnetic dipoles waves inside a spherical cavity: tan ka = ka c min = 4.49 . a (43)

Princeton University 2001

Ph501 Set 8, Problem 11

13

11. Maximum Energy of a Betatron A betatron is a circular device of radius R designed to accelerate electrons (charge e, = R2 B ave through the circle. mass m) via a changing magnetic ux Deduce the relation between the magnetic eld B at radius R and the magnetic eld Bave averaged over the area of the circle needed for a betatron to function. Also deduce the maximum energy E to which an electron could be accelerated by a betatron in terms ave and R. of B , B Hints: The electrons in this problem are relativistic, so it is useful to introduce the factor = E /mc2 where c is the speed of light. Recall that Newtons second law has the same form for nonrelativistic and relativistic electrons except that in the latter case the eective mass is m. Recall also that for circular motion the rest frame acceleration is 2 times that in the lab frame.

Princeton University 2001

Ph501 Set 8, Problem 12

14

12. a) An oscillating electric dipole of angular frequency is located at distance d away from a perfectly conducting plane. The dipole is oriented parallel to the plane, as shown below.

Show that the power radiated in the direction (, ) is dP = 4A sin2 sin2 , d 2 sin cos , d and the power radiated by the dipole alone is = dP = A sin2 . d Sketch the shape of the radiation pattern for d = /2 and d = /4. b) Suppose instead that the dipole was oriented perpendicular to the conducting plane. where (44)

(45)

(46)

Show that the radiated power in this case is dP = 4A sin2 cos2 , d where = 2 cos . d (47)

(48)

In parts a) and b), the polar angles and are measured with respect to the axes of the dipoles. c) Repeat parts a) and b) for a magnetic dipole oscillator in the two orientations.

Princeton University 2001

Ph501 Set 8, Problem 13

15

13. In an array of antennas, their relative phases can be adjusted as well as their relative spacing, which leads to additional freedom to shape the radiation pattern. Consider two short, center-fed linear antennas of length L , peak current I0 and frequency , as discussed on p. 191 of the Notes. The axes of the antennas are collinear, their centers are /4 apart, and the currents have a 90 phase dierence.

Show that the angular distribution of the radiated power is dU 2 2 2 2 I0 L sin 1 + sin = cos 3 dtd 16c 2 . (49)

Unlike the radiation patterns of previous examples, this is not symmetric about the plane z = 0. Therefore, this antenna array emits nonzero momentum Prad . As a consequence, there is a net reaction force F = dPrad/dt. Show that 1 dU F= c dt 2 z. 1 12 (50)

A variant on this problem is at http://puhep1.princeton.edu/~mcdonald/examples/endfire.pdf

Princeton University 2001

Ph501 Set 8, Problem 14

16

14. a) Consider a full-wave end-re antenna whose current distribution (along the z axis) is 2z it I (z ) = I0 sin , (L/2 < z < L/2), (51) e L where L = = 2c/ .

Use the result of p. 182 of the Notes to calculate the radiated power exactly. Note that the real part of the integral vanishes, so you must evaluate the imaginary part. Show that dP I 2 sin2 ( cos ) = 0 . (52) d 2c sin2 Sketch the radiation pattern. 1 1 1 = + 2 1u 1+u 1u to show that the total radiated power is P = I2 dP d = 0 d 2c
4 0

Use tricks like

(53)

I2 I2 1 cos v dv = 0 Cin(4 ) = 3.11 0 . v 2c 2c

(54)

(c.f. Abramowitz and Stegun, pp. 231, 244.) b) Calculate the lowest order nonvanishing multipole radiation. You may need the fact that z 2 cos z dz = (z 2 2) sin z + 2z cos z. (55) Show that to this order,
2 2 I0 82 I0 P = = 5.26 . 15 2c 2c which gives a sense of the accuracy of the multiple expansion.

(56)

Princeton University 2001

Ph501 Set 8, Problem 15

17

15. Scattering O a Conducting Sphere Calculate the scattering cross section for plane electromagnetic waves of angular frequency incident on a perfectly conducting sphere of radius a when the wavelength obeys a (ka 1). Note that both electric and magnetic dipole moments are induced. Inside the sphere, Btotal must vanish. Surface currents are generated such that Binduced = Bincident, and because of the long wavelength, these elds are essentially uniform over the sphere. (c.f. Set 4, Prob. 8a). Show that 1 ei(krt) ) n (B 0 n , (E 0 n (57) r 2 is along the vector r that points from the center of the sphere to the distant where n observer. Escat = a3k 2 Suppose that the incident wave propagates in the +z direction, and the electric eld = 0. Show that in this case l and lz is linearly polarized along direction l, so E0 = E0 the scattering cross section can be written, z d n = a6 k 4 1 d 2

3 )2 . ln ( 4

(58)

Consider an observer in the x-z plane to distinguish between the cases of electric polarization parallel and perpendicular to the scattering plane to show that 1 d = a6 k 4 cos d 2
2

d cos = a6 k 4 1 d 2

(59)

Then, for an unpolarized incident wave, show 5 d = a6k 4 (1 + cos2 ) cos , d 8 and d 10 6 4 d = ak . d 3 Sketch the angular distribution (60). Note that = d (180 ) d d (0 ) = 9, d (60)

(61)

(62)

so the sphere reects much more backwards than it radiates forwards. Is there any angle for which the scattered radiation is linearly polarized for unpolarized incident waves?

Princeton University 2001

Ph501 Set 8, Problem 16

18

16. Exotic radiation eects of charges that move at (essentially) constant velocity but cross boundaries between various media can be deduced from the radiation spectrum equation (14.70) from the textbook of Jackson (p. 182 of the Notes): 2 dU = 2 3 dd 4 c
nr)/c) J(r, t)ei(t( dt d3 r n 2

(63)

where dU is the radiated energy in angular frequency interval d emitting into solid is a unit vector towards the observer. angle d, J is the source current density, and n Consider the example of the sweeping electron beam in an (analog) oscilloscope. In the fastest of such devices (such as the Tektronix model 7104) the speed of the beam spot across the face of an oscilloscope can exceed the velocity of light, although of course the velocity of the electrons does not. Associated with this possibility there should be a kind of Cerenkov radiation, as if the oscilloscope trace were due to a charge moving with superluminal velocity. As a simple model, suppose a line of charge moves in the y direction with velocity u c, where c is the speed of light, but has a slope such that the intercept with the x axis moves with velocity v > c, as shown in the gure below. If the region y < 0 is occupied by, say, a metal the charges will emit transition radiation as they disappear into the metals surface. Interference among the radiation from the various charges then leads to a strong peak in the radiation pattern at angle cos = c/v , which is the Cerenkov eect of the superluminal source all of which can be deduced from eq. (63).

0.3 a) A sloping line of charge moves in the y direction with velocity vy = u c such that its intercept with the x axis moves with velocity vx = v > c. As the charge disappears into the conductor at y < 0 it emits transition radiation. The radiation appears to emanate from a spot moving at superluminal velocity and is concentrated on a cone of angle cos1 (c/v ). b) The angular distribution of the radiation is discussed in a spherical coordinates system about the x axis.

Princeton University 2001

Ph501 Set 8, Solution 1 Solutions

19

1. The suggested approach is to calculate the retarded potentials and then take derivatives to nd the elds. The retarded scalar and vector potentials and A are given by (x, t) = (x , t R/c) 3 d x, R and A(x, t) = 1 c J(x , t R/c) 3 d x , (64) R

where and J are the charge and current densities, respectively, and R = |x x |. In the present case, we assume the wire remains neutral when the current ows (compare Prob. 3, Set 4). Then the scalar potential vanishes. For the vector potential, we see that only the component Az will be nonzero. Also, J d3 x can be rewritten as I dz for current in a wire along the z -axis. For an observer at (r, 0, 0) and a current element at (0, 0, z ), we have R = r2 + z 2 . Further, the condition that I is nonzero only for time t > 0 implies that it contributes to the elds only for z such that (ct)2 > R2 = r2 + z 2. That is, we need to evaluate the integral only for |z | < z 0 (ct)2 r2 , (65)

which must be positive to have physical signicance. Altogether, Az (r, 0, 0, t) = t z0 1 ct + z0 2z0 dz = t ln c z0 c c ct z0 c r2 + z 2 2 z0 + ct z0 = t ln . c r c

(66)

[The two forms tend to arise depending on whether or not one notices that the integrand is even in z .] The magnetic eld is obtained via B = A. Since only Az is nonzero the only nonzero component of B is Az 2z0 B = = 2 . (67) r cr [Some chance of algebraic error in this step!] The only nonzero component of the electric eld is Ez = For long times, ct B 2 z0 + ct 1 Az = 2 ln . c t c r (68)

r, z0 ct, and the elds become Ez 2 2ct r 2ct ln = B ln 0 c2 r ct r B0 , (69)

2t 2I (t) = = B0(t), cr cr

where B0 (t) = 2I (t)/cr is the instantaneous magnetic eld corresponding to current I(t). That is, we recover the magnetostatic limit at large times. For short times, ct = r + with r, after the elds rst become nonzero we have (70) z0 = 2r + 2 2r ,

Princeton University 2001 so 2 B 2 c 2 , r and

Ph501 Set 8, Solution 1

20

2 r + + Ez 2 ln c r

2r

2 c2

2 = B . r

(71)

In this regime, the elds have the character of radiation, with E and B of equal magnitude, mutually orthogonal, and both orthogonal to the line of sight to the closest point on the wire. (Because of the cylindrical geometry the radiation elds do not have 1/r dependence which holds instead for static elds.) In sum, the elds build up from zero only after time ct = r. The initial elds propagate outwards at the speed of light and have the character of cylindrical waves. But at a xed r, the electric eld dies out with time, and the magnetic eld approaches the instantaneous magnetostatic eld due to the current in the wire. Of possible amusement is a direct calculation of the vector potential for the case of a constant current I0. First, from Amperes law we know that B = 2I0 /cr = Az /r, so we have that Az = 2I0 ln r + const. c (72)

Whereas, if we use the integral form for the vector potential we have Az (r, 0, 0) = 1 c

2I0 I dz 0 = c r2 + z 2

2I0 ln r + lim ln(z + z c

dz r2 + z 2 (73)

z 2 + r 2 ).

Only by ignoring the last term, which does not depend on r for a long wire, do we recover the elementary result.

Princeton University 2001 2. The expansion

Ph501 Set 8, Solution 2

21

+ ... r = Rr n implies that the retarded time derivative of the polarization vector is

(74)

/c) ] = p (r , t = t r/c) i p (r )ei(tR/c+r n [p = iei(kRt) p (r )eikr n ), iei(kRt)p (r )(1 ik r n (75)

where k = /c. Likewise,

r n 1 1 1+ . r R R

(76)

Then, the retarded vector potential can be written (in the Lorenz gauge) A(L) = 1 c ] [p ei(kRt) dVol i r cR p (r ) 1 + r n 1 ik + ... dVol , (77) R

The electric dipole (E 1) approximation is to keep only the rst term of eq. (77), AE 1 = i
(L)

ei(kRt) cR

p (r ) dVol ik

ei(kRt) P R

(Lorenz gauge).

(78)

We obtain the magnetic eld by taking the curl of eq. (78). The curl operation with respect to the observer acts only on the distance R. In particular, R = Hence, BE 1 = AE 1 = ik = k2
(L)

R . =n R

(79)

ei(kRt) ei(kRt) n P = ik ik n P R R R (80)

i ei(kRt) P. 1+ n R kR 1 EE 1 = ik EE 1. c t

The 4th Maxwell equation in vacuum tells us that BE 1 = Hence, EE 1 = i 1 ik 3 RP BE 1 = ei(kRt) 2 k R R 1 1 ik ik i(kRt) = ei(kRt) ( R P ) + e (R P) R2 R3 R2 R3 ik ik k2 1 1 ( 3 n 3 n P) 2Pei(kRt) = ei(kRt) 3 2 2 R R R R R = k2 1 ei(kRt) ik ) (P n ) + ei(kRt) [P 3(P n n]. n R R2 R3 (82) (81)

Princeton University 2001

Ph501 Set 8, Solution 2

22

We could also deduce the electric eld from the general relation E = V 1 A = V + ik A. c t (83)

For this, we need to know the scalar potential V (L) , which we can deduce from the Lorenz gauge condition: 1 V (L) A(L) + = 0. (84) c t For an oscillatory source this becomes i V (L) = A(L) . k In the electric dipole approximation (78) this yields6 VE 1 = ei(kRt)
(L)

(85)

ik 1 ) (P n 2 R R

(Lorenz gauge).

(86)

For small R the scalar potential is that of a time-varying dipole, VE 1,near The electric eld is given by EE 1 = VE 1 + ik AE 1
(L) (L) i(kRt) 1 1 ik ik i(kRt) 2e + e ( P R ) + k P R2 R3 R2 R3 R ik ik k2 1 1 ) 3 3 (P n n + ei(kRt) 3 P 2 2 R R R R R (L)

it Pn e . 2 R

(87)

= (P R)ei(kRt) = ei(kRt) +k 2 = k2

ei(kRt) ik 1 ) (P n ) + ei(kRt) [P 3(P n n], n R R2 R3

ei(kRt) P R

(88)

as before. The angular distribution in the far eld (for which the radial dependence is (P n ) = P (P n ) 1/R) is n n. The isotropic term P is due to the vector potential, ) while the variable term (P n n is due to the scalar potential and is purely radial. Spherical waves associated with a scalar potential must be radial (longitudinal), but the transverse character of electromagnetic waves in the far eld does not imply the absence of a contribution of the scalar potential; the latter is needed (in the Lorenz gauge) to cancel to radial component of the waves from the vector potential.
Equation (86) is not simply the electrostatic dipole potential times a spherical wave because the retarded positions at time t of the two charges of a point dipole correspond to two dierent retarded times t . For a calculation of the retarded scalar potential via V (L) = Vol []/r , see sec. 11.1.2 of Introduction to Electrodynamics by D.J. Griths.
6

Princeton University 2001

Ph501 Set 8, Solution 2

23

We could also work in the Coulomb gauge, meaning that we set A(C ) = 0. Recall (Lecture 15, p. 174) that the wave equations for the potentials in the Coulomb gauge are 2 V (C ) = 4, 1 2A(C ) 4 1 V (C ) = 2 A(C ) 2 J + . c t2 c c t (89) (90)

Equation (89) is the familiar Poisson equation of electrostatics, so the scalar potential is just the instantaneous electric-dipole potential, VE 1 =
(C )

it Pn e 2 R

(Coulomb gauge).

(91)

One way to deduce the Coulomb-gauge vector potential is via eq. (83), AE1
(C )

i i (C ) = EE1 VE1 k k ei(kRt) i(ei(kRt) eit ) ei(kRt) (P n ) + ) n = ik + [P 3(P n n] R R2 kR3


C) Afar + A( near (C )

(Coulomb gauge).

(92)

We learn that the far-zone, Coulomb gauge vector potential (i.e., the part of the vector potential that varies as 1/R) is purely transverse, and can be written as Afar = ik
(C )

ei(kRt) (P n ) n R

(Coulomb gauge).

(93)

Because the radiation part of the Coulomb-gauge vector potential is transverse, the Coulomb gauge is sometimes called the transverse gauge. The Coulomb-gauge scalar potential is negligible in the far zone, and we can say that the radiation elds are entirely due to the far-zone, Coulomb-gauge vector potential. That is, Efar = ik Afar = k 2 Bfar = Afar
(C ) (C )

ei(kRt) (P n ), n R i(kRt) (C ) 2e P. = ik Afar = k n R

(94) (95)

It is possible to choose gauges for the electromagnetic potentials such that some of their components appear to propagate at any velocity v , as discussed by J.D. Jackson, Am. J. Phys. 70, 917 (2002) and by K.-H. Yang, Am. J. Phys. 73, 742 (2005).7 The
http://puhep1.princeton.edu/~mcdonald/examples/EM/jackson_ajp_70_917_02.pdf http://puhep1.princeton.edu/~mcdonald/examples/EM/yang_ajp_73_742_05.pdf
7

Princeton University 2001

Ph501 Set 8, Solution 2

24

potentials A(v) and V (v) in the so-called velocity gauge with the parameter v obey the gauge condition c V (v) A(v) + 2 = 0. (96) v t The scalar potential V (v) is obtained by replacing the speed of light c in the Lorenzgauge scalar potential by v . Equivalently, we replace the wave number k = /c by k = /v . Thus, from eq. (86) we nd VE 1 = ei(k Rt) Then, as in eq. (88) we obtain VE 1
(v ) (v )

1 ik 3 (P R ) 2 R R

(velocity gauge).

(97)

= ei(k Rt) = k
2e

k2 1 ik 3 3 2 R R R ) (P n n + ei(k Rt)

) (P n n + ei(kRt)

1 ik 3 P 2 R R (98)

i(k Rt)

1 ik ) 3 [P 3(P n n]. 2 R R

The vector potential in the v -gauge can be obtained from eq. (83) as i i (v ) (v ) AE 1 = EE 1 VE 1 k k 1 ei(kRt) i (P n ) + ei(kRt) ) n = ik + [P 3(P n n] 2 R R kR3 i k 2 ei(k Rt) k ) ) + [P 3(P n n]. (99) (P n n ei(k Rt) i 2 k R kR kR3 This vector potential includes terms that propagate with velocity v both in the near and far zones. When v = c, then k = k and the velocity-gauge vector potential (99) reduces to the Lorenz-gauge potential (78); and when v , then k = 0 and the velocity-gauge vector potential reduces to the Coulomb-gauge potential (92).8 Turning to the question of energy ow, we calculate the Poynting vector S= c E B, 4 (100)

where we use the real parts of the elds (80) and (82), n (P ) E = k2 P n
8

cos(kR t) n ] cos(kR t) + k sin(kR t) , ) + P [3P nP R R3 R2 (101)

Thanks to J.D Jackson and K.-H. Yang for discussions of the Coulomb gauge and the velocity gauge. See also, J.D. Jackson and L.B. Okun, Historical roots of gauge invariance, Rev. Mod. Phys. 73, 663 (2001), http://puhep1.princeton.edu/~mcdonald/examples/EM/jackson_rmp_73_663_01.pdf

Princeton University 2001

Ph501 Set 8, Solution 2 cos(kR t) sin(kR t) . R kR2

25

) B = k 2P ( nP

(102)

The Poynting vector contains six terms, some of which do not point along the radial : vector n S =
2 c n ) cos (kR t) cos(kR t) sin(kR t) )] ( k 4 P 2 [ n (P nP 4 R2 kR3 2 2 n ] ( ) cos (kR t) sin (kR t) ) +k 2 P 2 [3(P nP nP R4 1 k + cos(kR t) sin(kR t) 3 R kR5 cos2 (kR t) cos(kR t) sin(kR t) c = k 4 P 2 sin2 n 4 R2 kR3 cos2(kR t) sin2 (kR t) 2 2 2 +k P [4 cos P + (3 cos 1) n] R4 k 1 + cos(kR t) sin(kR t) , (103) 3 R kR5

and P. As well as the expected radial ow where is the angle between vectors n of energy, there is a ow in the direction of the dipole moment P. Since the product cos(kR t) sin(kR t) can be both positive and negative, part of the energy ow is inwards at times, rather than outwards as expected for pure radiation. However, we obtain a simple result if we consider only the time-average Poynting vector, S . Noting that cos2 (kR t) = sin2 (kR t) = 1/2 and cos(kR t) sin(kR t) = (1/2) sin 2(kR t) = 0, eq (103) leads to S = ck 4 P 2 sin2 . n 8R2 (104)

The time-average Poynting vector is purely radially outwards, and falls o as 1/R2 at all radii, as expected for a ow of energy that originates in the oscillating point dipole (which must be driven by an external power source). The time-average angular distribution d P /d of the radiated power is related to the Poynting vector by d P ck 4 P 2 sin2 P 2 4 sin2 S = = R2 n = , d 8 8c3 (105)

which is the expression often quoted for dipole radiation in the far zone. Here we see that this expression holds in the near zone as well. We conclude that radiation, as measured by the time-average Poynting vector, exists in the near zone as well as in the far zone. Our considerations of an oscillating electric dipole can be extended to include an oscillating magnetic dipole by noting that if E(r, t) and B(r, t) are solutions to Maxwells

Princeton University 2001

Ph501 Set 8, Solution 2

26

equations in free space (i.e., where the charge density and current density J are zero), then the dual elds E (r , t ) = B (r , t ), B (r , t ) = E (r , t ), (106)

are solutions also. The Poynting vector is the same for the dual elds as for the original elds, c c S = E B = B E = S. (107) 4 4 Taking the dual of elds (10)-(11), we nd the elds E = EM1 = k 2 B = BM1 = k 2 i ei(kRt) M, 1+ n R kR (108)

i ei(kRt) 1 (M n ) + [3( n n M) n M] 2 2 . (109) R k R kR which are also solutions to Maxwells equations. These are the elds of an oscillating point magnetic dipole, whose peak magnetic moment is M. In the near zone, the magnetic eld (109) looks like that of a (magnetic) dipole. While the elds of eqs. (10)-(11) are not identical to those of eqs. (108)-(109), the Poynting vectors are the same in the two cases. Hence, the time-average Poynting vector, and also the angular distribution of the time-averaged radiated power are the same in the two cases. The radiation of a point electric dipole is the same as that of a point magnetic dipole (assuming that M = P), both in the near and in the far zones. Measurements of only the intensity of the radiation could not distinguish the two cases.

However, if measurements were made of both the electric and magnetic elds, then the near zone elds of an oscillating electric dipole, eqs. (10)-(11), would be found to be quite dierent from those of a magnetic dipole, eqs. (108)-(109). This is illustrated in the gure on the previous page, which plots the ratio E/H = E/B of the magnitudes of the electric and magnetic elds as a function of the distance r from the center of the dipoles.

Princeton University 2001

Ph501 Set 8, Solution 2

27

To distinguish between the cases of electric and magnetic dipole radiation, it suces to measure only the polarization (i.e., the direction, but not the magnitude) of either the electric of the magnetic eld vectors.

Princeton University 2001

Ph501 Set 8, Solution 3

28

3. The rotating dipole p can be thought of as two oscillating linear dipoles oriented 90 apart in space, and phased 90 apart in time. This is conveniently summarized in complex vector notation: )eit , x + iy (110) p = p0 ( for a rotation from the + x axis towards the + y axis. Thus, (t = t r/c) = 2p0 ( )ei(krt) . [ p] = p x + iy The radiation elds of this oscillating dipole are given by Brad = [ p] n k 2 p0 ei(krt) ) n = ( x + iy c2 r r k 2p0 ei(krt) i = (cos y l), r k 2 p0 ei(krt) = ). = Brad n (cos l+i y r (111)

(112) (113)

Erad

The time-averaged angular distribution of the radiated power is given by c 2 c 4 2 d P = r |Brad|2 = k p0 (1 + cos2 ), d 8 8 are orthogonal. The total radiated power is therefore since l and y P = d P 2c 2 4 2 d = k 4 p2 p0 . 0 = d 3 3c3 (115) (114)

The total power also follows from the Larmor formula, P = | = since |p |2 1 2 |p 2 = 3 4p2 0, 3 2 3c 3c (116)

2 2 p0 in the present example.

Princeton University 2001

Ph501 Set 8, Solution 4

29

4. According to the Larmor formula, the rate of magnetic dipole radiation is 2 2 m2 4 dU 2m = = , dt 3 c3 3 c3 (117)

where = 2/T is the angular velocity, taken to be perpendicular to the magnetic dipole moment m. The radiated power (117) is derived from a decrease in the rotational kinetic energy, U = I2 /2, of the pulsar: 2 dU = I = MR2 | |, dt 5 (118)

where the moment of inertia I is taken to be that of a sphere of uniform mass density. Combining eqs. (117) and (118), we have m2 = | c3 3 MR2 | . 5 3 (119)

/T 2 , we nd Substituting = 2/T , and | | = 2 T m2 = 3 c3 . MR2 T T 20 2 3(m r) rm , 3 r B= (120)

The static magnetic eld B due to dipole m is B= so the peak eld at radius R is (121)

2m . R3 Inserting this in eq. (120), the peak surface magnetic eld is related by B2 =

(122)

c3 3 MT T 3 (2.8 1033 )(7.5)(8 1011 )(3 1010 )3 = = 2.8 1030 gauss 2. 5 2 R4 5 2 (106 )4 (123) Thus, Bpeak = 1.7 1015 G = 38Bcrit , where Bcrit = 4.4 1013 G. When electrons and photons of kinetic energies greater than 1 MeV exist in a magnetic eld with B > Bcrit, they rapidly lose this energy via electron-positron pair creation. Kouveliotou et al. report that Bpeak = 8 1014 G without discussing details of their calculation.

Princeton University 2001

Ph501 Set 8, Solution 5

30

5. The time-average eld momentum density is given in terms of the Poynting vector as (the real part of) S c P eld = 2 (124) EB . c 8 Hence, the time-averaged angular momentum density is L
eld

= r Peld =

1 1 r (E B ) = r[E( n B ) B ( n E)]. 8c 8c

(125)

. writing r = rn The time-average rate of radiation of angular momentum into solid angle d is therefore d L = cr2 L dt d
eld

1 3 n B ) B ( n E)], r [E( 8

(126)

since the angular momentum density L is moving with velocity c. The radiation elds of an oscillating electric dipole moment p including both the 1/r and 1/r2 terms of eqs. (80) and (88) are, ei(krt) i 3i k+ p k+ ( n p) n , r r r i ei(krt) B = k2 1+ ( n p). r kr E = k (127) (128)

B = 0 for this case, only the second term in eq. (126) contributes to the Since n radiated angular momentum. We therefore nd k3r d L i 2i ik 3 = ( n p)( n p ), 1 ( np ) ( n p) = dt d 8 kr r 4 (129)

ignoring terms in the nal expression that have positive powers of r in the denominator, as these grow small at large distances. For the example of a rotating dipole moment (prob. 2), )eit , p = p0 ( x + iy we have ik 3p2 ik 3 p2 d L 0 0 )][ )] = Re +i = Re [ n ( x + iy n ( x iy sin [cos y l] dt d 4 4 k3 = p2 sin l. (131) 4 0 is in the x-z To nd d L /dt we integrate eq. (130) over solid angle. When vector n plane, vector l can be expressed as sin l = cos x z. (132) (130)

Princeton University 2001

Ph501 Set 8, Solution 5

31

, the contributions to d L /dt in the x-y plane As we integrate over all directions of n sum to zero, and only its z component survives. Hence, d L dt k3 d Lz z d = 2 p2 dt d 4 0 2ck 3 2 P p0 = z= z, 3 = z
1 1

sin2 d cos =

2k 3 2 z p 3 0 (133)

recalling eq. (116) for the radiated power P . Of course, the motion described by eq. (130) has its angular momentum along the +z axis.

Princeton University 2001

Ph501 Set 8, Solution 6

32

6. The magnetic eld radiated by a time-dependent, axially symmetric quadrupole is given by ... [Q] n (134) B= 6c3 r has rectangular components where the unit vector n = (sin , 0, cos ), n and the quadrupole vector Q is related to the quadrupole tensor Qij by Qi = Qij nj . (136) (135)

The charge distribution is symmetric about the z axis, so the quadrupole moment tensor Qij may be expressed entirely in terms of Qzz = Thus, Qij = (3z 2 r2 ) dVol = 4a2 e cos2 t = 2a2 e(1 + cos 2t).

(137)

Qzz /2 0 0

0 Qzz /2 0

0 , 0 Qzz

(138)

and the quadrupole vector can be written as Q = Qzz sin Qzz 3Qzz cos + , 0, Qzz cos = n z 2 2 2 = a2e(1 + cos 2t)( n 3 z cos ). n 3 z cos ), [Q] = 8 3 a2e sin 2t ( where the retarded time is t = t r/c. Hence, 4k 3 a2e [Q] n sin(2kr 2t) sin cos , = y B= 6c3 r r n =y sin . The radiated electric eld is given by since z = E = Bn n = using y l. As we are not using complex notation, we revert to the basic denitions to nd that the time-averaged angular distribution of radiated power is 2ck 6 a4e2 cr2 d P 2 =r S n= EBn = sin2 cos2 , d 4 =n . This integrates to give since ly
1 ... ...

(139) (140)

Then,

(141)

4k 3 a2e l sin(2kr 2t) sin cos , r

(142)

(143)

P = 2

16ck 6 a4e2 d P d cos = . d 15

(144)

Princeton University 2001

Ph501 Set 8, Solution 7

33

7. Since the charge is assumed to rotate with constant angular velocity, the magnetic moment it generates is constant in time, and there is no magnetic dipole radiation. Hence, we consider only electric quadrupole radiation in addition to the electric dipole radiation. The radiated elds are therefore [Q] n [ p] n B= + , c2 r 6c3 r
...

. E = Bn

(145)

The electric dipole radiation elds are given by eqs. (112) and 113) when we write p0 = ae. The present charge distribution is not azimuthally symmetric about any xed axis, so we must evaluate the full quadrupole tensor, Qij = e(3ri rj r2 ij ). (146)

to nd the components of the quadrupole vector Q. The position vector of the charge has components ri = (a cos t, a sin t, 0), (147) so the nonzero components of Qij are Qxx = e(3x2 r2 ) = a2e(3 cos2 t 1) = Qyy Qzz Qxy a2 e (1 + 3 cos 2t), 2 a2 e 2 2 2 2 = e(3y r ) = a e(3 sin t 1) = (1 3 cos 2t), 2 = er2 = a2e, 3a2e = Qyx = 3exy = 3a2e sin t cos t = sin 2t. 2

(148) (149) (150) (151)

Only the time-dependent part of Qij contributes to the radiation, so we write sin 2t 0 cos 2t 3a2 e Qij (time dependent) = sin 2t cos 2t 0 . 2 0 0 0

(152)

towards the observer has components given in eq. (135), so the The unit vector n time-dependent part of the quadrupole vector Q has components 3a2 e (cos 2t sin , sin 2t sin , 0). Qi = Qij nj = 2 Thus, [Qi ] = Qi (t = t r/c) = 12a2 e 3 sin (sin(2kr 2t), cos(2kr 2t), 0).
... ...

(153)

(154)

, y , and It is preferable to express this vector in terms of the orthonormal triad n n , by noting that l=y =n sin x l cos . (155)

Princeton University 2001 Hence,

Ph501 Set 8, Solution 7

34

cos(2kr 2t)). [Q] = 12a2e 3 sin ( n sin sin(2kr 2t) l cos sin(2kr 2t) + y (156) The eld due to electric quadrupole radiation are therefore BE 2 EE 2 2a2ek 3 [Q] n cos sin(2kr 2t)), = sin ( l cos(2kr 2t) + y (157) = 6c3 r r 2a2ek 3 = sin ( y cos(2kr 2t) l cos sin(2kr 2t)). (158) = BE 2 n r
...

...

The angular distribution of the radiated power can be calculated from the combined electric dipole and electric quadrupole elds, and will include a term k 4 due only to dipole radiation as found in prob. 2, a term k 6 due only to quadrupole radiation, and a complicated cross term k 5 due to both dipole and quadrupole eld. Here, we only display the term due to the quadrupole elds by themselves: d PE 2 ca4e2k 6 cr2 = = E E 2 BE 2 n (1 cos4 ), d 4 2 which integrates to give
1

(159)

PE 2 = 2

d P 8ca4 e2k 6 d cos = . d 5

(160)

Princeton University 2001

Ph501 Set 8, Solution 8

35

8. a) The dominant energy loss is from electric dipole radiation, which obeys eq. (25), 2a2 e2 4 dU . = PE 1 = dt 3c3 (161)

For an electron of charge e and mass m in an orbit of radius a about a xed nucleus of charge +e, F = ma tells us that v2 e2 = m = m 2 a, 2 a a e2 , ma3 and also the total energy (kinetic plus potential) is 2 = U = e2 e2 1 2 + mv = a 2 2a so that (162)

(163)

(164)

Using eqs. (163) and (164) in (161), we have dU 2e6 e2 = 4 2 3, = 2a dt 2a 3a m c or 1 da3 4 2 4e4 c, = 2 3 = r0 3 dt 3m c 3 where r0 = e2/mc2 is the classical electron radius. Hence, a2 a =
2 a3 = a3 0 4r0 ct.

(165)

(166)

(167)

The time to fall to the origin is tfall = a3 0 . 2 4r0 c (168)

With r0 = 2.8 1013 cm and a0 = 5.3 109 cm, tfall = 1.6 1011 s. This is of the order of magnitude of the lifetime of an excited hydrogen atom, but the ground state appears to have innite lifetime. This classical puzzle is pursued further in prob. 7. b) The analog of the quadrupole factor ea2 in prob. 5 for masses m1 and m2 in circular 2 2 orbits with distance a between them is m1 r1 + m 2 r2 , where r1 and r2 are measured from the center of mass. That is, m 1 r1 = m 2 r2 , so that r1 = m2 a, m1 + m2 and r1 + r2 = a, m1 a, m1 + m2 (169)

r2 =

(170)

Princeton University 2001 and the quadrupole factor is

Ph501 Set 8, Solution 8

36

2 2 m 1 r1 + m 2 r2 =

m1 m2 2 a. m1 + m2

(171)

We are then led by eq. (26) to say that the power in gravitational quadrupole radiation is 8G m1 m2 2 4 6 PG2 = 5 a . (172) 5 c m1 + m2 We insert a single factor of Newtons constant G in this expression, since it has dimensions of mass2 , and Gm2 is the gravitational analog of the square of the electric charge in eq. (26). We note that a general relativity calculation yields a result a factor of 4 larger than eq. (172): m1 m2 2 4 6 32 G a . (173) PG2 = 5 c5 m 1 + m 2 To nd tfall due to gravitational radiation, we follow the argument of part a):
2 2 Gm1 m2 v1 v2 2 = m = m r = m , 1 1 1 2 a2 r1 r2

(174)

G(m1 + m2) , a3 and also the total energy (kinetic plus potential) is 2 = U = Gm1 m2 1 1 Gm1m2 2 2 + m2 v2 = + m1 v1 a 2 2 2a

so that

(175)

(176)

Using eqs. (175) and (176) in (173), we have


2 32G4 m2 Gm1 m2 dU 1 m 2 (m 1 + m 2 ) a = , = dt 2a2 5a5 c5

(177)

or = a3 a Hence,

1 da4 64G3 m1m2(m1 + m2) . = 4 dt 5c5 256G3 m1m2 (m1 + m2) t. 5c5

(178)

a4 = a4 0 The time to fall to the origin is tfall =

(179)

5 5a4 0c . 256G3 m1m2(m1 + m2)

(180)

For the Earth-Sun system, a0 = 1.5 1013 cm, m1 = 6 1027 gm, m2 = 2 1033 cm, and G = 6.7 1010 cm2/(g-s2 ), so that tfall 1.5 1036 s 5 1028 years!

Princeton University 2001

Ph501 Set 8, Solution 9

37

9. The solution given here follows the succinct treatment by Landau, Classical Theory of Fields, sec. 74. For charges in steady motion at angular frequency in a ring of radius a, the current density J is periodic with period 2/ , so the Fourier analysis (34) at the retarded , where R is time t can be evaluated via the usual approximation that r R r n the distance from the center of the ring to the observer, r points from the center of is the unit vector pointing from the center of the ring the ring to the electron, and n to the observer. Then, [J] = J(r , t = t r/c) = =
m m im r n /c /c Jm (r )eim(tR/c+r n

im(kRt)

J m (r )e

(181)

where k = /c. We rst consider a single electron, whose azimuth varies as = t + 0 , and whose velocity is, of course, v = a . The current density of a point electron of charge e can be written in a cylindrical coordinate system (, , z ) (with volume element d d dz ) using Dirac delta functions as = ev( a)(z )(( t )) . J = chargev 0 The Fourier components Jm are given by 1 Jm = T Also, +sin y ), r = (cos x = sin x +cos n z, and = sin x +cos y . (184)
T 0

(182)

J (r , t )e

imt

eim(0 ) dt = ev( a)(z ) . T

(183)

Using eqs. (183) and (184) in (181) and noting that T = 2, we nd [ J] = ev 2


m

. eim(kRt) eim(0 sin cos /c) ( a)(z )

(185)

Inserting this in eq. (33), we have A =


m

1 cR

[J] d d dz = Am e
imt

ev 2cR

eim(kRt0 )

2 0

d eim(a sin cos /c) (186)

so that the Fourier components of the vector potential are Am = ev im(kR0 ) e 2cR
2 0

+ cos y ) d. eim(v sin cos /c) ( sin x

(187)

Princeton University 2001

Ph501 Set 8, Solution 9

38

The integrals yield Bessel functions with the aid of the integral representation (40). part of eq. (187) can be found by taking the derivative of this relation with The y respect to z : im+1 2 imiz cos J m (z ) = e cos d, (188) 2 0 part of eq. (187) we play the trick For the x 0 =
0 2

ei(mz cos ) d(m z cos )


2

= m so that 1 2
2 0

eimiz cos d + z m 1 z 2
2 0

2 0

eimiz cos sin d, m J m (z ). im z

(189)

eimiz cos sin d =

eimiz cos d =

(190)

Using eqs. (188) and (190) with z = mv sin /c in (187) we have Am = 1 ev im(kR0 ) 1 m+1 Jm (mv sin /c) y . (191) e Jm (mv sin /c) x m cR i v sin /c i

We skip the calculation of the electric and magnetic elds from the vector potential, and proceed immediately to the angular distribution of the radiated power according to eq. (39), dPm cR2 ck 2 m2R2 Am |2 = Am |2 = |imk n |n d 2 2 ck 2 m2R2 = cos2 |Am,x|2 + |Am,y |2 2 ce2k 2 m2 v2 2 2 = cot2 Jm (mv sin /c) + 2 Jm (mv sin /c) . 2 c

(192)

The present interest in this result is for v/c 1, but in fact it holds for any value of v/c. As such, it can be used for a detailed discussion of the radiation from a relativistic electron that moves in a circle, which emits so-called synchrotron radiation. This topic is discussed further in Lecture 20 of the Notes. We now turn to the case of N electrons uniformly spaced around the ring. The initial azimuth of the nth electron can be written 2n . (193) n = N The mth Fourier component of the total vector potential is simply the sum of components (191) inserting n in place of 0: Am = = ev im(kRn ) 1 1 e Jm (mv sin /c) x m+1 Jm (mv sin /c) y (194) m i v sin /c i n=1 cR eveimkR cR 1 1 x m+1 Jm (mv sin /c) y Jm (mv sin /c) m i v sin /c i
N n=1 N

ei2mn/N .

Princeton University 2001

Ph501 Set 8, Solution 9

39

This sum vanishes unless m is a multiple of N , in which case the sum is just N . The lowest nonvanishing Fourier component has order N , and the radiation is at frequency N . We recognize this as N th-order multipole radiation, whose radiated power follows from eq. (192) as ce2k 2 N 2 v2 2 dPN 2 = cot2 JN (Nv sin /c) + 2 JN (Nv sin /c) . d 2 c For large N but v/c (195)

1 we can use the asymptotic expansion (41), and its derivative, (ex/2)m Jm (mx) 2m x (m 1, x 1), (196)

to write eq. (195) as ce2 k 2N dPN 2 2 d 4 sin ev sin 2c


2N

(1 + cos2 )

dPE 1 d

(N

1, v/c

1). (197)

In eqs. (196) and (197) the symbol e inside the parentheses is not the charge but rather the base of natural logarithms, 2.718... For currents in, say, a loop of copper wire, v 1 cm/s, so v/c 1010 , while N 1023 . The radiated power predicted by eq. (197) is extraordinarily small! Note, however, that this nearly complete destructive interference depends on the electrons being uniformly distributed around the ring. Suppose instead that they were distributed with random azimuths n . Then the square of the magnetic eld at order m has the form |Bm |
2 N 2

e
n=1

imn

=N+
l=n

eim(l n ) = N.

(198)

Thus, for random azimuths the power radiated by N electrons (at any order) is just N times that radiated by one electron. If the charge carriers in a wire were localized to distances much smaller than their separation, radiation of steady currents could occur. However, in the quantum view of metallic conduction, such localization does not occur. The random-phase approximation is relevant for electrons in a so-called storage ring, for which the radiated power is a major loss of energy or source of desirable photon beams of synchrotron radiation, depending on ones point of view. We cannot expound here on the interesting topic of the formation length for radiation by relativistic electrons, which length sets the scale for interference of multiple electrons. See, for example, http://puhep1.princeton.edu/~mcdonald/accel/weizsacker.pdf

Princeton University 2001

Ph501 Set 8, Solution 10

40

10. We repeat the derivation of Prob. 1, this time emphasizing the advanced elds. The advanced vector potential for the point electric dipole p = p0 eit located at the origin is AE 1,adv = where k = /c. We obtain the magnetic eld by taking the curl of eq. (199), BE 1,adv = AE 1,adv = ik = k2 Then, EE 1,adv = i 1 ik BE 1,adv = ei(kr+t) 2 + 3 r p0 k r r 1 1 ik ik = ei(kr+t) 2 + 3 (r p0 ) ei(kr+t) 2 + 3 (r p0 ) r r r r 2 k 1 1 ik ik ( n p0 ) + 2p0 ei(kr+t) 2 + 3 = ei(kr+t) + 3 2 + 3 n r r r r r 2 2 ik k k 1 1 ik ) 2 3 p0 . (201) = ei(kr+t) + 3 2 + 3 (p0 n n+ r r r r r r ei(kr+t) ei(kr+t) n p0 = ik ik n p0 r r r (200) (t = t + r/c) } p ei(kr+t) {p ei(kr+t) = = i p0 = ik p0 , cr cr cr r (199)

i ei(kr+t) p0 . 1 + n r kr

The retarded elds due to a point dipole p are, from Prob. 1, BE 1,ret = k 2 EE 1,ret ei(krt) i p0 , n 1+ (202) r kr 1 1 k2 ik k 2 ik ) + 3 2 3 (p0 n + 2 3 p0 . (203) = ei(krt) n r r r r r r

We now consider the superposition of the elds (200)-(203) inside a conducting sphere of radius a. The spatial part of the total electric eld is then EE 1 = 3 k2 ik ) 3 (eikr eikr ) + 3 2 (eikr + eikr ) (p0 n n r r r ik k2 1 3 (eikr eikr ) + 2 (eikr + eikr ) p0 r r r 2 k k 3 ) = 2i n 3 sin kr + 3 2 cos kr (p0 n r r r 1 k k2 3 sin kr + 2 cos kr p0 . 2i r r r

(204)

Princeton University 2001

Ph501 Set 8, Solution 10

41

Remarkably, this electric eld is nite at the origin, although each of the elds (201) and (203) diverges there. We also recognize that this electric eld could be expressed in terms of the so-called spherical Bessel functions, j0 (x) = sin x , x j1 (x) = sin x cos x , x2 x j2 (x) = 1 3 cos x 3 sin x , x3 x x2 ... (205)

An expansion of the spherical cavity eld in terms of spherical Bessel functions occurs naturally when we use the more standard approach to this problem, seeking solutions to the Helmholtz wave equation via separation of variables in spherical coordinates. See Electromagnetic Theory by J.A. Stratton (McGraw-Hill, New York, 1941) for details of this method. Because the sum of the magnetic elds (200) and (202) is purely transverse, this cavity mode is called a TM mode. The boundary conditions at the surface of the sphere are that the radial component of the magnetic eld and the transverse component of the electric eld must vanish. Since the magnetic elds (200) and (202) are transverse at any radius, we examine the electric eld at r = a. Of the terms in eq. (204), only those in p0 have transverse components, so the boundary condition is 0= or cot ka = k2 k 1 cos ka + sin ka 3 , 2 a a a 1 ka ka ka = 2.744. (206)

(207)

In case of a point magnetic dipole m = m0 eit at the origin, the elds have the same form as for an electric dipole, but with E and B interchanged. That is, the advanced elds would be EM 1,adv = k 2 BM 1,adv i ei(kr+t) m0 , 1 + (208) n r kr k2 1 1 ik k 2 ik i(kr +t) ) 2 3 m0(209) = e + 3 2 + 3 (m 0 n n+ , r r r r r r i ei(krt) m0 , 1+ n (210) r kr 1 1 k2 ik k 2 ik ) = ei(krt) n .211) + 3 2 3 (m 0 n + 2 3 m0 ( r r r r r r

and the retarded eld due to magnetic dipole m would be EM 1,ret = k 2 BM 1,ret

If the advanced and retarded magnetic dipole elds are superposed inside a spherical cavity of radius a, the condition that the transverse electric eld vanish at the conducting surface is 0 = eika 1 + i i sin ka + eika 1 = 2 cos ka , ka ka ka (212)

Princeton University 2001 or

Ph501 Set 8, Solution 10

42

tan ka = ka

ka = 4.493.

(213)

The electric eld of this mode is purely transverse, so it is called a TE mode. Clearly, other modes of a spherical cavity can be found by superposing the advanced and retarded elds due to higher multipoles at the origin.

Princeton University 2001

Ph501 Set 8, Solution 11

43

11. This problem is due to D. Iwanenko and I. Pomeranchuk, On the Maximal Energy Attainable in a Betatron, Phys. Rev. 65, 343 (1944).9 The electron is held in its circular orbit by the Lorentz force due to the eld B . Newtons law, F = ma, for this circular motion can be written F = ma = mv 2 v = e B. R c eRB . mc2 (214)

For a relativistic electron, v c, so we have (215)

The electron is being accelerated by the electric eld that is induced by the changing magnetic ux. Applying the integral form of Faradays law to the circle of radius R, we have (ignoring the sign) 2RE = and hence, ave RB , 2c The rate of change of the electrons energy E due to E is E = ave eRB dE = F v ecE = , dt 2 Since E = mc2, we can write mc 2= which integrates to eRBave . (220) 2mc2 Comparing with eq. (215), we nd the required condition on the magnetic eld: = B= Bave . 2 (221) ave eRB , 2 (217) ave R2 B = , c c (216)

(218)

(219)

As the electron accelerates it radiates energy at rate given by the Larmor formula in the rest frame of the electron, dE 2 2e2p 2e2a = = dt 3c3 3c3
2

(222)

Because E and t are both the time components of 4-vectors their transforms from the rest frame to the lab frame have the same form, and the rate dE /dt is invariant.
9

http://puhep1.princeton.edu/~mcdonald/examples/EM/iwanenko_pr_65_343_44.pdf

Princeton University 2001

Ph501 Set 8, Solution 11

44

However, acceleration at right angles to velocity transforms according to a = 2 a. Hence, the rate of radiation in the lab frame is dE 2e4 2 B 2 2e2 4 a2 = , = dt 3c3 3m2c3 using eq. (214) for the acceleration a. The maximal energy of the electrons in the betatron obtains when the energy loss (223) cancels the energy gain (218), i.e., when
2 ave 2e4 2 eRB max B , = 2 3m2 c3

(223)

(224)

and max = ave 3m2 c3 RB = 4e3 B 2 ave Bcrit 3R B 4c B B 3R Bcrit , 4c B (225)

c = 1/137 is the ne structure constant, Bcrit = m2c3 /eh = 4.4 1013 G where = e2 /h is the so-called QED critical eld strength, and is the characteristic cycle time of the ave = B/ . For example, with R = 1 m, = 0.03 sec (30 Hz), betatron such that B 4 and B = 10 G, we nd that max 200, or Emax 100 MeV. We have ignored the radiation due to the longitudinal acceleration of the electron, since in the limiting case this acceleration ceases.

Princeton University 2001

Ph501 Set 8, Solution 12

45

12. Since the dipole is much less than a wavelength away from the conducting plane, the elds between the dipole and the plane are essentially the instantaneous static elds. Thus, charges arranges themselves on the plane as if there were an image dipole at distance d on the other side of the plane. The radiation from the moving charges on the plan is eectively that due to the oscillating image dipole. A distant observer sees the sum of the radiation elds from the dipole and its image.

The image dipole is inverted with respect to the original, i.e., the two dipoles are 180 out of phase. Furthermore, there is a dierence s in path length between the two dipole and the distant observer at angles (, ). We rst calculate in a spherical coordinate system with z axis along the rst dipole, and x axis pointing from the plane to that dipole. Then the path dierence is n = 2d sin cos . s = 2dx (226)

This path dierence results in an additional phase dierence between the elds from the two dipoles at the observer, in the amount = 2 4d s = sin cos . (227)

If we label the electric elds due to the original and image dipoles as E1 and E2, respectively, then the total eld is E = E1 + E2 = E1(1 ei ), and, recalling eq. (46), the power radiated is
2 |E|2 dP1 dP = = 1 ei A sin2 = 2A sin2 (1 cos ) = 4A sin2 sin2 /2 2 d |E1 | d (229) = 4A sin2 sin2 ,

(228)

where =

2d = sin cos . 2

(230)

Suppose we had chosen to use a spherical coordinate system (r, , ) with the z axis pointing from the plane to dipole 1, and the x axis parallel to dipole 1. Then the

Princeton University 2001

Ph501 Set 8, Solution 12

46

phase dierence would have the simple form = 2d = = 2d z n cos , 2 (231)

but the factor sin2 would now become


2 2 2 2 2 2 2 2 sin2 = n2 x + ny = nz + ny = cos + sin cos = 1 sin sin .

(232)

If d = /4, then dP = 4A sin2 sin2 sin cos . d 2 In the side view, = 0, so the pattern has shape sin2 sin2 sin 2 (side view), (233)

(234)

while in the top view, = /2 and the shape is sin2 cos 2 (top view). (235)

This pattern has a single lobe in the forward hemisphere, as illustrated below:

If instead, d = /2, then dP = 4A sin2 sin2 ( sin cos ) . d In the side view, = 0, so the pattern has shape sin2 sin2 ( sin ) (side view), (237) (236)

Princeton University 2001

Ph501 Set 8, Solution 12

47

while in the top view, = /2 and the shape is sin2 ( cos ) (top view). (238)

This pattern does not radiate along the line from the plane to the dipole, as illustrated below:

b) If the electric dipole is aligned with the line from the plane to the dipole, its image has the same orientation.

The only phase dierence between the radiation elds of the dipole and its image is that due to the path dierence , whose value has been given in eqs. (230) and (231). It is simpler to use the angles ( , ) in this case, since the radiation pattern of a single dipole varies as sin2 . Then, E = E1 + E2 = E1 (1 + ei ), and, recalling eq. (46), the power radiated is
2 |E|2 dP1 dP = = 1 + ei A sin2 = 2A sin2 (1 + cos ) = 4A sin2 cos2 /2 2 d |E1| d = 4A sin2 cos2 , (240)

(239)

2d = cos . (241) 2 This radiation pattern is axially symmetric about the line from the plane to the dipole. =

with

Princeton University 2001 If d = /4, then

Ph501 Set 8, Solution 12

48

dP = 4A sin2 cos2 cos d 2

(242)

This pattern is a attened version of the donut pattern sin2 , as illustrated below:

If instead, d = /2, then dP = 4A sin2 cos2 ( cos ) . d (243)

This pattern has a forward lobe for < /6 and a donut for /6 < < /2, as illustrated below:

c) For a magnetic dipole with axis parallel to the conducting plane, the image dipole has the same orientation, the image consists of the opposite charge rotating in the opposite direction, as shown below:

We use angles (, ) and modify the argument of part a) to nd E = E1 + E2 = E1 (1 + ei ), (244)

Princeton University 2001

Ph501 Set 8, Solution 12

49

and, recalling eq. (46), the power radiated is |E|2 dP1 dP i 2 1 + e A sin2 = 2A sin2 (1 + cos ) = 4A cos2 sin2 /2 = = 2 d |E1 | d = 4A sin2 cos2 , (245) where = If d = /4, then dP = 4A sin2 cos2 sin cos . d 2 In the side view, = 0, so the pattern has shape sin2 cos2 sin 2 (side view), (247) 2d = sin cos . 2

(246)

(248)

while in the top view, = /2 and the shape is cos2 cos 2 (top view). (249)

This pattern, shown below, is somewhat similar to that of part a) for d = /2.

If instead, d = /2, then dP = 4A sin2 cos2 ( sin cos ) . d In the side view, = 0, so the pattern has shape sin2 cos2 ( sin ) (side view), (251) (250)

while in the top view, = /2 and the shape is cos2 ( cos ) (top view). (252)

This pattern, shown below, is somewhat similar to that of part b) for d = /2.

Princeton University 2001

Ph501 Set 8, Solution 12

50

Finally, we consider the case of a magnetic dipole aligned with the line from the plane to the dipole, in which case its image has the opposite orientation.

As in part b), the only phase dierence between the radiation elds of the dipole and its image is that due to the path dierence , whose value has been given in eqs. (230) and (231). We use the angles ( , ) in this case, since the radiation pattern of a single dipole varies as sin2 . Then, E = E1 + E2 = E1(1 ei ), and, recalling eq. (46), the power radiated is
2 |E|2 dP1 dP = = 1 ei A sin2 = 2A sin2 (1 cos ) = 4A sin2 sin2 /2 2 d |E1| d (254) = 4A sin2 sin2 ,

(253)

2d (255) = cos . 2 This radiation pattern is axially symmetric about the line from the plane to the dipole. = If d = /4, then dP = 4A sin2 sin2 cos . (256) d 2 This pattern, shown below, is somewhat similar to that of part a) for d = /2.

with

Princeton University 2001

Ph501 Set 8, Solution 12

51

If instead, d = /2, then dP = 4A sin2 sin2 ( cos ) . d (257)

This pattern is qualitatively similar to that for d = /4, shown just above, but the maximum occurs at a larger value of .

Princeton University 2001

Ph501 Set 8, Solution 13

52

13. From p. 191 of the Notes we recall that a single, short, center-fed, linear antenna of dipole moment I0Leit p(t) = i (258) 2 radiates time-averaged power (according to the Larmor formula)
2 2 2 I0 p 2 sin2 L sin2 dU1 = . = dtd 4c3 32c3

(259)

For the record, the current distribution in this short antenna is the triangular waveform I (z, t) = I0eit 1 2 |z | . L (260)

The associated charge distribution (z, t) is related by current conservation, J = , which for a 1-D distribution is simply = so that = and the dipole moment is given by p= as claimed above. Turning to the case to two antennas, we proceed as in the previous problem and write their combined electric eld as E = E1 + E2 = E1(1 ei ), (264)
L/2 L/2

2 I = I0eit , z L 2iI0eit , L I0Leit , 2

(261)

(262)

z dz = i

(263)

where now the phase dierence has contributions due to the path dierence for radiation from the two antennas (whose separation is d = /4), as well as from their intrinsic phase dierence of 90 . That is, = 2 cos + = (1 + cos ). 4 2 2 (265)

From eqs. (259), (264) and (265) we nd


2 2 2 dU1 L sin2 2 I0 dU = 1 ei = (1 cos ) dtd dtd 16c3 2 2 L sin2 2 I0 1 + sin cos . = 3 16c 2

(266)

This angular distribution favors the forward hemisphere, as shown in the sketch:

Princeton University 2001

Ph501 Set 8, Solution 13

53

The total radiated power is


2 2 dU L 2 I0 . = dt 6c3

(267)

is momentum P = U n /c. Thus, the Associated with energy U radiated in direction n angular distribution of radiated momentum is
2 2 dP L sin2 2 I0 = cos 1 + sin 4 dtd 16c 2

. n

(268)

On integrating this over solid angle to nd the total momentum radiated, only the z component is nonzero, dPz dt
2 2 2 I0 L 1 cos cos d cos sin2 1 + sin 4 16c 1 2 2 2 2 2 I0 L 12 12 dU 12 0.26 dU . = 1 = 2 1 2 4 2 2 c c dt c dt

= 2

(269)

The radiation reaction force on the antenna is Fz = dPz /dt. For a broadcast antenna radiating 105 Watts, the reaction force would be only 104 N. The radiation reaction force (269) cannot be deduced as the sum over charges of the /3c3 . Lorentz result is obtained by radiation reaction force of Lorentz, Frad = 2e2 v an integration by parts of the integral of the radiated power over a period. This procedure can be carried out if the power is a sum/integral of a square, as holds for an isolated radiating charge. But it cannot be carried out when the power is the square of a sum/integral as holds for (coherent) radiation by an extended charge/current distribution. Rather, the radiation reaction force on a current distribution must be deduced from the rate of radiation of momentum, as done here.

Princeton University 2001

Ph501 Set 8, Solution 14

54

14. According to p. 181 of the Notes, the power radiated from a known current distribution that oscillates at angular frequency is given by dP 1 = d 8c /c. where k = n We take the z axis along the antenna, so the radiated power due to the current distribution 2z it I (z ) = I0 sin (L/2 < z < L/2), (271) e L is dP d
L/2 1 2z = I0 sin k sin eikz cos dz 8c L/2 L 2 2 k 2 I0 sin2 = , 8c L/2 L/2 2

J (r ) k eikr dVol

(270)

(272)

where =

sin

2z [cos(kz cos ) i sin(kz cos )] dz. L

(273)

For L = , we have k = 2/L and =


/k /k

sin kz [cos(kz cos ) i sin(kz cos )] dz.

(274)

The real part of this integral vanishes, while Im = 2


/k 0

sin kz sin(kz sin ) dz =

2 k

sin x sin(x sin ) dx.

(275)

Using the identity 2 sin A sin B = cos(A B ) cos(A + B ), we have Im 1 {[cos[x(1 + cos )] cos[x(1 cos )]} dx. k 0 1 sin[(1 + cos )] sin[(1 cos )] = k 1 + cos 1 cos 1 sin( cos ) sin( cos ) = k 1 + cos 1 cos 2 sin( cos ) . = k sin2 = dP I 2 sin2( cos ) . = 0 d 2c sin2 (276)

(277)

Inserting this in eq. (272), we nd (278)

Princeton University 2001

Ph501 Set 8, Solution 14

55

The radiation pattern is sketched below:

The total radiated power is, setting cos = u, P =


2 1 1 cos(2u) I 2 1 sin2 ( cos ) I0 dP du d = 0 d cos = d c 1 sin2 2c 1 1 u2 I 2 1 1 cos(2u) I 2 1 1 cos(2u) 1 cos(2u) + du. (279) = 0 du = 0 4c 1 1+u 1u 2c 1 1+u

To cast this in the form of a known special function, we let 1 + u = v/2 , so that P =
2 I0 2c 4 0

1 cos v I2 I2 dv = 0 Cin(4 ) = 3.11 0 , v 2c 2c

(280)

where Cin is the so-called cosine integral. b) To calculate the radiation in the multipole approximation, we need to convert the current distribution I (z )eit to a charge distribution (z, t). This is accomplished via the continuity equation, I = = i. (281) z For the current distribution (271) we nd = 2i 2z it I0 cos e . L L
L/2 L/2

(282)

The dipole moment of this distribution is p= z dz = 0, (283)

so there is no electric dipole radiation. As the current ows along a line, and not in a loop, there is no magnetic dipole radiation either. The charge distribution is symmetric about the z axis, so its tensor quadrupole moment can be characterized in terms of the single quantity Qzz =
L/2 L/2 2

2z 2 dz = 4
0

L/2 0

z 2 dz = 2L i I0 ,
2

8i I0 L

L/2 0

z 2 cos

2z dz L (284)

Li I0 2

x2 cos x dx =

Princeton University 2001 using the integral (55).

Ph501 Set 8, Solution 14

56

The total power radiated by the symmetric quadrupole moment is, according to p. 190 of the Notes, 2 |Qzz |2 6 L4 4 I0 P = = . (285) 240c5 30 2 c4 2c When L = = 2c/ , this becomes P =
2 I2 82 I0 = 5.26 0 . 15 2c 2c

(286)

In this example, higher multipoles must contribute signicantly to the total power, which is given by the exact result (280).

Princeton University 2001

Ph501 Set 8, Solution 15

57

15. The scattering cross section is given by d power scattered into d Sscat(, ) |Escat| = = r2 = r2 , 2 d incident power per unit area Sincident E0 where in the dipole approximation, the far-zone scattered electric eld is Escat = k 2 ei(krt) n m0 ] , [( n p0 ) n r (288)
2

(287)

and p0 eit and m0eit are the electric and magnetic dipole moments induced in the conducting sphere by the incident wave. Because the incident wavelength is large compared to the radius of the sphere, the incident elds are essentially uniform over the sphere, and the induced elds near the sphere are the same as the static elds of a conducting sphere in an otherwise uniform electric and magnetic eld. Then, from p. 57 of the Notes, the induced electric dipole moment is given by (289) p0 = a3 E0. For the induced magnetic dipole, we recall p. 98 of the Notes, remembering that a conducting sphere can be thought of a permeable sphere with zero permeability and a dielectric sphere of innite dielectric constant. Hence, the magnetic dipole moment is m0 = Then a3 B0 . 2 (290)

1 ei(krt) (E 0 n ) + ( n (291) n B0 , r 2 is along the vector r that points from the center of the sphere to the distant where n observer. Escat = k 2a3

For a wave propagating in the +z direction with electric eld linearly polarized along l, and the magnetic eld obeys B0 = direction l, E0 = E0 z E0. Then, Escat = k 2a3E0 ei(krt) ( ) + n ln r ) ( nz ei(krt) 2 3 = k a E0 l 1 r 2

1 ( n z l) 2 z + n ( n l) . 2

(292)

Inserting this in (287) we nd z d n = k 4 a6 1 d 2


2

3 ( n l )2 . 4

(293)

For an observer in the x-z plane, n z = cos . Then for electric polarization parallel to the scattering plane n l = sin , while for polarization perpendicular to the scattering plane n l = 0.

Princeton University 2001

Ph501 Set 8, Solution 15

58

Thus, eq. (293) yields d 1 = k 4 a6 cos d 2 For an unpolarized incident wave, d 1 = d 2 and so = d d + d d = k 4 a6
1 1 2

d cos = a6 k 4 1 d 2

(294)

5 (1 + cos2 ) cos , 8 10a2 4 4 k a . 3

(295)

d 10 4 6 d = k a d 8

(1 + cos2 ) d cos =

(296)

From eqs. (294) we see that d /d is always nonzero, but d /d = 0 for = /3, so for this angle, the scattered radiation is linearly polarized parallel to the scattering plane for arbitrary incident polarization. Addendum: The Fields and Poynting Vector Close to the Sphere Using the results of prob. 2 we can also discuss the elds close to the sphere. In particular, from eqs. (10) and (108) the scattered electric eld at any position r outside the sphere is Escat(r, t) = k 2 i ei(krt) 1 + [3( ( n p0 ) n n p0) n p0 ] 2 2 r k r kr i m0 1+ n kr ei(krt) 1 i + [3( = k 2 a3 ( n E0 ) n n E0) n E0 ] 2 2 r k r kr

Princeton University 2001 +

Ph501 Set 8, Solution 15 i 1 B0 , 1+ n 2 kr

59

(297)

also using eqs. (289) and (290). In this Addendum, we suppose that the electric eld of and B0 = E0 y , while the the incident plane wave is along the x-axis, so that E0 = E0 x point of observation is at r = (r, , ). We express the electric eld vector in spherical coordinates, noting that n x y z Thus, Escat(r, t) = k 2 a3E0 ei(krt) sin cos cos r + sin ) +(2 sin cos r cos cos = = = = r, sin , sin cos r + cos cos + cos , sin sin r + cos sin . cos r sin (298) (299) (300) (301)

1 k 2 r2

i kr

1 i cos sin ) 1+ (cos 2 kr i ei(krt) 1 r = k 2 a3E0 2 sin cos 2 2 r k r kr 1 i 1 i + cos cos 1 2 2 + 1+ k r kr 2 kr cos 1 i i . 1+ sin 1 2 2 + k r kr 2 kr Similarly, using eqs. (8) and (109) the scattered magnetic eld can be written Bscat(r, t) = k 2 i ei(krt) 1 + [3( ( n m0 ) n n m0) n m0 ] 2 2 r k r kr i p0 + 1+ n kr ei(krt) 1 i + [3( = k 2 a 3 ( n B0 ) n n B0 ) n B0 ] 2 2 2r k r kr i E0 2 1 + n kr ei(krt) + cos = k 2a3E0 cos sin 2r i 1 cos ) +(2 sin sin r cos sin 2 2 k r kr i + cos cos ) 2 1 + (sin kr ei(krt) 1 i = k 2a3E0 2 sin sin r 2 2 2r k r kr

(302)

Princeton University 2001

Ph501 Set 8, Solution 15 1

60

+ sin cos 1

i i 2 1+ kr kr 1 i i . 2 cos 1 + + cos 1 2 2 + k r kr kr k 2 r2 +

(303)

On the surface of the sphere, r = a, the scattered electromagnetic elds are, to the leading approximation when ka 1, + sin ), Escat(r = a) E0 eit (2 sin cos r cos cos E0 cos ). Bscat(r = a) eit (2 sin sin r cos sin 2 (304) (305)

In the same approximation, the incident electromagnetic elds at the surface of the sphere are sin ), (306) = E0 eit (sin cos r + cos cos Ein(r = a) E0 eit x + cos ). (307) = E0 eit (sin sin Bin(r = a) E0 eit y r + cos sin Thus, the total electric eld, Etot(r = a) = Ein (r = a) + Escat(r = a) = 3E0 eit sin cos r, on the surface of the sphere is purely radial, and the total magnetic eld, 3 + cos ), Btot(r = a) = Bin(r = a) + Bscat(r = a) = E0 eit (cos sin 2 is purely tangential, as expected for a perfect conductor. The total charge density tot on the surface of the conducting sphere follows from Gauss law as tot = Etot(r = a) 3 3E0 it r = e sin cos = scat, 4 4 2 (310) (309) (308)

where scat is the surface charge density corresponding to the scattered eld (304). Similarly, the total current density Ktot on the surface of the sphere follows from Amp` eres law as Ktot = c 3cE0 it + cos sin ) = 3Kscat, e r Btot(r = a) = ( cos 4 8 (311)

where Kscat is the surface charge density corresponding to the scattered eld (305). We can now discuss the energy ow in the vicinity of the conductor sphere from two perspectives. These two views have the same implications for energy ow in the far zone, but dier in their description of the near zone. First, we can consider the Poynting vector constructed from the total electromagnetic elds, c Etot Btot. (312) Stot = 4

Princeton University 2001

Ph501 Set 8, Solution 15

61

Because the tangential component of the total electric eld vanishes at the surface of the sphere, lines of the total Poynting vector do not begin or end on the sphere, but rather they pass by it tangentially. In this view, the sphere does not absorb or emit energy, but simply redirects (scatters) the ow of energy from the incident wave. However, this view does not correspond closely to the microscopic interpretation that atoms in the sphere are excited by the incident wave and emit radiation as a result, thereby creating the scattered wave. We obtain a second view of the energy ow that better matches the microscopic interpretation if we write Stot = c Etot Btot 4 c = (Ein + Escat) (Bin + Bscat) 4 c c c = Ein Bin + (Ein Bscat + Escat Bin) + Escat Bscat 4 4 4 = Sin + Sinteraction + Sscat. (313)

Since the scattered elds (304)-(305) at the surface of the sphere include tangential components for both the electric and the magnetic eld, the scattered Poynting vector, Sscat, has a radial component, whose time average we wish to interpret as the ow of energy radiated by the sphere. The scattered Poynting vector at any r is given by Sscat c Re(Escat Bscat) 8 c Re (E,scatB,scat E,scatB,scat) r + (E,scatBr,scat Er,scatB,scat) = 8 +(E B,scat E Br,scat) = =
2 c k 4 a6E0 cos2 8 r2
r,scat ,scat

1 cos 2

cos + sin2 1 2 .

1 k 4 r4

cos r + sin 2

(314)

The radial term of eq. (314) in square brackets is identical to the far-zone Poynting vector. However, close to the sphere we nd additional terms in Sscat , so that in the near zone Srad = Sscat . Indeed, at the surface of the sphere we nd Sscat (r = a) c 2 4 4 2 1 = E0 k a cos cos 8 2 cos r + sin 2 c 2 cos . E0 r + sin 8 2

2

cos + sin2 1 2

(315)

Of course, the conducting sphere is not an energy source by itself, and the radiated energy is equal to the energy absorbed from the incident wave. For a description of

Princeton University 2001

Ph501 Set 8, Solution 15

62

the ow of energy that is absorbed, we look to the time-average of the incident and interaction terms of eq. (313). Lines of the incident Poynting vector, Sin = c 2 c 2 ), E0 E (cos z= r sin 8 8 0 (316)

enter and leave the sphere with equal strength, and are therefore not to be associated with energy transfer to the radiation elds. So, we look to the interaction term, Sinteraction = c Re (E,scatB,in + E,inB,scat E,scatB,in E,inB,scat) r 8 +(E,scatBr,in + E,inBr,scat Er,scatB,in Er,inB,scat) B,in + E B,scat E Br,in E Br,scat) +(E
r,scat r,in ,scat ,in

c k a 8 r

2 3

2 E0

cos[kr(1 cos )]

cos 2k 2r2

+(1 + cos ) cos2

1 cos cos + sin2 1 2 2

sin[kr(1 cos )] cos[kr(1 cos )] r kr sin 9 + cos[kr(1 cos )] 2 2 2 cos2 + . . . k r 2 sin 2 sin 2 9 , cos[kr(1 cos )] + ... + 8 k 2 r2

(317)

where the omitted terms are small close to the sphere. Note that in the far zone the time-average interaction Poynting vector contains terms that vary as 1/r times cos[kr(1 cos )]. These large terms oscillate with radius r with period , and might be said to describe a radial sloshing of energy in the far zone, rather than a radial ow. It appears in practice that one cannot detect this sloshing by means of a small antenna placed in the far zone, so we consider these terms to be unphysical. Nonetheless, it is interesting that they appear in the formalism. At the surface of the sphere we have, again for ka Sinteraction(r = a) = 1,

c 2 cos 9 . + 9 sin 2 sin 2 E0 r + sin 2 cos2 8 2 2 8 (318) The total Poynting vector on the surface of the sphere is the sum of eqs. (315), (316) and (318), Stot (r = a) = c 2 9 . + 9 sin 2 sin 2 E0 sin cos2 8 2 8 (319)

The radial component of the total Poynting vector vanishes on the surface of the sphere, as expected for a perfect conductor.

Princeton University 2001

Ph501 Set 8, Solution 15

63

This exercise permits an additional perspective, or possible relevance to thinking about radiation from antennas. Suppose that instead of knowing that a plane wave was incident on the conducting sphere, we were simply given the surface current distribution Kscat of eq. (311). Then, by use of retarded potentials, or the antenna formula (63), we could calculate the radiated power in the far zone, and would arrive at the usual expression (314) (ignoring the terms that fall o as 1/r6 ). However, this procedure would lead to an incomplete understanding of the near zone. In particular, the excitation of the conducting sphere by an external plane wave leads to a total surface current that is three times larger than the current Kscat. For a good, but not perfectly conducting sphere, an analysis based on Kscat alone would lead to only 1/9 the actual amount of Joule heating of the sphere. And, if we attempted to assign some kind of impedance 2 or radiation resistance to the sphere via the form Prad = RradI0 /2 where I0 is meant to be a measure of the peak total current, an analysis based only on knowledge the current Kscat would lead to a value of Rrad that is 9 times larger than desired.

Princeton University 2001

Ph501 Set 8, Solution 16

64

16. The possibility of radiation from superluminal sources was rst considered by Heaviside in 1888. He considered this topic many times over the next 20 years, deriving most of the formalism of what is now called Cerenkov radiation. However, despite being an early proponent of the concept of a velocity-dependent electromagnetic mass, Heaviside never acknowledged the limitation that massive particles must have velocities less than that of light. Consequently many of his pioneering eorts (and those of his immediate followers, Des Coudres and Sommerfeld), were largely ignored, and the realizable case of radiation from a charge with velocity greater than the speed of light in a dielectric medium was discovered independently in an experiment by Cerenkov in 1934.10 An insightful discussion of the theory of Cerenkov radiation by Tamm (J. Phys. U.S.S.R. 11 1, 439 (1939), in English!) revealed its close connection with what is now called transition radiation, i.e., radiation emitted by a charge in uniform motion that crosses a boundary between metallic or dielectric media. The present problem was inspired by a work of Bolotovskii and Ginzburg, Sov. Phys. Uspekhi 15, 184 (1972),12 on how aggregates of particles can act to produce motion that has superluminal aspects and that there should be corresponding Cerenkov-like radiation in the case of charged particles. The classic example of aggregate superluminal motion is the velocity of the point of intersection of a pair of scissors whose tips approach one another at a velocity close to that of light. Here we consider the example of a sweeping electron beam in a high-speed analog oscilloscope such as the Tektronix 7104. In this device the writing speed, the velocity of the beam spot across the faceplate of the oscilloscope, can exceed the speed of light. The transition radiation emitted by the beam electrons just before they disappear into the faceplate has the character of Cerenkov radiation from the superluminal beam spot, according to the inverse of the argument of Tamm. Referring to the gure in the statement of the problem, the line of charge has equation y= so the current density is J = yNe(z ) t x y + , v u (321) u x ut, v z = 0, (320)

where N is the number of electrons per unit length intercepting the x axis, and e < 0 is the electrons charge. We also consider the eect of the image current, Jimage = +y(Ne)(z ) t x y . v u (322)

We will nd that to a good approximation the image current just doubles the amplitude of the radiation. For u c the image current would be related to the retarded elds
10 11

http://puhep1.princeton.edu/~mcdonald/examples/EM/cerenkov_pr_52_378_37.pdf http://puhep1.princeton.edu/~mcdonald/examples/EM/tamm_jpussr_1_439_39.pdf 12 http://puhep1.princeton.edu/~mcdonald/examples/EM/bolotovskii_spu_15_184_72.pdf

Princeton University 2001

Ph501 Set 8, Solution 16

65

of the electron beam, but we avoid this complication when u c. Note that the true current exists only for y > 0, while the image current applies only for y < 0. We insert the current densities (321) and (322) into eq. (63) and integrate using rect given by angular coordinates, with components of the unit vector n nx = cos , ny = sin cos , and nz = sin sin , (323)

as indicated in part b) of the gure. The current impinges only on a length L along the x axis. The integrals are elementary and we nd, noting /c = 2/,
L c dU e2N 2 L2 u2 cos2 + sin2 sin2 sin ( v cos ) = . 2 L c dd 2 c c2 (1 u ( cos ) sin2 cos2 )2 v c2

(324)

The factor of form sin2 /2 appears from the x integration, and indicates that this leads to a single-slit interference pattern. We will only consider the case that u 2 1 u sin2 cos2 by 1. c2 c, so from now on we approximate the factor

Upon integration over the azimuthal angle from /2 to /2 the factor cos2 + (1 + cos2 ). sin2 sin2 becomes 2 It is instructive to replace the radiated energy by the number of radiated photons: dU = h dN . Thus sin L ( c cos ) d 2 2 u2 dN v 2 , = N L 2 (1 + cos ) L c d cos 2 c ( cos ) v
2

(325)

where = e2/h c 1/137. This result applies whether v < c or v > c. But for v < c, the argument = L ( c cos ) can never become zero, and the diraction pattern v never achieves a principal maximum. The radiation pattern remains a slightly skewed type of transition radiation. However, for v > c we can have = 0, and the radiation pattern has a large spike at angle C such that c cos C = , v which we identify with Cerenkov radiation. Of course the side lobes are still present, but not very prominent. Discussion The present analysis suggests that Cerenkov radiation is not really distinct from transition radiation, but is rather a special feature of the transition radiation pattern which emerges under certain circumstances. This viewpoint actually is relevant to Cerenkov radiation in any real device which has a nite path length for the radiating charge. The walls which dene the path length are sources of transition radiation which is always present even when the Cerenkov condition is not satised. When the Cerenkov condition is satised, the so-called formation length for transition radiation becomes

Princeton University 2001

Ph501 Set 8, Solution 16

66

longer than the device, and the Cerenkov radiation can be thought of as an interference eect. If L/ 1, then the radiation pattern is very sharply peaked about the Cerenkov angle, and we may integrate over noting d = to nd dN L d cos and

sin2 = 2

(326)

c2 d L u2 1 + . (N)2 2 c2 v2

(327)

In this we have replaced cos2 by c2/v 2 in the vicinity of the Cerenkov angle. We have also extended the limits of integration on to [, ]. This is not a good approximation for v < c, in which case > 0 always and dN is much less than stated. For v = c the radiation rate is still about one half of the above estimate. For comparison, the expression for the number of photons radiated in the ordinary Cerenkov eect is d L 2 (328) sin C dN 2 . The ordinary Cerenkov eect vanishes as 2 near the threshold, but the superluminal C eect does not. This is related to the fact that at threshold ordinary Cerenkov radiation is emitted at small angles to the electrons direction, while in the superluminal case the radiation is at right angles to the electrons motion. In this respect the moving spot on an oscilloscope is not fully equivalent to a single charge as the source of the Cerenkov radiation. In the discussion thus far we have assumed that the electron beam is well described by a uniform line of charge. In practice the beam is discrete, with uctuations in the spacing and energy of the electrons. If these uctuations are too large we cannot expect the transition radiation from the various electrons to superimpose coherently to produce the Cerenkov radiation. Roughly, there will be almost no coherence for wavelengths smaller than the actual spot size of the electron beam at the metal surface, Thus there will be a cuto at high frequencies which serves to limit the total radiated energy to a nite amount, whereas the expression derived above is formally divergent. Similarly the eect will be quite weak unless the beam current is large enough that N 1. We close with a numerical example inspired by possible experiment. A realistic spot size for the beam is 0.3 mm, so we must detect radiation at longer wavelengths. A convenient choice is = 3 mm, for which commercial microwave receivers exist. The bandwidth of a candidate receiver is d/ = 0.02 centered at 88 GHz. We take L = 3 cm, so L/ = 10 and the Cerenkov cone will actually be about 5 wide, which happens to match the angular resolution of the microwave receiver. Supposing the electron beam energy to be 2.5 keV, we would have u2/c2 = 0.01. The velocity of the moving spot is taken as v = 1.33c = 4 1010 cm/sec, so the observation angle is 41 . If the electron beam current is 1 A then the number of electrons deposited per cm along the metal surface is N 150, and N 45.

Princeton University 2001

Ph501 Set 8, Solution 16

67

Inserting these parameters into the rate formula we expect about 7 103 detected photons from a single sweep of the electron beam. This supposes we can collect over all azimuth which would require some suitable optics. The electron beam will actually be swept at about 1 GHz, so we can collect about 7 106 photons per second. The corresponding signal power is 2.6 1025 Watts/Hz, whose equivalent noise temperature is about 20 mK. This must be distinguished from the background of thermal radiation, the main source of which is in the receiver itself, whose noise temperature is about 100 K. A lock-in amplier could be used to extract the weak periodic signal; an integration time of a few minutes of the 1-GHz-repetition-rate signal would suce assuming 100% collection eciency. Realization of such an experiment with a Tektronix 7104 oscilloscope would require a custom cathode ray tube that permits collection of microwave radiation through a portion of the wall not coated with the usual metallic shielding layer. Bremsstrahlung Early reports of observation of transition radiation were considered by skeptics to be due to bremsstrahlung instead. The distinction in principle is that transition radiation is due to acceleration of charges in a medium in response to the far eld of a uniformly moving charge, while bremsstrahlung is due to the acceleration of the moving charge in the near eld of atomic nuclei. In practice both eects exist and can be separated by careful experiment. Is bremsstrahlung stronger than transition radiation in the example considered here? As shown below the answer is no, but even if it were we would then expect a Cerenkovlike eect arising from the coherent bremsstrahlung of the electron beam as it hits the oscilloscope faceplate. The angular distribution of bremsstrahlung from a nonrelativistic electron will be sin2 with dened with respect to the direction of motion. The range of a 2.5-keV electron in, say, copper is about 5 106 (as extrapolated from the table on p. 240 of Studies in Penetration of Charged Particles in Matter, National Academy of Sciences National Research Council, PB-212 907 (Washington, D.C., 1964)), while the skin depth at 88 GHz is about 2.5 105 cm. Hence the copper is essentially transparent to the backward hemisphere of bremsstrahlung radiation, which will emerge into the same half space as the transition radiation. The amount of bremsstrahlung energy dUB emitted into energy interval dU is just Y dU where Y is the so-called bremsstrahlung yield factor. For 2.5-keV electrons in copper, Y = 3 104 . The number dN of bremsstrahlung photons of energy h in a bandwidth d/ is then dN = dUB /h = Y d/ . For the 2% bandwidth of our example, dN = 6 106 per beam electron. For a 3-cm-long target region there will be 500 beam electrons per sweep of the oscilloscope, for a total of 3 104 bremsstrahlung photons into a 2% bandwidth about 88 GHz. Half of these emerge from the faceplate as a background to 7 103 transition-radiation photons per sweep. Altogether, the bremsstrahlung contribution would be about 1/50 of the transition-radiation signal in the proposed experiment.

Princeton University 2003 Reading: Griths secs. 9.4-9.5. 1. Griths prob. 9.19.

Ph304 Problem Set 10

2. Griths prob. 9.22. In part a), use dimensional analysis to determine the functional form of the velocity of deep-water waves, which might depend on the wavelength , the density of water, and the acceleration g of gravity (which provides the restoring force against wavy deformations), but which does not depend on the depth of the water. 3. Variant of Griths prob. 9.25. Consider a medium that is suciently well described by a single resonance of angular frequency ! 0, and with a nonzero absorption coecient . The oscillator strength may be taken as unity. Show that at the central frequency, ! = ! 0, the group velocity is given by vg = where !
p

c 1 = ; 2 = 2 dk=d! 1 ! p

is the so-called plasma frequency, !


2 p

N q2 : m 0

If it happens that < ! p, then the group velocity is negative! However, since this bizarre eect occurs for frequencies where absorption is very strong, it was thought to be largely irrelevant. But recent experiments involving a pair of closely spaced absorption lines that are pumped into inverted populations has been able to demonstrate negative group velocity without attenuation. The concept of negative group velocity requires careful thought, as one can be misled into thinking it implies faster-than-light eects. See, http://puhep1.princeton.edu/mcdonald/examples/negativegroupvelocity.pdf A related, but less controversial, phenomenon attainable with pumped pairs of absorption lines is slow light: http://puhep1.princeton.edu/mcdonald/examples/slowlight.pdf 4. Griths prob. 9.29. You are asked to compute the so-called energy velocity, hSi da ; vE = R hui da and compare this with the group velocity vg . 5. Griths prob. 9.30. 6. Griths prob. 9.38.
R

Princeton University 2003 Reading: Griths secs. 9.4-9.5. 1. Griths prob. 9.19.

Ph304 Problem Set 10

2. Griths prob. 9.22. In part a), use dimensional analysis to determine the functional form of the velocity of deep-water waves, which might depend on the wavelength , the density of water, and the acceleration g of gravity (which provides the restoring force against wavy deformations), but which does not depend on the depth of the water. 3. Variant of Griths prob. 9.25. Consider a medium that is suciently well described by a single resonance of angular frequency ! 0, and with a nonzero absorption coecient . The oscillator strength may be taken as unity. Show that at the central frequency, ! = ! 0, the group velocity is given by vg = c 1 = 2= 2; dk=d! 1 ! p

where ! p is the so-called plasma frequency, 2 2 = Nq : !p m 0 If it happens that ! p, then the group velocity is negative! However, since this bizarre eect occurs for frequencies where absorption is very strong, it was thought to be largely irrelevant. But recent experiments involving a pair of closely spaced absorption lines that are pumped into inverted populations has been able to demonstrate negative group velocity without attenuation. The concept of negative group velocity requires careful thought, as one can be misled into thinking it implies faster-than-light eects. See, http://puhep1.princeton.edu/mcdonald/examples/negativegroupvelocity.pdf A related, but less controversial, phenomenon attainable with pumped pairs of absorption lines is slow light: http://puhep1.princeton.edu/mcdonald/examples/slowlight.pdf 4. Griths prob. 9.29. You are asked to compute the so-called energy velocity, R hSi da ; vE = R h ui da and compare this with the group velocity vg. 5. Griths prob. 9.30. 6. Griths prob. 9.38.

Princeton University 2003 Reading: Griths chap. 10.

Ph304 Problem Set 11

1. Griths prob. 10.9, part a) only. Verify that the electric and magnetic elds rst seen by an observer at distance r from the wire obey the radiation condition E = cB r. That is, consider the elds at time r=c + . Show also that for t r=c the magnetic eld is that given by the instantaneous magnetostatic value: B 0kt=2r. However, the electric eld never drops to its instantaneous static value (zero), because the ever increasing magnetic eld continually induces more electric eld. 2. Griths prob. 10.14. It may be simpler to proceed from eq. (10.39) than from (10.42). 3. Griths prob. 10.24. Hint: At what time does the observer at the origin rst become aware of the elds of the moving charge? 4. Griths prob. 10.26. For another example of how electromagnetic eld momentum is needed for Newtons 3rd law to be satised in electrodynamics, see http://puhep1.princeton.edu/mcdonald/examples/transmom2.pdf 5. Griths prob. 11.3. The dipole is that of sec. 11.1.2. Verify that
q

0=0 = 377 .

6. Griths prob. 11.4. Besides following Griths hint, another way to work the problem )e i!t , and use Griths eqs. (11.56) is to write the dipole moment as p = p0( x + iy and (11.57).

Princeton University 2003 Reading: Griths chap. 10.

Ph304 Problem Set 11

1. Griths prob. 10.9, part a) only. Verify that the electric and magnetic elds rst seen by an observer at distance r from the wire obey the radiation condition E = cB r. That is, consider the elds at time r=c+ . Show also that for t r=c the magnetic eld is that given by the instantaneous magnetostatic value: B 0kt= 2r . However, the electric eld never drops to its instantaneous static value (zero), because the ever increasing magnetic eld continually induces more electric eld. 2. Griths prob. 10.14. It may be simpler to proceed from eq. (10.39) than from (10.42). 3. Griths prob. 10.24. Hint: At what time does the observer at the origin rst become aware of the elds of the moving charge? 4. Griths prob. 10.26. For another example of how electromagnetic eld momentum is needed for Newtons 3rd law to be satised in electrodynamics, see http://puhep1.princeton.edu/mcdonald/examples/transmom2.pdf q 5. Griths prob. 11.3. The dipole is that of sec. 11.1.2. Verify that 0= 0 = 377 . 6. Griths prob. 11.4. Besides following Griths hint, another way to work the problem )ei!t , and use Griths eqs. (11.56) is to write the dipole moment as p = p0( x + iy and (11.57).

Вам также может понравиться